Вы находитесь на странице: 1из 68

A 36-year-old nulliparous woman presents to her primary care physician 36-летняя нулевая женщина представляет своему врачу первичной

врачу первичной медицинской


reporting breast pain of several days’ duration. She denies a history of breast помощи сообщение о боли в груди продолжительностью в несколько дней. Она
cancer in her family but is very worried about the possibility of malignancy. отрицает историю рака молочной железы в своей семье, но очень беспокоится о
There is no warmth, swelling, or erythema; and there is no bleeding or discharge возможности малигнизации. Там нет тепла, отек или эритема; и нет никаких
at the nipples. There is no cervical or axillary lymphadenopathy. Breast кровотечений или выделений на сосках. Цервикальной или подмышечной
examination reveals several small, mobile lumps on both breasts. The patient лимфаденопатии нет. Обследование молочных желез выявляет несколько
states that she thinks she may have felt similar lumps before around the time of небольших подвижных комков на обеих молочных железах. Пациентка заявляет,
her periods. что, по ее мнению, она могла чувствовать схожие комки раньше во время
менструации.
Which of the following would a biopsy of the breast lumps most likely reveal?
Что из нижеперечисленного наиболее вероятно выявит при биопсии комков
молочной железы?
Central necrosis
Центральный некроз
*Increase in number of acini and intralobular fibrosis
*Увеличение числа ацинусов и интралобулярного фиброза
Large cells with clear "halos"
Большие клетки с четкими "ореолами"
Lymphatic involvement
Лимфатическое поражение
Sheets of pleomorphic cells infiltrating adjacent stroma
Листы плеоморфных клеток, проникающих в соседнюю строму
This patient presents with cyclic, menstruation-associated breast pain without
any additional symptoms. On examination, she is found to have multiple bilateral Этот пациент испытывает циклическую, связанную с менструацией боль в груди без
small, mobile lumps. This is the classic presentation of fibrocystic changes of каких-либо дополнительных симптомов. При осмотре у нее обнаружены
the breast. множественные двусторонние маленькие подвижные комочки. Это классическое
представление фиброзно-кистозных изменений молочной железы.
Fibrocystic disease is a benign condition and manifests with diffuse breast pain
and multiple bilateral masses, commonly found in the upper outer quadrants of Фиброзно-кистозная болезнь представляет собой доброкачественное состояние и
the breast. It is one of the most common causes of breast lumps in women from проявляется диффузной болью в груди и множественными двусторонними массами,
the age of 35 years to the onset of menopause. Common risk factors include обычно встречающимися в верхних наружных квадрантах молочной железы. Это
nulliparity and being age 35 or older. Fibrocystic changes can be characterized одна из наиболее распространенных причин образования опухолей у женщин в
histologically as fibrosis (fibrous stroma, indicated by the red arrow in the image) возрасте от 35 лет до наступления менопаузы. Общие факторы риска включают в
and cysts (indicated by the black arrow in the image). Fibrocystic changes can себя нулевой статус и возраст 35 лет и старше. Фиброзно-кистозные изменения
cause masses that fluctuate in size with menstruation and caffeine intake. можно охарактеризовать гистологически как фиброз (фиброзная строма,
Proliferation of the acini may be seen in patients with fibrocystic disease; if обозначенная красной стрелкой на изображении) и кисты (обозначенные черной
present, it may be associated with a low risk of carcinoma. стрелкой на изображении). Фиброзно-кистозные изменения могут вызывать
Central necrosis is characteristic of comedocarcinoma, a subtype of ductal изменения массы при менструации и приеме кофеина. Пролиферация ацинусов
carcinoma in situ (DCIS). Comedocarcinoma would present with a palpable может наблюдаться у пациентов с фиброзно-кистозной болезнью; если он
mass, nipple discharge, or Paget disease. This patient has no nipple discharge присутствует, это может быть связано с низким риском карциномы.
and no evidence of Paget disease. Large cells with clear ?halos? refer to Paget Центральный некроз характерен для комедокарциномы, подтипа протоковой
disease, which presents with unilateral eczematous nipple changes. This patient карциномы in situ (DCIS). Комедокарцинома может проявляться пальпируемой
does not have any nipple or skin changes. Lymphatic involvement is indicative массой, выделениями из сосков или болезнью Педжета. У этого пациента нет
of inflammatory carcinoma, which results in a peau d’orange appearance of the выделений из сосков и нет признаков болезни Педжета. Большие клетки с
skin. This patient does not have any erythema, swelling, warmth, or skin прозрачными «ореолами» обратитесь к болезни Педжета, которая проявляется
changes, making lymphatic involvement unlikely. Sheets of pleomorphic cells односторонними экзематозными изменениями сосков. У этого пациента нет никаких
infiltrating adjacent stroma describes the histology of invasive ductal carcinoma. изменений соска или кожи. Поражение лимфатических узлов свидетельствует о
Invasive ductal carcinoma can present as a firm, irregular, and fixed mass; воспалительной карциноме, которая приводит к появлению на коже оранжевого
unilateral bloody nipple discharge; and skin changes; this patient has none of цвета. У этого пациента нет эритемы, отечности, тепла или изменений кожи, что
these findings. делает маловероятным поражение лимфатических узлов. Листы плеоморфных
клеток, инфильтрирующих соседнюю строму, описывают гистологию инвазивного
протокового рака. Инвазивная протоковая карцинома может представлять собой
твердую, нерегулярную и фиксированную массу; односторонние кровянистые
выделения из сосков; и изменения кожи; у этого пациента нет ни одного из этих
результатов.
A 63-year-old woman presents to her physician with a 2-month history of weight 63-летняя женщина представляет своему врачу двухмесячную историю увеличения
gain and heaviness around her abdomen. Upon further examination, the веса и тяжести в области живота. При дальнейшем осмотре обнаруживается, что
abdominal expansion is found to be due to mucinous fluid build-up in the intra- расширение брюшной полости происходит из-за скопления муциновой жидкости во
abdominal cavity. внутрибрюшной полости.
Mucinous build-up in the intra-abdominal cavity is most consistent with which of Наращивание слизистой во внутрибрюшной полости наиболее согласуется с тем,
the following? что из следующего?

A.Endometrioma эндометриома
B.Hepatic congestion
Застой в печени
C.Pseudomyxoma peritonei
*Псевдомиксома брюшины
D.Struma ovarii
Струма яичников
Pseudomyxoma peritonei is the filling of the intra-abdominal cavity upon rupture
of a mucinous tumors. The clinical buzzword that is usually used to describe the Pseudomyxoma peritonei - это заполнение внутрибрюшной полости при разрыве
presentation is "jelly belly." Pseudomyxoma peritonei occurs most commonly слизистых опухолей. Клиническое модное словосочетание, которое обычно
secondary to appendiceal mucinous tumors and, in a minority of cases, primary используется для описания презентации, - «животик желе». Pseudomyxoma peritonei
ovarian tumors. Symptoms are fairly nonspecific, and will typically include чаще всего возникает вторично по отношению к слизистым опухолям
abdominal pain, cramping, loss of appetite (mass effect due to tumor growth), червеобразного отростка и, в меньшинстве случаев, к первичным опухолям
constipation, and weight loss. яичников. Симптомы довольно неспецифичны и, как правило, включают боль в
животе, судороги, потерю аппетита (эффект массы вследствие роста опухоли),
запоры и потерю веса.
A 33-year-old woman with no significant medical history presents to her 33-летняя женщина без значительного анамнеза предъявляет гинекологу для
gynecologist for a routine examination. Speculum examination reveals a normal- обычного обследования. При осмотре зеркала выявляется нормально выглядящая
appearing cervix and vaginal canal with no lesions or discharge. Bimanual шейка матки и влагалищный канал без каких-либо повреждений или выделений. При
examination reveals a firm, nongravid uterus of normal size with a notable бимануальном исследовании выявляется твердая, невесомая матка нормального
discrepancy in the size of the ovaries. Ultrasound confirms the presence of a размера с заметным несоответствием размеров яичников. УЗИ подтверждает
mass on the left ovary. A biopsy of the left ovarian mass is performed, and the наличие массы на левом яичнике. Выполнена биопсия массы левого яичника, и на
pathology slide is shown in the image. снимке показан слайд с патологией.
Какое из следующих утверждений наиболее соответствует клинической картине
этой опухоли?

Эти опухоли связаны с фибромой, асцитом и гидротораксом

Эти опухоли часто связаны с односторонней, мультилокулярной массой яичников

Эти опухоли иногда связаны с аритмиями, особенно мерцательной аритмией

*Эти опухоли обычно доброкачественные

Опухоли этого типа секретируют эстрогены и связаны с преждевременным половым


созреванием

Этот пациент молодой и не имеет симптомов. Она представляет для обычного


Image courtesy of Dr. Michael Bonert гинекологического обследования, и обнаруживается несоответствие в размере
Which of the following statements is most consistent with this tumor’s clinical яичников. Ультразвук выявляет случайную яичниковую массу. Это общая картина
presentation? опухоли Бреннера, которая представляет собой доброкачественную опухоль
яичника, состоящую из клеток, которые напоминают переходный эпителий мочевого
A.These tumors are associated with fibroma, ascites, and hydrothorax пузыря.
B.These tumors are often associated with a unilateral, multilocular Классический образец биопсии опухоли Бреннера, как показано на рисунке,
ovarian mass обнаруживает гнезда переходных клеток (обозначены кружками) с ядрами в форме
кофейных зерен среди фиброзной стромы. Эпителиальные опухоли составляют 85%
C.These tumors are sometimes associated with arrhythmias, notably
опухолей яичников, причем серозные эпителиальные опухоли являются наиболее
atrial fibrillation
распространенными эпителиальными опухолями, а опухоли Бреннера - наименее
D.These tumors are usually benign распространенными. Другие типы эпителиальных опухолей включают слизистые,
эндометриоидные, прозрачные клетки / мезонефрические и смешанные подтипы.
E.Tumors of this type secrete estrogen and are associated with
precocious puberty У этого пациента нет симптомов, поэтому некоторые другие варианты ответа могут
быть неправильными, поскольку они связаны с конкретными симптомами. Синдром
This patient is young and has no symptoms. She presents for a routine Мейгса, который характеризуется классической триадой фибромы, асцита и
gynecologic examination, and a discrepancy is found in the size of the ovaries. гидроторакса, наблюдается при фибромах яичников. Муцинозные цистаденомы
Ultrasound reveals an incidental ovarian mass. This is the general presentation яичников связаны с односторонней, множественной массой яичников. Они могут
of a Brenner tumor, which is a benign ovarian tumor composed of cells that вызвать вздутие живота, увеличение размера талии, увеличение веса и боли в
resemble bladder transitional epithelium. The classic biopsy specimen of a животе. Опухоли, связанные с аритмиями, относятся к язве зоба, которая может
Brenner tumor, as shown in the image, reveals nests of transitional cells вызывать симптомы гипертиреоза, включая непереносимость тепла, тахикардию,
(identified by the circles) with coffee bean–shaped nuclei among a fibrous диарею и тремор. Опухоли, которые выделяют эстрогены, относятся к гранулезным
stroma. Epithelial tumors comprise 85% of ovarian tumors, with serous epithelial клеточным опухолям, которые могут вызывать гиперплазию эндометрия и рак
tumors being the most common epithelial tumor and Brenner tumors being the эндометрия. Ультразвуковое исследование этого пациента не выявило каких-либо
least common. Other types of epithelial tumors include mucinous, endometrioid, патологий эндометрия.
clear cell/mesonephric, and mixed subtypes. This patient has no symptoms, so
some of the other answer options would be incorrect because they are
associated with specific symptoms. Meigs syndrome, which is characterized by
the classic triad of fibroma, ascites, and hydrothorax, is seen with ovarian
fibromas. Ovarian mucinous cystadenomas are associated with a unilateral,
multilocular ovarian mass. They can cause bloating, increase in waist size,
weight gain, and abdominal pain. Tumors associated with arrhythmias refer
to struma ovarii, which can cause symptoms of hyperthyroidism including heat
intolerance, tachycardia, diarrhea, and tremor. Tumors that secrete estrogen
refer to granulosa cell tumors, which can cause endometrial hyperplasia and
endometrial carcinomas. Ultrasound examination of this patient did not reveal
any endometrial abnormalities.
51-летняя женщина, гравида 3, параграф 3, приходит в клинику из-за прерывистого
A 51-year-old woman, gravida 3 para 3, comes to the clinic because of безболезненного влагалищного кровотечения в течение последнего месяца. Ее
intermittent painless vaginal bleeding during the past month. Her last menstrual последний менструальный цикл был 2 года назад, и в остальном она здорова. Ее
cycle was 2 years ago, and she is otherwise healthy. Her medical history история болезни включает полипы матки, которые были удалены 8 лет назад и
includes uterine polyps that were removed 8 years ago and Papanicolaou smear мазок Папаниколау 2 года назад с совместным тестированием на вирус папилломы
2 years ago with human papillomavirus co-testing that was negative. She does человека, которое было отрицательным. Она не принимает гормонозаместительную
not take hormone-replacement therapy. On physical examination, her abdomen терапию. При физикальном осмотре ее живот мягкий и безболезненный, без вздутия
is soft and nontender without distension or masses. A pelvic examination или массы. При осмотре органов малого таза выявляется масса непонятного левого
reveals a nontender mass in the left adnexa. Transvaginal ultrasound reveals a придатка. Трансвагинальное УЗИ выявляет 9-миллиметровую полосу эндометрия и
9-mm endometrial stripe and 7-cm cystic left ovarian mass. Subsequent 7-сантиметровую кистозную массу левого яичника. Последующая диагностическая
diagnostic laparoscopy reveals a firm, yellow, 7-cm ovarian mass. лапароскопия выявляет твердую желтую яичниковую массу 7 см. Представительные
Representative images of the mass are shown. изображения массы показаны.

Which of the following is most likely secreted by this patient's tumor?


A.Alpha-fetoprotein

B.Beta-hCG

C.CA-125

D.Estrogen
Что из следующего наиболее вероятно секретируется опухолью этого пациента?
E.Lactate dehydrogenase Альфа-фетопротеин
Бета-ХГЧ
CA-125
F.Testosterone *эстроген
Лактатдегидрогеназа
G.Thyroxine Тестостерон
This patient is a 51-year-old woman has postmenopausal bleeding, endometrial тироксин
hyperplasia (evidenced by thickened endometrial stripe on ultrasound), and an
ovarian mass. The appearance of a firm, yellow ovarian mass on laparoscopy is Этот пациент 51-летней женщины страдает постменопаузальным кровотечением,
most suggestive a granulosa cell tumor (GCT). гиперплазией эндометрия (о чем свидетельствует утолщенная полоса эндометрия
GCTs are large, unilateral ovarian masses that produce excess при УЗИ) и массой яичников. Появление твердой желтой массы яичников при
estrogen. Estrogen stimulates endometrial growth, which can lead to лапароскопии наиболее показательно при опухолях гранулезных клеток (GCT).
hyperplasia and present clinically as a thickened endometrial stripe on
ultrasound and/or with postmenopausal bleeding. Overall these patients have an GCT представляют собой большие односторонние массы яичников, которые
increased risk for endometrial carcinoma and must be evaluated promptly with продуцируют избыток эстрогена. Эстроген стимулирует рост эндометрия, что
endometrial biopsy. Notably, GCTs can also present in young girls and will может привести к гиперплазии и клинически проявляться в виде утолщенной
symptomatically manifest as precocious puberty. полоски эндометрия на ультразвуке и / или при постменопаузальном кровотечении.
Granulosa cell tumors classically demonstrate Call-Exner bodies, which are В целом, эти пациенты имеют повышенный риск карциномы эндометрия и должны
cells with coffee-bean (ie, grooved) nuclei arranged in a rosette pattern (see быть оперативно оценены с помощью биопсии эндометрия. Примечательно, что
image).Gross pathologic examination typically reveals a yellow ovary, which is GCT могут также присутствовать у молодых девушек и будут симптоматически
indicative of its lipid-laden contents required for estrogen production. проявляться как преждевременное половое созревание.
The other answer choices are incorrect for the following reasons:
 CA-125 is a marker that is elevated in epithelial ovarian cancers (serous Опухоли гранулезных клеток классически демонстрируют тела Call-Exner, которые
cystadenocarinoma, mucinous cystadenocarcinomas, etc). These представляют собой клетки с кофейными зернами (то есть с желобками),
tumors would not be associated with signs of estrogen excess (ie, расположенными в виде розетки (см. Изображение). При общем патологическом
endometrial hyperplasia, thickened endometrial stripe). исследовании обычно выявляется желтый яичник, который свидетельствует о его
 Alpha-fetoprotein (AFP) is a tumor marker for hepatocellular, липидном нагружении. содержание, необходимое для производства эстрогена.
nonseminomatous testicular, or ovarian cancers. A yolk sac tumor Другие варианты ответов неверны по следующим причинам:
(endodermal sinus tumor) would not present with signs of estrogen
excess and, rather than Call-Exner bodies,Schiller-Duval (glomeruli- CA-125 является маркером, повышенным при раке эпителия яичников (серозная
esque) bodies would be present on pathology. цистаденокаринома, муцинозная цистаденокарцинома и т. Д.). Эти опухоли не были
 Beta-hCG would be elevated in pregnancy, hydatidiform moles, бы связаны с признаками избытка эстрогена (т.е. гиперплазия эндометрия,
choriocarcinoma, testicular cancer, or mixed germ cell tumor. However, утолщенная полоса эндометрия).
a pregnancy or mole would have been detected on transvaginal Альфа-фетопротеин (АФП) является опухолевым маркером гепатоцеллюлярного,
ultrasound. несеминоматозного рака яичка или яичников. Опухоль желточного мешка
 Testosterone is elevated in Sertoli-Leydig tumors, which would present (эндодермальная синусовая опухоль) не будет иметь признаков избытка эстрогена,
with hyperandrogenism (male-pattern baldness, hirsutism, и, вместо тел Call-Exner, тела Шиллера-Дюваля (клубочкообразные) будут
clitoromegaly) присутствовать при патологии.
 Thyroxine is elevated in struma ovarii and associated with signs of Бета-ХГЧ будет повышен при беременности, гидатидоподобных родинках,
hyperthyroidism (palpitations, weight loss, heat intolerance, etc), not хориокарциноме, раке яичка или опухоли смешанных половых клеток. Однако
signs of excess estrogen. беременность или родинка были бы обнаружены при трансвагинальном УЗИ.
 Lactate dehydrogenase is elevated in ovarian dysgerminoma, most Уровень тестостерона повышен в опухолях Сертоли-Лейдига, что сопровождается
common in adolescents. Dysgerminoma is associated with a “fried-egg” гиперандрогенизмом (облысение по мужскому типу, гирсутизм, клиторомегалия)
appearance on pathology. lock Тироксин повышен при язве зоба и связан с признаками гипертиреоза (учащенное
сердцебиение, потеря веса, непереносимость тепла и т. Д.), А не признаками
избытка эстрогена.
Лактатдегидрогеназа повышена при дисгерминоме яичников, чаще всего
встречается у подростков. Дисгерминома связана с появлением «жареного яйца»
при патологии.

A 24-year-old woman, G1P0, presents at 7 weeks’ gestation, stating that she 24-летняя женщина, G1P0, представляет на 7 неделе беременности, заявив, что у
has had sharp pains in her lower abdominal area for the past day. She also нее были острые боли в нижней части живота в течение последнего дня. Она также
reports some vaginal spotting over the past 3 days. Three days earlier, at a сообщает о некоторых вагинальных пятнах за последние 3 дня. Тремя днями ранее
regular prenatal check, her β-human chorionic gonadotropin (β-hCG) level was при регулярной пренатальной проверке было установлено, что уровень β-
found to be 1700 mIU/mL, and a transvaginal ultrasound showed an empty человеческого хорионического гонадотропина (β-ХГЧ) составляет 1700 мМЕ / мл, а
uterus and no adnexal masses. She denies a history of sexually transmitted трансвагинальное УЗИ показало пустую матку и отсутствие добавочных масс. Она
diseases. Currently she is very nervous about the status of the baby. On отрицает историю заболеваний, передающихся половым путем. В настоящее время
examination, her cervix is closed and nontender. She has mild right adnexal она очень нервничает по поводу состояния ребенка. При осмотре ее шейка матки
tenderness, although no mass is appreciated. Her β-hCG level is 2100 mIU/mL, закрыта и без подтяжки. У нее мягкая болезненность правого придатка, хотя масса
and a transvaginal ultrasound still shows an empty uterus and no adnexal не ценится. Уровень β-ХГЧ у нее составляет 2100 мМЕ / мл, а трансвагинальное
masses. УЗИ по-прежнему показывает пустую матку и отсутствие придаточных масс.
Which of the following is a major risk factor related to this patient’s condition? Что из перечисленного является основным фактором риска, связанным с
состоянием этого пациента?
A.Extremes of age
Крайности возраста
B.History of miscarriage
История выкидыша
C.Multiparity Multiparity
Несоблюдение орального контрацептива
D.Noncompliance with an oral contraceptive *сальпингит

E.Salpingitis Представление этой женщины с аменореей, болью в нижней части живота и


вагинальным пятном является классическим для внематочной беременности.
Correct! Внематочная беременность существует, когда имплантация происходит где-то,
This woman’s presentation with amenorrhea, lower abdominal pain, and vaginal кроме нормальных, внутриматочных эндометриальных участков. Сальпингит
spotting is classic for ectopic pregnancy. Ectopic pregnancy exists when (воспаление тазовых органов) может привести к образованию рубцов и спаек,
implantation occurs anywhere other than the normal, intrauterine endometrial которые обычно приводят к хронической боли в области таза, бесплодию и
sites. Salpingitis (pelvic inflammatory disease) may cause tubal scarring and повышенному риску внематочной беременности.
adhesions, which commonly result in chronic pelvic pain, infertility, and В этот момент у этого пациента уровень ß-ХГЧ составляет 2100 мМЕ / мл, что
increased risk for ectopic pregnancy. превышает дискриминационный порог в 1500-2000 мМЕ / мл. При уровнях на или
At this point, this patient has a ß-hCG level of 2100 mIU/mL, which is above the выше этого порога внутриутробный гестационный мешок обычно наблюдается
discriminatory threshold of 1500–2000 mIU/mL. At levels at or above this трансвагинальной сонографией. Поскольку в этом случае не наблюдается
threshold, an intrauterine gestational sac is usually observable by transvaginal внутриутробной беременности, вероятность внематочной беременности очень
sonography. Because no intrauterine pregnancy is seen in this case, the высока; тот факт, что не отмечается придаточная масса, не исключает диагноз.
likelihood of ectopic pregnancy is very high; the fact that no adnexal mass is
noted, however, does not rule out the diagnosis. При нормальной внутриутробной беременности уровни β-ХГЧ обычно удваиваются
With normal intrauterine pregnancies, ß-hCG levels usually double every 1.5–2 каждые 1,5–2 дня до 6–7 недель беременности. У этого пациента уровень β-ХГЧ
days until 6–7 weeks' gestation. In this patient, the ß-hCG level increased from увеличился с 1700 до 2100 мМЕ / мл в течение 3 дней, что указывает на
1700 to 2100 mIU/mL over a period of 3 days, which indicates a nonviable нежизнеспособную беременность.
pregnancy. Помимо сальпингита (воспалительного заболевания органов малого таза), другие
Aside from salpingitis (pelvic inflammatory disease), other risk factors for ectopic факторы риска внематочной беременности включают сальпингостомию,
pregnancy include salpingostomy, endometriosis, prior tubal surgery, and эндометриоз, перенесенную операцию на трубке и измененную моторику трубки.
altered tubal motility. Другие варианты ответа не являются непосредственно факторами риска
The other answer choices are not directly risk factors for ectopic внематочной беременности. Несоблюдение оральных контрацептивов является
pregnancy. Noncompliance with oral contraceptive is a risk factor for фактором риска для беременности в целом. Экстремальные возрастные (<15 лет
pregnancy in general. Extremes of age (<15 years or >45 years) are risk или> 45 лет) являются факторами риска для формирования полного водоподобной
factors for the formation of a complete hydatidiform mole. History of родинки. В анамнезе невынашивание беременности и многоплодие не повышают
miscarriageand multiparity do not increase the risk of ectopic pregnancy. риск внематочной беременности.
A 26-year-old man presents to the urologist because he and his wife have failed 26-летний мужчина представляет урологу, потому что он и его жена не смогли
to conceive after trying for 14 months. His total and free testosterone levels are забеременеть после попытки в течение 14 месяцев. Его уровень общего и
normal, and initial semen analysis shows significantly decreased volume and no свободного тестостерона в норме, а первоначальный анализ спермы показывает
detectable sperm. Testicular fine-needle biopsy findings indicate normal sperm значительно уменьшенный объем и отсутствие обнаруживаемой спермы.
motility and normal sperm morphology. His medical history is significant for Результаты биопсии тонкой иглы яичка указывают на нормальную подвижность
hypercholesterolemia and a surgical bilateral inguinal hernia repair at 8 years of сперматозоидов и нормальную морфологию сперматозоидов. Его история болезни
age. He also admits to having smoked marijuana four or five times a week важна для гиперхолестеринемии и хирургического двустороннего восстановления
during college and to drinking 3–4 alcoholic beverages a day until recently. паховой грыжи в 8 лет. Он также признается, что курил марихуану четыре или пять
Which of the following components of his history is the most likely cause of this раз в неделю во время учебы в колледже и до недавнего времени употреблял 3-4
patient′s infertility? алкогольных напитка в день.
A.Four-year history of heavy marijuana use Какой из следующих компонентов его истории является наиболее вероятной
причиной бесплодия этого пациента?
B.History of drinking 3–4 alcoholic beverages a day
Четырехлетняя история употребления тяжелой марихуаны
C.Hypercholesterolemia
История употребления 3–4 алкогольных напитков в день
D.Insufficient amount of time attempting to conceive гиперхолестеринемия
Недостаточно времени, пытаясь зачать ребенка
E.Surgical inguinal hernia repair *Хирургическое лечение паховой грыжи
Correct!
This 26-year-old man presents because of difficulty conceiving a child. His total Этот 26-летний мужчина преподносит из-за трудности зачатия ребенка. Его общий и
and free testosterone levels are normal. Initial semen analysis shows свободный уровень тестостерона в норме. Первоначальный анализ спермы
significantly decreased volume and no detectable sperm, although fine-needle показывает значительно уменьшенный объем и не выявляет сперматозоидов, хотя
aspiration shows normal sperm motility and normal sperm morphology. The аспирация тонкой иглой показывает нормальную подвижность сперматозоидов и
patient’s history of surgical inguinal hernia repair strongly suggests an нормальную морфологию сперматозоидов. История хирургического лечения
obstructive abnormality of the vas deferens, leading to disordered sperm паховой грыжи у пациента свидетельствует о наличии обструктивной
transport. The vas deferens can be ligated accidentally during hernia repair, or аномалии семявыносящего протока, что приводит к нарушению транспорта
scar tissue can make passage of the sperm through the vas deferens сперматозоидов. Семявыносящий проток может быть случайно перевязан во
impossible. In the case of vas deferens obstruction, semen volume will be low время восстановления грыжи, или рубцовая ткань может сделать
with decreased or absent sperm. In these cases testicular biopsy can confirm невозможным прохождение сперматозоида через семявыносящий проток. В
normal sperm production, and sperm can be collected for intracytoplasmic случае обструкции семявыносящего протока объем спермы будет низким с
sperm injection with in vitro fertilization. уменьшением или отсутствием сперматозоидов. В этих случаях биопсия яичка
может подтвердить нормальное производство сперматозоидов, и сперма может
The other components of this patient's history would not be associated with an быть собрана для внутрицитоплазматического введения сперматозоидов с
absence of detectable sperm. Although alcohol or marijuana use can lead to экстракорпоральным оплодотворением. Другие компоненты истории этого пациента
decreased sperm production, neither would cause an absence of sperm in the не будут связаны с отсутствием выявляемой спермы. Хотя употребление алкоголя
ejected semen. Hypercholesterolemia is not linked with reduced sperm или марихуаны может привести к снижению производства спермы, ни один из них не
production or defective sperm transport. Infertility is suggested after 12 months приведет к отсутствию сперматозоидов в выброшенной сперме.
of attempting to conceive. Гиперхолестеринемия не связана с уменьшением производства сперматозоидов или
нарушением транспорта сперматозоидов. Бесплодие предлагается после 12
месяцев попытки зачать ребенка.
A 64-year-old woman presents to the physician because of new-onset 64-летняя женщина поступает к врачу из-за начального постменопаузального
postmenopausal bleeding. She has been pregnant twice and has given birth to кровотечения. Она дважды была беременна и родила двоих детей. Ее последний
two children. Her last menstrual period was 12 years ago. The patient has no менструальный период был 12 лет назад. У пациента нет других медицинских
other medical problems aside from hypertension, which is well controlled. Her проблем, кроме гипертонии, которая хорошо контролируется. Ее мать скончалась от
mother passed away from breast cancer at age 75. Ultrasonography of the рака молочной железы в возрасте 75 лет. Ультрасонография яичников показывает
ovaries reveals a small left adnexal mass, along with a thickened endometrial небольшую левую придаточную массу вместе с утолщенной полосой эндометрия
stripe (see image). (см. Изображение).
Что из следующего лучше всего описывает клетки этой опухоли?
Which of the following best describes the cells of this tumor?
Кистозная коллекция щитовидной железы, кости и нервной ткани
A cystic collection of thyroid, bone, and neural tissue Эпителиальные клетки, напоминающие слизистую оболочку маточной трубы с
концентрическими кальцификациями
Epithelial cells resembling fallopian tube lining with concentric calcifications *Эстроген-секретирующие клетки яичников
Эстрогенно-чувствительные клетки яичников, напоминающие ткани эндометрия
Estrogen-secreting ovarian cells Муцин-секретирующие клетки перстня-кольца желудочного происхождения

Estrogen-sensitive ovarian cells resembling endometrial tissue Новое влагалищное кровотечение у этой женщины в постменопаузе должно
вызывать подозрение на гиперплазию эндометрия, вызванную патологическим, не
Mucin-secreting signet-ring cells of gastric origin имеющим аналогов секрецией эстрогена клетками яичника. Утолщенная полоса
эндометрия, видимая на УЗИ этого пациента, дает больше признаков гиперплазии
This postmenopausal woman’s new-onset vaginal bleeding should raise или карциномы.
suspicion for endometrial hyperplasia influenced by pathologic unopposed
estrogen secretion by ovarian cells. The thickened endometrial stripe seen on Гиперэстрогенизм следует подозревать у пожилых женщин с постменопаузальным
this patient’s ultrasound provides more evidence of hyperplasia or carcinoma. кровотечением и / или кистозной грудью, а также у молодых пациентов с
преждевременным половым созреванием. Важно помнить об этой патофизиологии,
Hyperestrogenism should be suspected in elderly women with postmenopausal поскольку эстроген-секретирующие опухоли приводят к раку эндометрия у 15%
bleeding and/or cystic breasts and in young patients with precocious puberty. It пациентов. В этом случае основной причиной является гранулезная клетка опухоли
is important to keep this pathophysiology in mind, as estrogen-secreting tumors яичника. Классическим гистологическим открытием в таких опухолях является
lead to endometrial carcinoma in 15% of patients. In this case, the underlying наличие тел Call-Exner (см. Кружки на изображении ниже), которые представляют
cause is a granulosa cell tumor of the ovary. The classic histologic finding in собой фолликулы, состоящие из гранулезных клеток, случайно расположенных
such tumors is the presence of Call-Exner bodies (see circles in image below), вокруг пространства, которое может содержать эозинофильные выделения.
which are follicles composed of granulosa cells haphazardly arranged around a
space which may contain eosinophilic secretions. A cystic collection of thyroid, Кистозное скопление щитовидной железы, костей и нервной ткани относится к
bone, and neural tissue refers to a mature teratoma. Teratomas contain tissue зрелой тератоме. Тератомы содержат ткани, полученные по меньшей мере из двух
derived from at least two different embryonic layers (for example cartilage, разных эмбриональных слоев (например, хряща, желудочно-кишечного эпителия и
gastrointestinal epithelium, and adipose tissue); one would not expect to see жировой ткани); никто не ожидал бы увидеть тела Call-Exner или вагинальное
Call-Exner bodies or vaginal bleeding with a teratoma. Epithelial cells кровотечение с тератомой. Эпителиальные клетки, напоминающие слизистую
resembling fallopian-tube lining with concentric calcifications describes a serous оболочку маточной трубы с концентрическими кальцификациями, описывают
borderline tumor (or serous cystadenoma). Instead of Call-Exner bodies, one серозную пограничную опухоль (или серозную цистаденому). Вместо тел Call-Exner
would see a tumor lined with epithelium resembling that of the fallopian tube and можно было бы увидеть опухоль, выстланную эпителием, напоминающим опухоль
psammoma bodies (concentric rings of calcification). Because they do not маточной трубы и тела псаммомы (концентрические кольца кальцификации).
secrete estrogen, they do not classically present with vaginal bleeding and Поскольку они не выделяют эстрогены, они не имеют классического вагинального
would not cause a thickened endometrial stripe. An ovarian endometrioid кровотечения и не вызывают утолщение эндометриальной полоски.
carcinoma has estrogen-sensitive ovarian cells that resemble malignant Эндометриоидная карцинома яичника имеет чувствительные к эстрогену клетки
endometrial tissue. Although they may resemble endometrial tissue, the cells яичника, которые напоминают злокачественную ткань эндометрия. Хотя они могут
are usually estrogen sensitive (as opposed to estrogen secreting); Call-Exner напоминать ткань эндометрия, клетки обычно чувствительны к эстрогену (в отличие
bodies are absent; and the patient would not present with vaginal bleeding. от секреции эстрогена); Call-Exner тела отсутствуют; и пациент не будет с
Krukenberg tumors are composed of mucin-secreting signet-ring cells, usually of вагинальным кровотечением. Опухоли Крукенберга состоят из секретирующих
gastric origin; however, they may be from other sites. The classic histologic муцин клеток кольцевых перстней, обычно желудочного происхождения; однако они
finding is a mucin-secreting signet-ring cell. могут быть с других сайтов. Классическим гистологическим открытием является
секретирующая муцином клетка с кольцом перстня.
An 18-day-old female infant with ambiguous genitalia is brought to the 18-дневного ребенка женского пола с неоднозначными гениталиями доставляют к
pediatrician with a history of vomiting for the past 24 hours and lethargy for the педиатру с рвотой в течение последних 24 часов и летаргией на прошлой неделе.
past week. On physical examination, the infant’s vital signs include a При физикальном осмотре показатели жизнедеятельности ребенка включают
temperature of 37°C (98.6°F), blood pressure of 68/48 mm Hg, and pulse of температуру 37 ° C (98,6 ° F), артериальное давление 68/48 мм рт.ст. и пульс 182 /
182/min. The infant's anterior fontanelle feels somewhat depressed, and the мин. Передняя родничок младенца чувствует себя несколько подавленным, и у
infant has decreased skin turgor. младенца уменьшился тургор кожи.

Laboratory studies show: Лабораторные исследования показывают:


WBC count: 9000/mm3 WBC count: 9000 / мм3
Hematocrit: 48% Гематокрит: 48%
Hemoglobin: 14.8 g/dL Гемоглобин: 14,8 г / дл
Platelet count: 475,000/mm3 Количество тромбоцитов: 475 000 / мм3
Sodium: 129 mEq/L Натрий: 129 мг-экв / л
Potassium: 6.1 mEq/L Калий: 6,1 мг-экв / л
Chloride: 94 mEq/L Хлорид: 94 мг-экв / л
Carbon dioxide: 16 mEq/L Углекислый газ: 16 мг-экв / л
Blood urea nitrogen: 24 mg/dL Азот мочевины крови: 24 мг / дл
Creatinine: 0.3 mg/dL Креатинин: 0,3 мг / дл
Glucose: 70 mg/dL Глюкоза: 70 мг / дл
If testing is continued, what other laboratory result would be expected to be
noted in this patient? Если тестирование будет продолжено, какие еще лабораторные результаты следует
A.Decreased plasma renin ожидать у этого пациента?

Снижение ренина плазмы


B.Decreased serum dehydroepiandrosterone Снижение сывороточного дегидроэпиандростерона
Повышенный уровень 11-дезоксикортикостерона в сыворотке крови
C.Elevated serum 11-deoxycorticosterone Повышенная сыворотка 17-гидроксипрогестерон
Повышенный уровень кортизола в сыворотке
D.Elevated serum 17-hydroxyprogesterone Этот ребенок женского пола имеет неоднозначные гениталии, гипотонию и признаки
дегидратации и нарушений электролита, включая низкую концентрацию
E.Elevated serum cortisol бикарбоната, гипонатриемию и гиперкалиемию. Это указывает на то, что у этого
Correct! ребенка врожденная гиперплазия надпочечников, скорее всего, возникшая
This female infant presents with ambiguous genitals, hypotension, and signs of вследствие дефицита 21-гидроксилазы. У этой пациентки отсутствуют ферменты,
dehydration and electrolyte disturbances, including a low bicarbonate необходимые для превращения предшественников стероидов в альдостерон или
concentration, hyponatremia, and hyperkalemia. This indicates that this infant кортизол, что приводит к гипотонии и нарушениям электролита, о которых
has congenital adrenal hyperplasia, most likely occurring secondary to 21- свидетельствуют результаты ее теста. У пациентов с дефицитом 21-гидроксилазы
hydroxylase deficiency. This patient lacks enzymes necessary to convert будет накопление сывороточного 17-гидроксипрогестерона, его субстрата. Избыток
steroid precursors to aldosterone or cortisol, leading to hypotension and the субстрата направляется на выработку андрогена, что приводит к вирилизации.
electrolyte disturbances indicated by her test results. Patients with 21- Дефицит 21-гидроксилазы является наиболее частой причиной врожденной
hydroxylase deficiency will have an accumulation of serum 17- гиперплазии надпочечников.
hydroxyprogesterone, its substrate. The excess substrate is shunted to Другие варианты ответа не будут представлены с этой клинической картиной.
androgen production, leading to virilization. 21-Hydroxylase deficiency is the Снижение содержания ренина в плазме будет противоречить представлению,
most common cause of congenital adrenal hyperplasia. которое включает гипонатриемию; концентрация ренина будет повышена в качестве
The other answer choices would not present with this clinical компенсации. Снижение уровня дегидроэпиандростерона в сыворотке (DHEA)
picture. Decreased plasma reninwould be contrary to a presentation that свидетельствует об отсутствии избытка андрогенов, но у этой девочки есть
includes hyponatremia; the renin concentration would be elevated in вирилизация половых органов, что указывает на повышенный уровень андрогенов.
compensation. Decreased serum dehydroepiandrosterone (DHEA) suggests Андрогены включают DHEA, андростендион и тестостерон - некоторые из которых
a lack of excess androgens, but this female infant has virilization of her genitals, являются предшественниками стероидов. Накопление предшественников стероидов
which indicates an elevated level of androgens. Androgens include DHEA, при дефиците 21-гидроксилазы направлено на выработку избытка андрогенов. Даже
androstenedione, and testosterone—some of which are steroid precursors. у пациентов с дефицитом 3β-гидроксистероиддегидрогеназы уровни DHEA остаются
Accumulating steroid precursors in 21-hydroxylase deficiency are driven toward повышенными, несмотря на низкие уровни андростендиона, вызывая некоторую
the production of excess androgens. Even in patients with 3ß-hydroxysteroid степень вирилизации. Повышенный уровень 11-дезоксикортикостерона в сыворотке
dehydrogenase deficiency, DHEA levels remain elevated despite low levels of обнаруживается у пациентов с дефицитом 11β-гидроксилазы. Тем не менее, у
androstenedione, causing some degree of virilization. Elevated serum 11- пациентов с дефицитом 11-гидроксилазы не происходит потеря соли, и гипотония
deoxycorticosterone is found in patients with 11ß-hydroxylase deficiency. обычно не наблюдается. Повышенный уровень кортизола в сыворотке был бы
However, salt wasting would not occur in a patient with 11ß-hydroxylase невозможен у пациента с врожденной гиперплазией надпочечников, учитывая
deficiency, and hypotension would not commonly be seen. Elevated serum дефицит ферментов, необходимых для его синтеза.
cortisolwould not be possible in a patient with congenital adrenal hyperplasia,
given the deficiency in enzymes required for its synthesis.

A 32-year-old mother presents at 16 weeks′ gestation for a routine prenatal 32-летняя мать предъявляет на 16 неделе беременности для обычного
evaluation. The pregnancy has been without complications. She has had no пренатального обследования. Беременность прошла без осложнений. У нее не было
nausea, vomiting, abdominal pain, vaginal discharge, or headache. Her previous тошноты, рвоты, болей в животе, выделения из влагалища или головной боли. Ее
two pregnancies were without complications; both were term, vaginal deliveries. предыдущие две беременности были без осложнений; оба были срочные,
Quadruple screening test of the maternal serum is performed, and results show вагинальные роды. Проводится четырехкратный скрининг материнской сыворотки, и
an elevated α-fetoprotein level but normal estriol, β-human chorionic результаты показывают повышенный уровень α-фетопротеина, но нормальный
gonadotropin (β-hCG), and dimeric inhibin A levels. уровень эстриола, β-человеческого хорионического гонадотропина (β-ХГЧ) и
димерного ингибина А.
Which of the following is the underlying pathophysiology in this patient’s
pregnancy?
Что из следующего является основной патофизиологией в беременности этого
A.Abnormal conception leading to excessive trophoblastic development пациента?

B.Defective primary neurulation


Ненормальная концепция, приводящая к чрезмерному развитию трофобластов
C.Full or partial trisomy 18
Дефектная первичная нейруляция
D.Nondisjunction in maternal gametes Полная или частичная трисомия 18

E.Splitting of the fertilized ovum 2–12 days after fertilization Ненаразъединение в материнских гаметах
Correct!
This patient, who presents at 16 weeks’ gestation, has no obvious symptoms, Расщепление оплодотворенной яйцеклетки через 2–12 дней после оплодотворения
but routine quadruple screening test results show elevated a-fetoprotein, which Показать пояснения
is indicative of an “open” (ie, not skin covered) neural tube defect (NTD), such Правильный!
as meningocele, myelomeningocele, encephalocele, or anencephaly. This У этого пациента, у которого наступила беременность через 16 недель, явных
patient's levels of inhibin, estriol, and ß-hCG are normal. The open NTD allows симптомов нет, но результаты рутинного четырехкратного скрининга показывают
the leakage of both a-fetoprotein and acetylcholinesterase to occur. However, a- повышенный уровень а-фетопротеина, который указывает на «открытый» (то есть
fetoprotein can cross the placenta and therefore is detected in the maternal не покрытый кожей) дефект нервной трубки (NTD), такие как менингоцеле,
serum according to quadruple screening test results. миеломенингоцеле, энцефалоцеле или анэнцефалия. Уровни ингибина, эстриола и
β-ХГЧ у этого пациента в норме. Открытый NTD позволяет протекать как a-
Open NTDs usually result from defective primary neurulation, whereas closed фетопротеина, так и ацетилхолинэстеразы. Тем не менее, a-фетопротеин может
NTDs are the result of defective secondary neurulation. The exact cause of an проникать через плаценту и, следовательно, обнаруживается в материнской
NTD is unknown and is likely multifactorial; factors may include radiation, drugs, сыворотке по результатам четырехкратного скрининга.
malnutrition, and genetics. Maternal folic acid supplementation reduces the risk
for NTDs. However, some medications reduce or inhibit production of folic acid,
including valproic acid and methotrexate. The major NTDs include spina bifida Открытые NTD обычно являются результатом дефектной первичной нейруляции,
occulta, meningocele, myelomeningocele, encephalocele, and anencephaly. тогда как закрытые NTD являются результатом дефектной вторичной нейруляции.
Note that in spina bifida occulta, the a-fetoprotein level can be normal or Точная причина NTD неизвестна и, вероятно, является многофакторной; факторы
elevated. могут включать излучение, лекарства, недоедание и генетику. Материнская добавка
фолиевой кислоты снижает риск развития NTD. Однако некоторые лекарства
The other answer choices would have different presentations. Abnormal снижают или ингибируют выработку фолиевой кислоты, включая вальпроевую
conception leading to excessive trophoblastic development, which is кислоту и метотрексат. Основные NTD включают в себя spina bifida occulta,
diagnostic for gestational trophoblastic disease, presents with characteristically менингоцеле, миеломенингоцеле, энцефалоцеле и анэнцефалию. Обратите
elevated serum ß-hCG, not a-fetoprotein. Full or partial trisomy 18 is внимание на то, что у spina bifida occulta уровень а-фетопротеина может быть
suggested in quadruple screening test results by decreased a-fetoprotein, нормальным или повышенным.
decreased estriol, and decreased ß-hCG. Nondisjunction in maternal
gametes, indicative of trisomy 21 or Down syndrome, is suggested in quadruple
screening test results by decreased a-fetoprotein, decreased estriol, increased Другие варианты ответа будут иметь разные презентации. Ненормальное зачатие,
inhibin A, and increased ß-hCG. Splitting of the fertilized ovum 2–12 days приводящее к избыточному развитию трофобластов, которое является
after fertilization would indicate twins, based on quadruple screening test диагностическим признаком гестационного трофобластического заболевания,
results of increased a-fetoprotein, increased estriol, and increased ß-hCG. характеризуется характерно повышенным уровнем β-ХГЧ в сыворотке, а не а-
фетопротеином. Полная или частичная трисомия 18 предлагается в результатах
четырехкратного скрининга за счет снижения уровня α-фетопротеина, снижения
уровня эстриола и снижения уровня β-ХГЧ. Несоответствие в материнских гаметах,
свидетельствующее о трисомии 21 или синдроме Дауна, предлагается в
результатах четырехкратного скрининга с помощью снижения уровня α-
фетопротеина, снижения уровня эстриола, повышения уровня ингибина A и
повышения уровня β-ХГЧ. Расщепление оплодотворенной яйцеклетки через 2–12
дней после оплодотворения указывало бы на близнецов, основываясь на
результатах четырехкратного скрининга повышенного уровня α-фетопротеина,
повышенного уровня эстриола и повышенного уровня β-ХГЧ.
A 36-year-old woman comes to her primary care physician for her first physical 36-летняя женщина приходит к своему врачу первичной медицинской помощи для
examination since she was in high school. She has not had routine medical ее первого физического осмотра, так как она была в средней школе. У нее не было
examinations because she lacked health insurance. She is currently sexually плановых медицинских осмотров, потому что ей не хватало медицинской страховки.
active with her husband of 5 years. She had multiple sexual partners while in В настоящее время она сексуально активна с мужем 5 лет. У нее было несколько
her 20s, when she was treated twice for chlamydia. The patient denies using половых партнеров, когда ей было 20, когда она дважды лечилась от хламидиоза.
tobacco, alcohol, or illicit drugs. Her menstrual cycles are regular, and she has Пациент отрицает употребление табака, алкоголя или запрещенных наркотиков. У
never been pregnant. Medical history is unremarkable. Her temperature is нее регулярные менструальные циклы, и она никогда не была беременна. История
98.6°F (37°C), blood pressure is 117/70 mm Hg, pulse is 88/min, respiratory болезни ничем не примечательна. Ее температура составляет 98,6 ° F (37 ° C),
rate is 16/min. Physical examination is normal. A Pap test is performed and two артериальное давление 117/70 мм рт.ст., пульс 88 / мин, частота дыхания 16 / мин.
weeks later, results show evidence of severe dysplasia with undifferentiated Физикальное обследование в норме. Пап-тест проводится, и через две недели
neoplastic cells spanning more than two-thirds of the epithelium. результаты показывают признаки тяжелой дисплазии с недифференцированными
Which of the following best describes the pathogenesis of this patient’s results? опухолевыми клетками, охватывающими более двух третей эпителия.
A.Decreased transcription of p21
Что из нижеперечисленного лучше всего описывает патогенез результатов этого
пациента?
B.Decreased ubiquination of p53

C.Hypophosphorylation of retinoblastoma protein Снижение транскрипции р21

Снижение убиквинации р53


D.Inhibition of signal transduction
Гипофосфорилирование белка ретинобластомы
E.Repair of DNA
Correct! Ингибирование передачи сигнала
This patient presents at age 36 years for her first physical examination since
high school, which means she did not have her routine Pap test at age 21 years. Ремонт ДНК
Her recent Pap test results, as described in the question stem as severe Показать пояснения
dysplasia with undifferentiated neoplastic cells that span more than two-thirds of Правильный!
the epithelium, is diagnostic for cervical intraepithelial neoplasia (CIN). CIN is Эта пациентка предъявляет в возрасте 36 лет свое первое физическое
a premalignant transformation of the cells of the cervix, often secondary to обследование со средней школы, а это значит, что в возрасте 21 года у нее не было
exposure to the human papillomavirus (HPV), which may progress to cervical обычного теста Папаниколау. Ее недавние результаты Пап-теста, описанные в
cancer if left untreated. вопросе, представляют собой тяжелую дисплазию с недифференцированными
HPV promotes neoplasia through production of the viral proteins E6 and E7. неопластическими клетками, которые охватывают более двух третей эпителия,
The E6 protein binds and ubiquinates the p53 (tumor suppressor protein), являются диагностическими для интраэпителиальной неоплазии шейки матки (CIN).
leading to the degradation of p53 by proteasomes. Decreased presence of p53 CIN - это предраковая трансформация клеток шейки матки, часто вторичная по
decreases the transcription of p21 protein. The p21 protein is a cyclin- отношению к воздействию вируса папилломы человека (ВПЧ), который может
dependent kinase inhibitor that functions as a cell-cycle regulator between the перейти в рак шейки матки, если его не лечить.
G1-to-S phase of the cell cycle. Therefore, the decreased levels of p21 protein,
as a result of the presence of the E6 protein, subsequently cause inappropriate ВПЧ способствует образованию новообразований путем производства вирусных
G1–S progression. белков E6 и E7. Белок E6 связывает и убихинитирует p53 (белок-супрессор
In addition, the normal function of the retinoblastoma (Rb) protein is to stop the опухолей), что приводит к деградации p53 протеасомами. Снижение присутствия р53
progression of G1–S phase of the cell cycle. The second viral protein, E7, снижает транскрипцию белка р21. Белок p21 является ингибитором циклин-
produced by HPV binds to the Rb protein and prevents it from mediating the зависимой киназы, который функционирует в качестве регулятора клеточного цикла
G1–S phase inhibition. Thus, loss of both p53 and Rb by HPV viral proteins, E6 между G1-S-фазой клеточного цикла. Следовательно, пониженные уровни белка р21
and E7, respectively, leads to inhibition of cell-cycle regulation and evasion of в результате присутствия белка Е6 впоследствии вызывают неадекватную
apoptosis. прогрессию G1-S.
Vaccines exist to help protect against HPV infections and the associated
cancers. The vaccine is recommended for girls aged 9-26 years and boys aged Кроме того, нормальной функцией белка ретинобластомы (Rb) является остановка
9-21 years. Two vaccines exist: a quadrivalent vaccine that immunizes against прогрессирования фазы G1-S клеточного цикла. Второй вирусный белок, E7,
HPV types 6, 11, 16, and 18 and a 9-valent vaccine that immunizes against продуцируемый HPV, связывается с белком Rb и не дает ему опосредовать
HPV types 6, 11, 16, 18, 31, 33, 45, 52, and 58. HPV types 16 and 18 are the ингибирование фазы G1-S. Таким образом, потеря как p53, так и Rb вирусными
most common types associated with the development of malignant lesions. белками HPV, E6 и E7, соответственно, приводит к ингибированию регуляции
The other answer choices do not address the correct pathogenesis of cervical клеточного цикла и уклонению от апоптоза.
cancer by HPV viral proteins E6 and E7:
 Hyperphosphorylation of Rb is seen in healthy cells, as Вакцины существуют для защиты от ВПЧ-инфекции и связанных с ней раковых
hypophosphorylation activates Rb and inhibits G1–S progression. заболеваний. Вакцина рекомендуется для девочек в возрасте 9–26 лет и мальчиков
 As mentioned previously, E6 increases, not decreases, ubiquitination в возрасте 9–21 года. Существуют две вакцины: четырехвалентная вакцина, которая
of Rb. иммунизирует против ВПЧ типов 6, 11, 16 и 18, и 9-валентная вакцина, которая
 DNA repair and inhibition of signal transduction are not appreciated иммунизирует против ВПЧ типов 6, 11, 16, 18, 31, 33, 45, 52 и 58. Типы ВПЧ 16 и 18
in the E6 and E7 pathway. являются наиболее распространенными типами, связанными с развитием
злокачественных поражений.

Другие варианты ответа не относятся к правильному патогенезу рака шейки матки с


помощью вирусных белков ВПЧ Е6 и Е7:

Гиперфосфорилирование Rb наблюдается в здоровых клетках, так как


гипофосфорилирование активирует Rb и ингибирует прогрессирование G1-S.

Как упоминалось ранее, E6 увеличивает, а не уменьшает убиквитинирование Rb.

Восстановление ДНК и ингибирование передачи сигнала не оцениваются в пути E6 и


E7.
A 25-year-old woman, who is pregnant for the first time, arrives in the labor and 25-летняя женщина, которая впервые беременна, поступает в отделение родов
delivery unit at 37 weeks' gestation after her membranes have ruptured. A baby через 37 недель после разрыва ее оболочек. Девочка доставляется вагинально. При
girl is delivered vaginally. On physical examination of the infant, the physician физическом осмотре младенца врач отмечает наличие катаракты и признаков
notes the presence of cataracts and signs of hepatosplenomegaly and detects a гепатоспленомегалии и выявляет непрерывный шум при аускультации сердца,
continuous murmur while auscultating the heart just inferior to the left clavicle. которое только уступает левой ключице. Мать не пила алкоголь и не употребляла
The mother did not drink alcohol or use any drugs during pregnancy. However, наркотики во время беременности. Тем не менее, мать помнит сыпь с ноющими
the mother does remember having a rash with aching joints for a few weeks суставами в течение нескольких недель во время беременности, но отмечает, что
during her pregnancy but notes that the symptoms were gone before her симптомы исчезли до того, как сломались ее оболочки.
membranes ruptured.
Какие из следующих дополнительных врожденных дефектов чаще всего
Which of the following additional congenital defects is most likely to be seen in наблюдаются у этого ребенка?
this infant?

Костная деформация носового моста


Bone deformity of the nasal bridge
энцефалит
Encephalitis
гидроцефалия
Hydrocephalus
Перивентрикулярные кальцификации
Periventricular calcifications
Нейросенсорная тугоухость
Sensorineural hearing loss Показать пояснения
Show Explanations Правильный!
Correct! У этого новорожденного есть катаракта, гепатоспленомегалия и постоянный шум,
This newborn has cataracts, hepatosplenomegaly, and a continuous murmur, который, скорее всего, вызван открытым артериальным протоком (КПК). Всякий раз,
which is most likely caused by a patent ductus arteriosus (PDA). Whenever a когда у новорожденного есть такая комбинация признаков, следует подозревать
newborn has this combination of signs, a congenital infection with the rubella врожденную инфекцию вирусом краснухи. Матери могут приобретать краснуху через
virus should be suspected. Mothers can acquire rubella via respiratory droplets дыхательные капли и могут иметь сыпь, лимфаденопатию, полиартрит и
and may have a rash, lymphadenopathy, polyarthritis, and polyarthralgia. полиартралгию. Краснуха вызывает легкое заболевание у детей, но это инфекция
Rubella causes mild disease in children, but it is a ToRCHeS infection that can ToRCHeS, которая может вызвать серьезные врожденные дефекты, если
cause severe congenital defects if transmitted in utero. ToRCHeS infections are передается внутриутробно. Инфекции ToRCHeS вызываются микробами, которые
caused by microbes that may pass from mother to fetus, most commonly могут передаваться от матери к плоду, чаще всего трансплацентарно. К инфекциям
transplacentally. The ToRCHeS infections include the following: Toxoplasmosis, ToRCHeS относятся следующие: токсоплазмоз, краснуха, цитомегаловирус, герпес /
Rubella, Cytomegalovirus, Herpes/HIV, and Syphilis. All of these infections can ВИЧ и сифилис. Все эти инфекции могут проявляться гепатоспленомегалией,
present with hepatosplenomegaly, jaundice, thrombocytopenia, and growth желтухой, тромбоцитопенией и задержкой роста у новорожденных. Нейросенсорная
retardation in the neonate. Sensorineural hearing loss, cataracts, and cardiac тугоухость, катаракта и сердечные заболевания (особенно КПК) являются
disease (specifically PDA) are the classic manifestations of congenital rubella классическими проявлениями врожденной инфекции краснухи.
infection.
Bone deformity of the nasal bridge, or saddle nose, is seen in infants with
congenital syphilis infection.?? Деформация костей носового моста или седла носа наблюдается у детей с
Periventricular calcifications are seen in infants with congenital cytomegalovirus врожденной инфекцией сифилиса.
infection. Перивентрикулярные кальцификации наблюдаются у детей с врожденной
Hydrocephalus can be seen in infants with congenital Toxoplasma gondii цитомегаловирусной инфекцией.
infection. Гидроцефалия может наблюдаться у детей с врожденной инфекцией Toxoplasma
Encephalitis is classically seen in infants with congenital herpes simplex virus-2 gondii.
infection. Энцефалит классически наблюдается у детей с врожденной инфекцией вируса
простого герпеса-2.
A 33-year-old woman who is pregnant for the first time and has had no previous 33-летняя женщина, которая беременна впервые и ранее не имела дородового
prenatal care visits a gynecologist. She has a younger sibling who has a ухода, посещает гинеколога. У нее есть младший брат, у которого врожденное
congenital disease and she wishes to undergo prenatal testing. A triple screen заболевание, и она хочет пройти пренатальное обследование. Тройной скрининг
reveals an α-fetoprotein level that is half of the normal level, a β-human показывает уровень α-фетопротеина, который составляет половину от нормального
chorionic gonadotropin level twice the normal level, and an estriol level that is уровня, уровень хорионического гонадотропина β-человека в два раза выше
three-quarters of the normal level. нормального уровня и уровень эстриола, который составляет три четверти
From which birth defect will the baby most likely suffer? нормального уровня.

От какого врожденного дефекта, скорее всего, пострадает ребенок?


A.Aortic coarctation

Коарктация аорты
B.Endocardial cushion defect
Эндокардиальный дефект подушки
C.Holoprosencephaly
Holoprosencephaly
D.Prominent occiput Выдающийся затылок

E.Tuft of hair at the small of the back Пучок волос на пояснице


Show Explanations Показать пояснения
Correct! Правильный!
A decreased α-fetoprotein (AFP) level with an increased ß-human Пониженный уровень α-фетопротеина (АФП) с повышенным уровнем бета-
chorionic gonadotropin level indicates that this fetus may develop Down хорионического гонадотропина у человека указывает на то, что у этого плода может
syndrome due to trisomy of chromosome 21. Individuals with Down syndrome развиться синдром Дауна из-за трисомии 21-й хромосомы. У людей с синдромом
have excessive skin at the nape of the neck, upslanting palpebral fissures, and Дауна кожа на затылке слишком большая, что приводит к наклону глазных щелей и
epicanthic folds, among other characteristic features. In addition, Down эпикантические складки, среди других характерных особенностей. Кроме того,
syndrome is associated with multiple congenital anomalies, including ventricular синдром Дауна связан с множественными врожденными аномалиями, включая
septal defects, endocardial cushion defects, and omphalocele. Cardiac дефекты межжелудочковой перегородки, дефекты эндокардиальной подушки и
disease is an important cause of death in these patients. омфалоцеле. Сердечные заболевания являются важной причиной смерти у этих
Aortic coarctation would be found in a patient with Turner syndrome, пациентов.
and holoprosencephalywould be seen in Trisomy 13. Additionally, Коарктация аорты может быть обнаружена у пациента с синдромом Тернера, а
a prominent occiput is generally found in Edwards syndrome, and is холопрозэнцефалия - при трисомии 13. Кроме того, заметный затылок обычно
associated with a low hCG and AFP on prenatal screening. A tuft of hair on the обнаруживается при синдроме Эдвардса и связан с низким уровнем ХГЧ и АФП при
lower back would be seen in a neural tube defect such as spina bifida and is пренатальном скрининге. Пучок волос в нижней части спины будет виден при
associated with an elevated AFP on prenatal screening. дефекте нервной трубки, таком как расщелина позвоночника, и связан с
повышенным АФП при пренатальном скрининге.
A 24-year-old woman presents to her gynecologist with a report of increasing, 24-летняя женщина представляет своему гинекологу сообщение об увеличении
thick vaginal discharge for the past 3 days and new-onset pain during sexual густых выделений из влагалища в течение последних 3 дней и появлении новой
intercourse. She takes oral contraceptive pills and uses condoms infrequently боли во время полового акта. Она принимает оральные контрацептивы и редко
with her new sexual partner. Her gynecologist treats her empirically and also использует презервативы со своим новым сексуальным партнером. Ее гинеколог
cultures a sample of the discharge. The culture reveals gram-negative cocci in относится к ней эмпирически, а также культивирует образец выделений. Культура
pairs. выявляет грамотрицательные кокки в парах.
Which of the following describes the cell type that mounts the initial immune
response against this organism? Что из нижеперечисленного описывает тип клеток, который устанавливает
первоначальный иммунный ответ против этого организма?

A.Bilobular nucleus with numerous pink granules on hematoxylin and eosin


stain Билобулярное ядро с многочисленными розовыми гранулами на окраске
гематоксилином и эозином
B.Bilobular nucleus with numerous purple granules on hematoxylin and Билобулярное ядро с многочисленными пурпурными гранулами на окраске
eosin stain гематоксилином и эозином

C.Contains a single, slightly off-center nucleus with heterochromatin in a Содержит одно, немного смещенное от центра ядро с гетерохроматином в
clock-face arrangement расположении циферблата

Мультилобулярное ядро с крупными сферическими азурофильными гранулами


D.Multilobular nucleus with large, spherical, azurophilic granules
По размеру похож на эритроциты и содержит одно большое ядро с небольшим
E.Similar in size to erythrocytes and containing one large nucleus with little количеством окружающей цитоплазмы
surrounding cytoplasm Показать пояснения
Show Explanations Правильный!
Correct! Эта сексуально активная женщина имеет увеличивающиеся, густые выделения из
This sexually active woman presents with increasing, thick vaginal discharge влагалища и боль во время полового акта. Грамное пятно выявляет
and pain on intercourse. A Gram stain reveals gram-negative cocci in pairs, грамотрицательные кокки в парах, что подтверждает диагноз Neisseria gonorrhoeae.
supporting a diagnosis of Neisseria gonorrhoeae. For bacterial infections, Для бактериальных инфекций полиморфноядерные клетки (см. Изображение ниже)
polymorphonuclear cells (see image below) are the most common cell type in an являются наиболее распространенным типом клеток при остром ответе на
acute response to bacterial infection. They have a characteristic segmented бактериальную инфекцию. Они имеют характерное сегментированное ядро с тремя-
nucleus with three to five connected lobes with large, spherical, пятью соединенными лепестками с крупными сферическими азурофильными
azurophilic granules and hydrolytic enzymes. гранулами и гидролитическими ферментами.

Bilobed nuclei with bright pink granules on hematoxylin and eosin stain Двухъядерные ядра с ярко-розовыми гранулами на гематоксилине и эозиновом
are characteristic of eosinophils, which respond to parasitic infections and пятне характерны для эозинофилов, которые отвечают на паразитарные инфекции и
allergic reactions/asthma. Cells with a bilobed nucleus and numerous blue to аллергические реакции / астму. Клетки с двухъярусным ядром и многочисленными
purple granules on hematoxylin and eosin stain are basophils, which are сине-фиолетовыми гранулами на окраске гематоксилином и эозином являются
elevated in the blood in the setting of some parasitic infections. Plasma cells базофилами, которые повышаются в крови в условиях некоторых паразитарных
have one large, slightly off-center “clockface” nucleus with perinuclear инфекций. Плазматические клетки имеют одно большое, немного смещенное от
clearing called a Hoff; they are part of the secondary, not the immediate, центра ядро «циферблата» с перинуклеарным очищением, называемым Hoff; они
immune response. Lymphocytes also have one nucleus (and little являются частью вторичного, а не немедленного иммунного ответа. Лимфоциты
surrounding cytoplasm), which takes up most of the cell on microscopic view. также имеют одно ядро (и небольшую окружающую цитоплазму), которое занимает
Lymphocytes are most common in the setting of chronic inflammation. микроскопическую часть большей части клетки. Лимфоциты чаще всего встречаются
в условиях хронического воспаления.
A 15-year-old boy presents with a painless testicular mass. He is otherwise У 15-летнего мальчика безболезненная масса яичка. В остальном он здоров и
healthy and has been meeting normal developmental milestones. After surgical встречает нормальные вехи развития. После хирургического удаления яичка
removal of the testicle, the parents are informed that the mass is a common родителям сообщают, что масса является распространенной опухолью
germ cell tumor that is highly sensitive to radiotherapy. They are told their son зародышевой клетки, которая очень чувствительна к лучевой терапии. Им говорят,
has a good prognosis. что у их сына хороший прогноз.
Which of the following histologic findings was most likely present in this patient’s
tumor? Какие из следующих гистологических результатов наиболее вероятно
присутствовали в опухоли этого пациента?
A.An alveolar/papillary morphologic pattern
Альвеолярный / папиллярный морфологический паттерн
B.Large cells in lobules with clear cytoplasm Крупные клетки в долях с прозрачной цитоплазмой

C.Reinke crystals Рейнке кристаллы

Структуры, напоминающие примитивные клубочки


D.Structures resembling primitive glomeruli
Синцитиотрофобластические и цитотрофобластические клетки с кровоизлиянием и
E.Syncytiotrophoblastic and cytotrophoblastic cells with hemorrhage and некрозом
necrosis Показать пояснения
Show Explanations Правильный!
Correct! У этого 15-летнего здорового мальчика безболезненная масса яичка. После
This otherwise healthy 15-year-old boy presents with a painless testicular mass. орхиэктомии масса оказалась распространенной опухолью зародышевой клетки,
After orchiectomy, the mass is found to be a common germ cell tumor that is которая очень чувствительна к лучевой терапии. У пациента, скорее всего, есть
highly sensitive to radiotherapy. The patient most likely has a seminoma. семинома. Семиномы являются наиболее распространенными новообразованиями
Seminomas are the most common testicular neoplasms and are common in яичек и распространены у мальчиков и мужчин в возрасте от 15 до 35 лет. Они
boys and men from 15 to 35 years of age. They typically have a painless обычно имеют безболезненное представление и радиочувствительны.
presentation and are radiosensitive.
As shown in the image, this patient has a testicular tumor characterized by large Как показано на рисунке, у этого пациента опухоль яичка, характеризующаяся
cells in lobules with watery cytoplasm and a “fried egg” appearance. наличием крупных клеток в долях с водянистой цитоплазмой и появлением
Numerous lymphocytes may be seen in and around the tumor. «жареного яйца». Многочисленные лимфоциты можно увидеть в и вокруг опухоли.

The other histologic findings indicate the presence of other testicular Другие гистологические данные указывают на наличие других патологий яичек.
pathologies. An alveolar or papillary pattern is seen in embryonal carcinoma, Альвеолярный или папиллярный рисунок наблюдается при эмбриональной
which is not radiosensitive and has a poor prognosis. These tumors are also карциноме, которая не является радиочувствительной и имеет плохой прогноз. Эти
painful. Syncytiotrophoblastic and cytotrophoblastic cells with hemorrhage опухоли также болезненны. Синцитиотрофобластические и цитотрофобластические
and necrosis are consistent with choriocarcinoma, which normally presents клетки с кровоизлиянием и некрозом согласуются с хориокарциномой, которая
with systemic symptoms due to the production of human chorionic gonadotropin обычно проявляется системными симптомами из-за производства хорионического
and is highly sensitive to chemotherapy. Reinke crystals are seen in Leydig cell гонадотропина человека и очень чувствительна к химиотерапии. Кристаллы Рейнке
tumors, which produce androgens, leading to precocious puberty or обнаруживаются в клеточных опухолях Лейдига, которые продуцируют андрогены,
gynecomastia. They are not radiosensitive. Structures resembling primitive что приводит к преждевременному половому созреванию или гинекомастии. Они не
glomeruli are seen in yolk sac tumors, which are most commonly seen in boys являются радиочувствительными. Структуры, напоминающие примитивные
< 3 years of age. клубочки, обнаруживаются в опухолях желточного мешка, которые чаще всего
наблюдаются у мальчиков в возрасте до 3 лет.
A 35-year-old African-American woman at 32 weeks′ gestation comes to the 35-летняя афроамериканка в возрасте 32 недель приходит в отделение неотложной
emergency department because of the sudden onset of lower abdominal pain помощи из-за внезапного появления боли в нижней части живота и обильного темно-
and copious, dark red vaginal bleeding. She has three other children at home. красного влагалищного кровотечения. У нее дома трое детей. Она отрицает какое-
She denies any illicit drug use, but smokes one pack of cigarettes per day, либо незаконное употребление наркотиков, но курит одну пачку сигарет в день, что
which she continues during her pregnancy despite efforts to cut down. She also она продолжает во время беременности, несмотря на попытки сократить
denies any trauma or recent sexual activity. потребление. Она также отрицает какую-либо травму или недавнюю сексуальную
On examination, her temperature is 37.1°C (98.8°F), blood pressure is 146/90 активность.
mm Hg, and pulse is 76/min. The fetal heart rate is approximately 170/min.
Abdominal examination reveals a tender uterus with frequent contractions and a При осмотре ее температура составляет 37,1 ° C (98,8 ° F), артериальное давление -
closed cervical os. Urinalysis shows no blood cells or bacteria. 146/90 мм рт.ст., пульс - 76 / мин. Сердцебиение плода составляет около 170 / мин.
Which of the following is the most likely diagnosis? При обследовании органов брюшной полости выявляется болезненная матка с
частыми сокращениями и закрытым шейным отделом. Анализ мочи не показывает
клеток крови или бактерий.
A.Abruptio placentae
Какой из следующих диагнозов является наиболее вероятным?
B.Placenta previa
Отслойка плаценты
C.Umbilical cord prolapse
Предлежание плаценты
D.Uterine rupture Выпадение пуповины

E.Vasa previa Разрыв матки


Show Explanations
Correct! Vasa previa
This pregnant patient with a significant smoking history presents with sudden Показать пояснения
onset abdominal pain, vaginal bleeding, fetal tachycardia, and a tender uterus. Правильный!
This is a common clinical picture of abruptio placentae, the premature Эта беременная пациентка со значительным курением имеет внезапные боли в
separation of the placenta from the uterine wall, that usually occurs during the животе, вагинальное кровотечение, тахикардию плода и болезненную матку. Это
third trimester of pregnancy. It manifests as painful vaginal bleeding that распространенная клиническая картина отслойки плаценты, преждевременного
appears dark red due to the darker appearance of venous blood. This is in отделения плаценты от стенки матки, которая обычно происходит в третьем
contrast to the bright red bleeding seen in placenta previa. As in this patient, триместре беременности. Это проявляется как болезненное влагалищное
initiation of contractions can also be seen in placental abruptions. Severe кровотечение, которое кажется темно-красным из-за более темного появления
abruption may result in fetal death and disseminated intravascular coagulation. венозной крови. Это контрастирует с ярко-красным кровотечением, наблюдаемым в
Risk factors include hypertension, trauma, smoking, and cocaine use during предлежании плаценты. Как и у этого пациента, начало сокращений также можно
pregnancy. увидеть при отслойке плаценты. Тяжелая отслойка может привести к гибели плода и
Placenta previa presents with painless vaginal bleeding, in contrast to this распространению внутрисосудистой коагуляции. Факторы риска включают
patient’s painful bleeding. Vasa previa can present with painless vaginal гипертонию, травму, курение и употребление кокаина во время беременности.
bleeding, however it presents with fetal bradycardia (and the fetal heart rate
here is 170/min). Umbilical cord prolapse does not present with any bleeding. Предлежание плаценты сопровождается безболезненным вагинальным
While uterine rupture is associated with abdominal pain and vaginal bleeding, it кровотечением, в отличие от болезненного кровотечения у этого пациента. Vasa
occurs only during labor. previa может проявляться безболезненным влагалищным кровотечением, однако
она сопровождается брадикардией плода (и частота сердечных сокращений плода
здесь составляет 170 / мин). Выпадение пуповины не сопровождается
кровотечением. Хотя разрыв матки связан с болью в животе и влагалищным
кровотечением, он происходит только во время родов.
A 24-year-old woman reports a 1-month history of weight loss, heart 24-летняя женщина сообщает о потере веса в 1 месяц, учащенном сердцебиении,
palpitations, jitteriness, and sweating. The patient’s mother recently had a нервозности и потоотделении. Мать пациента недавно перенесла тиреоидэктомию
thyroidectomy after experiencing similar symptoms, and the patient wants to после появления подобных симптомов, и пациент хочет узнать, нуждается ли она в
know if she needs surgery. Her temperature is 100.4°F (38°C), pulse is 110, and операции. Ее температура составляет 100,4 ° F (38 ° C), пульс - 110, а частота
respiratory rate is 24. She denies abdominal pain, nausea, or vomiting but does дыхания - 24. Она отрицает боль в животе, тошноту или рвоту, но сообщает о новом
report a new “pulling” sensation in her groin. The patient is actively sweating. No ощущении «растяжения» в паху. Пациент активно потеет. Нет склерального шоу, и
scleral show is present, and there is full range of ocular movement. Her thyroid есть полный диапазон глазного движения. Ее щитовидная железа нормальна к
gland is normal to palpation, her thyroid-stimulating hormone level is 0.3 µU/mL, пальпации, уровень гормонов, стимулирующих щитовидную железу, составляет 0,3
and her thyroxine level is 22 μg/dL. No serum β-human chorionic gonadotropin мкЕ / мл, а уровень тироксина - 22 мкг / дл. Сывороточный β-хорионический
is detected. Pelvic examination reveals a nontender, 9-cm mass. Histologic гонадотропин человека не обнаружен. При осмотре таза выявляется бездонка,
preparation of a representative sample is shown in the image. массой 9 см. Гистологическая подготовка репрезентативного образца показана на
изображении.
Эти выводы наиболее соответствуют тому, что из следующего?

Внематочная беременность

Болезнь Грейвса

Тиреоидит Хашимото

Серозная цистаденома

Струма яичников
Показать пояснения
Правильный!
У этого пациента наблюдается потеря веса, учащенное сердцебиение, нервозность
и потоотделение. Ее мать недавно перенесла тиреоидэктомию после появления
подобных симптомов. Кроме того, пациентка сообщает о «потянувшем» ощущении в
ее паху, и при осмотре таза обнаружена масса 9-сантиметрового пациента. Уровень
These findings are most consistent with which of the following? гормонов, стимулирующих щитовидную железу, составляет 0,3 мкЕ / мл
(нормальный диапазон 0,5–5,0 мкЕ / мл), уровень тироксина в ней составляет 22
мкг / дл (нормальный диапазон 5–12 мкг / дл), а ее щитовидная железа нормально
A.Ectopic pregnancy при сердцебиении.

Тазовая масса и сопутствующие симптомы гипертиреоза согласуются со стромой


B.Graves disease яичников, тиратомой яичников, секретирующей тиреоидные гормоны.
Доброкачественные результаты исследования щитовидной железы делают
C.Hashimoto thyroiditis маловероятным заболевание щитовидной железы, а другие новообразования
яичников вряд ли могут вызывать явные симптомы щитовидной железы. Эти
опухоли проявляются как односторонние кистозные образования в области таза с
D.Serous cystadenoma сопутствующими симптомами гипертиреоза. Биопсия образца тазовой массы
покажет ткань щитовидной железы, содержащую фолликулы, которые
вырабатывают гормон щитовидной железы, как видно на гистологическом образце
E.Struma ovarii
на изображении.
Show Explanations
Correct!
This patient presents with weight loss, heart palpitations, jitteriness, and
sweating. Her mother recently had a thyroidectomy after having similar У этого пациента не обнаружен сывороточный бета-хорионический гонадотропин,
symptoms. In addition, the patient reports a “pulling” sensation in her groin, and исключающий внематочную беременность. Кроме того, внематочная беременность
a nontender 9-cm mass is found on pelvic examination. Her thyroid-stimulating не будет иметь признаков гипертиреоза. Хотя болезнь Грейвса может быть
hormone level is 0.3 µU/mL (normal range, 0.5–5.0 µU/mL), her thyroxine level причиной большинства симптомов гипертиреоза у пациента, она обычно не
is 22 µg/dL (normal range, 5–12 µg/dL), and her thyroid is found to be normal on проявляется с массами яичников; глазное вовлечение более распространено.
palpitation. Тиреоидит Хашимото является причиной гипотиреоза, но у этого пациента
The pelvic mass and the concurrent symptoms of hyperthyroidism are наблюдаются признаки гипертиреоза. Серозная цистаденома проявляется с
consistent with struma ovarii, a thyroid hormone–secreting ovarian teratoma. придаточной массой и вряд ли может вызвать гипертиреоз.
The benign findings from the thyroid examination make this unlikely to be a
thyroid disease, and other ovarian neoplasms are unlikely to cause overt thyroid
symptoms. These tumors manifest as unilateral, cystic, pelvic masses with
concomitant hyperthyroid symptoms. A biopsy specimen of the pelvic mass will
reveal thyroid-type tissue containing follicles that produce thyroid hormone, as
seen in the histologic sample in the image.

No serum ß-human chorionic gonadotropin is detected in this patient, ruling out


an ectopic pregnancy. Furthermore, an ectopic pregnancy would not present
with signs of hyperthyroidism. Although Graves disease may account for most
of the patient’s symptoms of hyperthyroidism, it does not commonly manifest
with ovarian masses; ocular involvement is more common. Hashimoto
thyroiditis is a cause of hypothyroidism, but this patient presents with signs of
hyperthyroidism. A serous cystadenoma would manifest with an adnexal mass
and would be unlikely to cause hyperthyroidism.
A 62-year-old man with a previous diagnosis of benign prostatic hyperplasia is У 62-летнего мужчины с предыдущим диагнозом доброкачественной гиперплазии
seen at his annual physical and found to have a prostate-specific antigen (PSA) предстательной железы наблюдается его ежегодное физическое состояние, и у него
level of 11.2 ng/mL, up from 6.4 ng/mL the previous year. Ultrasound-guided обнаружен уровень простат-специфического антигена (ПСА) 11,2 нг / мл по
transrectal prostate biopsies are performed. сравнению с 6,4 нг / мл в предыдущем году. Проводится ультразвуковая
трансректальная биопсия простаты.
A biopsy specimen from which zone of the prostate is most likely to yield
atypical cellular architecture?
Образец биопсии из какой зоны предстательной железы, скорее всего, приведет к
нетипичной клеточной архитектуре?

A.Central zone
Центральная зона
B.Peripheral
периферический
C.Periurethral
околомочеточниковый

D.Seminal vesicle Семеная везикула


переходный
E.Transitional Показать пояснения
Show Explanations Правильный!
Correct! У этого человека с доброкачественной гиперплазией предстательной железы в
This man, who has a history of benign prostatic hyperplasia, is found to have an анамнезе обнаружен повышенный уровень ПСА 11,2 нг / мл (по сравнению с 6,4 нг /
elevated PSA level of 11.2 ng/mL (up from 6.4 ng/mL at his last annual мл при его последнем ежегодном обследовании). Хотя использование уровней PSA
examination). Although using levels of PSA to help diagnose malignancy of для диагностики злокачественной опухоли простаты является спорным, обычно
the prostate is controversial, it is generally thought that a rapid increase in PSA считается, что быстрое увеличение PSA в течение года требует дальнейшего
over the course of a year warrants further consideration. рассмотрения.
As seen in the image below, the prostate gland is divided into several anatomic
zones. In about 75% of cases, prostate cancer is present in the peripheral Как видно на рисунке ниже, предстательная железа делится на несколько
zone, usually arising near the rectum. This explains why a digital rectal анатомических зон. Примерно в 75% случаев рак предстательной железы
examination is a useful screening test. The peripheral zone has a different присутствует в периферической зоне, обычно возникающей вблизи прямой кишки.
embryologic derivation from the transition zone, which is the most common site Это объясняет, почему цифровое ректальное исследование является полезным
of benign prostatic hyperplasia. скрининговым тестом. Периферическая зона имеет другое эмбриологическое
происхождение от переходной зоны, которая является наиболее распространенным
The central zone, which surrounds the ejaculatory duct, is an uncommon местом доброкачественной гиперплазии предстательной железы.
location for prostate cancer. The periurethral zone and the transitional zone
are synonymous and are the most common sites of benign prostatic hyperplasia Центральная зона, которая окружает эякуляторный проток, является редким местом
but are uncommon sites for cancer. для рака простаты. Периуретральная зона и переходная зона являются синонимами
Finally, the seminal vesicles, while located adjacent to the prostate, are not и являются наиболее распространенными участками доброкачественной
made of prostatic tissue and therefore are not a site of origin for prostate cancer. гиперплазии предстательной железы, но являются редкими участками рака.

Наконец, семенные пузырьки, хотя и расположены рядом с простатой, не состоят из


ткани предстательной железы и, следовательно, не являются местом
происхождения рака простаты.
A 25-year-old woman presents to her primary care physician for an annual 25-річна жінка подає своєму лікареві первинної медичної допомоги на щорічний
physical examination. During the visit she describes noticing less than a медичний огляд. Під час візиту вона щоденно описує менше, ніж чайну ложку
teaspoon of thick white vaginal discharge daily. Vaginal pH is 4.0. Saline товстого білого виділення з піхви. Вагінальний рН становить 4,0. Сольова мікроскопія
microscopy of scant discharge found at the external os identifies epithelial cells мізерного розряду, знайдена на зовнішньому осі, визначає епітеліальні клітини з
with scarce polymorphonuclear leukocytes. дефіцитними поліморфноядерними лейкоцитами.
What is the best description of these clinical findings?
Який найкращий опис цих клінічних даних?

A.Bacterial vaginosis
Бактеріальний вагіноз
B.Gonococcal cervicitis
Гонококовий цервіцит

C.Normal Звичайний
D.Trichomoniasis Трихомоніаз

Вульвовагінальний кандидоз
E.Vulvovaginal candidiasis Показати пояснення
Show Explanations Правильно!
Correct! Цей пацієнт має вагінальний рН 4,0 і щоденний виділення з білого вагінального
This patient has a vaginal pH of 4.0 and thick white vaginal discharge daily that виділення, яке вона описує як менше, ніж чайну ложку в обсязі. Як її рівень рН, так і
she describes as being less than a teaspoon in volume. Both her pH level and кількість і якість її розряду є нормальними. Нормальний діапазон рН піхви становить
the amount and quality of her discharge are normal. The normal vaginal pH від 4,0 до 4,5. Крім того, жінки зазвичай мають щоденно 1–4 мл прозорого до білого
range is between 4.0–4.5. Moreover, women normally have 1–4 mL of clear to або жовтуватого вагінального виділення (переважно «фізіологічної лейкореї»). Не всі
white to yellowish, mostly odorless vaginal discharge (“physiologic leukorrhea”) виділення є патологічними.
daily. Not all discharge is pathologic. Бактеріальний вагіноз представляв би згубний сірий виділення. Трихомоноз
Bacterial vaginosis would present with a malodorous grey характеризується гнійним і неприємним запахом виділень, а також
discharge. Trichomoniasis is characterized by a purulent and foul-smelling вульвовагінальним кандидозом (або дріжджовою інфекцією) з більш товстим
discharge, and vulvovaginal candidiasis (or yeast infection) with a thicker виділенням і свербінням. Гонококовий цервіцит має більшу кількість гнійних
discharge and pruritis. Gonococcal cervicitis would present with a greater виділень, можливо, за відсутності інших симптомів.
amount of more purulent discharge, possibly in the absence of other symptoms.
A 32-year-old primigravid woman presents at 32 weeks’ gestation because of 32-річна жінка первинного віку представлена на терміні вагітності 32 тижні через
sudden vaginal bleeding and painful abdominal cramps. She denies trauma. раптові вагінальні кровотечі і болісні спазми в животі. Вона заперечує травму. Вона
She has a 10-pack-year smoking history and admits to cocaine use during her має 10-тирічну історію куріння і допускає використання кокаїну під час вагітності. Її
pregnancy. Her blood pressure is 95/60 mm Hg, pulse is 112/min, and blood артеріальний тиск становить 95/60 мм рт.ст., пульс - 112 / хв, а насичення крові
oxygen saturation is 97% on room air (normal: 95%-99%). Physical examination киснем - 97% на кімнатному повітрі (норма: 95% -99%). Фізичне обстеження виявляє
reveals a tense abdomen with a firm, tender uterus. Urinalysis shows no protein, напружений живіт з твердою, ніжною маткою. Аналіз сечі не демонструє білка,
leukocytes, or bacteria, with few RBCs. Pelvic examination reveals dark-red лейкоцити або бактерії з невеликою кількістю еритроцитів. Тазове дослідження
blood in the vaginal vault and a hypertonic uterus. Pelvic ultrasound shows a виявляє темно-червону кров у склепіні піхви і гіпертонічну матку. УЗД тазових
high posterior placenta with no abnormalities of placentation. Fetal heart tones відділів показує високу задню плаценту без відхилень від плацентації. Тони серця
indicate fetal distress. плода вказують на дистрес плода.
This patient is at increased risk for which of the following complications?
Цей пацієнт піддається підвищеному ризику, для якого з наступних ускладнень?

A.Disseminated intravascular coagulation


Розсіяна внутрішньосудинна коагуляція
B.Infertility
Безпліддя

C.Postpartum bleeding Післяпологова кровотеча

Прееклампсія
D.Preeclampsia
Синдром Шихана
E.Sheehan syndrome Показати пояснення
Show Explanations Правильно!
Correct! Пацієнт має раптові вагінальні кровотечі і болісні спазми в черевній порожнині при
The patient presents with sudden vaginal bleeding and painful abdominal терміні 32 тижнів без травми. Вона має низький кров'яний тиск і швидке серцевий
cramps at 32 weeks' gestation without trauma. She has low blood pressure and ритм, що вказує на кровотечу. Плід знаходиться в тяжкому стані в межах
rapid heart rate indicating a bleed. The fetus is in distress within a hypertonic гіпертонічної матки. В історії пацієнта куріння та вживання кокаїну також вказують на
uterus. The patient's history of smoking and cocaine use also point to a діагноз абсурдо плаценти або відшарування плаценти.
diagnosis of abruptio placentae, or placental abruption.
Placental abruption is the partial or complete premature separation of placenta Відшарування плаценти - це часткове або повне передчасне відділення плаценти від
from the uterine wall. Risk factors include smoking and cocaine use as seen in стінки матки. Фактори ризику включають в себе паління і вживання кокаїну, як це
this patient, as well as trauma, hypertension, and preeclampsia. Patients спостерігається у даного пацієнта, а також травми, гіпертонія і прееклампсія.
present with sudden painful bleeding in the third trimester, as seen in the Пацієнти присутні з раптовими болючими кровотечами в третьому триместрі, як
vignette. Complications are life threatening and include DIC from tissue factor видно з віньєтки. Ускладнення є загрозливими до життя і включають ДВЗ з
entering maternal circulation, maternal shock, and fetal distress or death. тканинного фактора, що надходить з материнського кровообігу, материнського шоку і
Postpartum hemorrhage and Sheehan syndrome are associated with дистрес плода або смерть.
placenta accreta/increta/percreta, where the maternal portion of the placenta,
the desidua basalis, has abnormal attachment and separation after delivery. Післяпологові кровотечі і синдром Шихана асоціюються з плацентою accreta /
This condition would be visible on ultrasound, unlike what is seen in this patient. increta / percreta, де материнська частина плаценти, desidua basalis, має
Preeclampsia is a condition of new-onset hypertension with either proteinuria or ненормальне прикріплення і відділення після пологів. Ця умова буде видно на УЗД,
end-organ dysfunction after the 20th week of gestation. Abruptio placentae can на відміну від того, що спостерігається у даного пацієнта. Прееклампсія - це стан
be a complication of preeclampsia, but not a cause. This patient’s condition also гіпертонії, що спричиняє новоутворення або з протеїнурією, або з дисфункцією
does not include infertility as a complication. кінцевого органу після 20-го тижня гестації. Abruptio плацента може бути
ускладненням прееклампсії, але не є причиною. Стан цього пацієнта також не
включає безпліддя як ускладнення.
A 57-year-old man with no significant past medical history presents with trouble 57-річний чоловік, який не мав істотної історії хвороби, має проблеми з
urinating for the past 2 months. He has increased urinary frequency and cannot сечовипусканням протягом останніх 2 місяців. Він збільшив частоту сечовипускання і
completely void his bladder. The patient’s review of systems is negative. His не може повністю позбавити сечовий міхур. Огляд пацієнтів системами є негативним.
prostate is enlarged. The examiner was unable to appreciate discrete nodules. Його простата збільшена. Експерт не зміг оцінити дискретні вузлики. Його
His prostate-specific antigen is elevated at 4.4 ng/mL (normal: ≤4 ng/mL). One простатично-специфічний антиген підвищений на 4,4 нг / мл (нормальний: ≤4 нг / мл).
year earlier, his prostate-specific antigen was 3.9 ng/mL. Рік раніше його простатично-специфічний антиген становив 3,9 нг / мл.
Which of the following is an appropriate treatment for this patient’s condition?
Яке з наведеного нижче є відповідним лікуванням для стану цього пацієнта?

A.An α1-adrenergic antagonist


Α1-адренергічний антагоніст
B.An antagonist at muscarinic receptors
Антагоніст мускаринових рецепторів

C.External beam radiation with androgen deprivation Зовнішнє променеве випромінювання з позбавленням андрогенів

Лікування не показано
D.No treatment is indicated
Радикальна простатектомія
E.Radical prostatectomy Показати пояснення
Show Explanations Правильно!
Correct! Цей людина середнього віку з підвищеною частотою сечовипускання і
This middle-aged man with increased urinary frequency and incomplete bladder неповноцінним сечовим міхуром представляє симптоми, характерні для доброякісної
voiding presents with symptoms that are characteristic of benign prostatic гіперплазії передміхурової залози (ДГПЗ). Ця умова може також пояснити збільшену
hyperplasia (BPH). This condition may also explain the patient’s enlarged передміхурову залозу пацієнта, без будь-яких порушень, і злегка підвищений рівень
prostate, without any irregularities, and mildly elevated prostate-specific antigen специфічного антигену простати.
level. Α1-адренергічний антагоніст, такий як тамсулозин, являє собою терапію першої лінії
An α1-adrenergic antagonist, such as tamsulosin, constitutes a first-line при лікуванні ДГПЗ. α1-блокатори - це гладкі м'язові релаксатори, які діють на шийку
therapy in the treatment of BPH. α1 blockers are smooth-muscle relaxers that сечового міхура, що дозволяє легко проходити сечу.
act on the bladder neck, thereby allowing urine to pass through more easily.
Antagonists at muscarinic receptors can lead to urinary retention in people Антагоністи мускаринових рецепторів можуть призвести до затримки сечі у людей з
with BPH. External beam radiation with androgen deprivation and radical ДГПЗ. Зовнішнє променеве випромінювання з позбавленням андрогенів і
prostatectomy are reserved for treatment of prostate cancer and are not радикальної простатектомії зарезервовано для лікування раку передміхурової
indicated for benign hyperplasia of the prostate. Not treating this symptomatic залози і не показані для доброякісної гіперплазії передміхурової залози. Не
patient when there are effective options available is not the best choice. лікування цього симптоматичного пацієнта при наявності ефективних варіантів є не
найкращим вибором.
A 58-year-old obese G3P3 woman comes for evaluation because of heavy 8-річна жінка з ожирінням G3P3 приходить на оцінку через важкі менструальні
menstrual bleeding for the past few months. She denies any nausea, vomiting, кровотечі протягом останніх кількох місяців. Вона заперечує будь-яку нудоту,
or pelvic or abdominal pain. Until the past few months, she had not had a блювоту або тазовий або абдомінальний біль. До останніх місяців у неї не було
menstrual period in 3 years. Onset of menses was at age 10. Her medical менструального циклу через 3 роки. Початок менструації був у віці 10 років. Її
history is significant for type 2 diabetes and hypertension, both of which are анамнез істотний для цукрового діабету 2 типу та гіпертонії, причому обидва з них
managed medically. Her family history is unremarkable. She has a 15-year лікуються медично. Її сімейна історія нічим не примітна. Вона має 15-річну історію
smoking history. She occasionally drinks alcohol but does not take illicit drugs. куріння. Вона іноді вживає алкоголь, але не приймає наркотики. Вона повідомляє, що
She reports having only one sexual partner and denies any previous sexually має тільки одного сексуального партнера і заперечує будь-які попередні
transmitted diseases. захворювання, що передаються статевим шляхом.

On evaluation today, her temperature is 37°C (98.6°F), blood pressure is 138/79 За оцінкою сьогодні, її температура становить 37 ° C (98,6 ° F), артеріальний тиск
mm Hg, pulse is 62/min, and respiratory rate is 15/min. On physical 138/79 мм рт. Ст., Пульс 62 / хв, частота дихання 15 / хв. При медичному огляді
examination, there is abdominal fullness; however, the abdomen is soft and спостерігається повнота живота; однак, живіт м'який і не ніжний, і немає охорони.
non-tender, and there is no guarding. Pelvic examination discloses mild Тазове обстеження виявляє слабку болючість при введенні дзеркала. У
tenderness on insertion of the speculum. An abnormal, watery, blood-tinged вагінальному каналі відзначається ненормальний, водянистий, відтінок крові.
discharge is noted in the vaginal canal. Bimanual palpation of the ovaries is Бімануальна пальпація яєчників нормальна. Для підтвердження підозрюваного
normal. In order to confirm the suspected diagnosis, a hysterectomy is діагнозу проводять гістеректомію. Показано зображення зразка.
performed. An image of a specimen is shown.
Яке з наведеного нижче було б найбільш пов'язане зі зниженням ризику для
діагностики цього пацієнта?

Припинення вживання алкоголю

Пізній початок клімаксу

Негативна сімейна історія злоякісності

Припинення куріння
Втрата ваги
Показати пояснення
Правильно!
Наявність у пацієнта аномального менструального кровотечі у жінки, яка, ймовірно, є
пост або в періменопаузі, є дуже натяком на рак ендометрію. Зразок з гістеректомії
демонструє дезорганізований проліферативний ендометрій з залозистою інвазією
міометрія (див. Лівий нижній кут у віньєтному зображенні).
Основним фактором ризику для більшості карцином ендометрію є підвищена
експозиція естрогену, що може бути наслідком нераціональності, пізньої менопаузи,
раннього менархе та ожиріння. Вважається, що ожиріння збільшує периферичну
Which of the following would be most associated with decreased risk for this конверсію андрогенів в естрогени в жировій тканині. Жінки з надмірною вагою мають
patient's diagnosis? 2-кратне збільшення ризику розвитку раку ендометрію порівняно з ризиком
нормальної ваги; жінки з ожирінням мають 3-кратне збільшення ризику. Втрата ваги
була б ефективним заходом зниження ризику у даного пацієнта. Іншими факторами,
A.Cessation of alcohol consumption
які підвищують ризик розвитку раку ендометрію, є вік, діабет, історія гіперплазії
ендометрію та сімейний анамнез раку кишечника, сечового міхура або матки (тобто
B.Late onset of menopause синдром Лінча).

Інший вибір відповіді не був би найбільш ефективним у запобіганні раку ендометрію.


C.Negative family history of malignancy Пізня менопауза підвищить ризик розвитку раку ендометрію у пацієнта, але не
сприятиме тому, що ожиріння. Вживання алкоголю та генетики / сімейна історія не
D.Smoking cessation підвищують ризик розвитку раку ендометрію, але пов'язані з іншими видами раку,
включаючи рак молочної залози у жінок. Куріння фактично показало зменшення
ризику раку ендометрію у жінок.
E.Weight loss
Show Explanations
Correct!
This patient's presentation of abnormal menstrual bleeding in a woman who is
likely post or perimenopausal is highly suggestive of endometrial
carcinoma. The specimen from a hysterectomy demonstrates disorganized
proliferative endometrium with glandular invasion of the myometrium (see left
lower corner in the vignette image).
The primary risk factor for most endometrial carcinomas is increased estrogen
exposure, which can result from nulliparity, late menopause, early menarche,
and obesity. Obesity is thought to increase peripheral conversion of androgens
to estrogens in adipose tissue. Overweight women have a 2-fold increase in risk
of endometrial cancer compared to those of normal weight; obese women have
a 3-fold increase in risk. Weight loss would have been an effective risk-
reducing measure in this patient. Other factors that would increase the risk of
endometrial cancer are age, diabetes, history of endometrial hyperplasia, and
a family history of bowel, bladder or uterine cancer (ie, Lynch syndrome).
The other answer choices would not be the most effective in preventing
endometrial cancer. Late menopause would increase the patient’s risk of
endometrial cancer but would not contribute as much as obesity. Drinking
alcohol and genetics/family history do not increase the risk of endometrial
cancer but are related to other cancers including breast cancer in
women. Smokinghas actually been shown to decrease the risk of endometrial
cancer in women.
A 34-year-old woman at 35 weeks’ gestation in her first pregnancy comes to the 34-річна жінка у 35-тижневій вагітності у своїй першій вагітності приходить до клініки,
clinic because she is concerned about her weight gain. She gained 3 lb the тому що вона стурбована її збільшенням ваги. Вона набрала 3 фунта на
previous week and notes that her face and hands have gotten very “puffy” попередньому тижні і зазначає, що її обличчя і руки стали дуже «пухкими» останнім
recently, even though she has been eating a healthy diet. The patient denies часом, навіть якщо вона їла здорову дієту. Пацієнт заперечує будь-які зміни в
any changes in bowel habits, nausea, vomiting, or abdominal pain. She takes no звичках кишечника, нудоту, блювоту або біль у животі. Вона не приймає ніяких ліків і
medications and has been otherwise healthy. At her previous prenatal є іншим здоровим. На її попередньому пренатальному призначенні у терміні
appointment at 20 weeks' gestation, she was free of symptoms, and her blood вагітності 20 тижнів вона була вільна від симптомів, і її кров'яний тиск (АТ)
pressure (BP) was within the normal range. During this visit, her temperature is знаходився в межах норми. Під час цього візиту, її температура становить 97,7 ° F
97.7°F (36.5°C), pulse is 95, and BP is 150/90 mm Hg. (36,5 ° C), пульс - 95, а BP - 150/90 мм рт.
Which of the following tests would be the best next step in diagnosing her
condition? Який з наступних тестів буде найкращим наступним кроком у діагностиці її стану?

A.Electroencephalogram Електроенцефалограма

B.Peripheral blood smear Мазок периферичної крові

Рівень сироваткового кортизолу


C.Serum cortisol level
Кетони сечі
D.Urine ketones
Білок сечі
Показати пояснення
E.Urine protein Правильно!
Show Explanations Цей первинний пацієнт має набряк і гестаційну гіпертензію (систолічний АТ> 140 та /
Correct! або діастолічний АТ> 90 після гестації 20 тижнів). Вона, ймовірно, має прееклампсію,
This primigravid patient presents with edema and gestational hypertension яка визначається як гіпертонія з новою настанням з протеїнурією або ознаками
(systolic BP >140 and/or diastolic BP > 90 after 20 weeks' gestation). She likely пошкодження кінцевих органів після 20 тижнів гестації. Щоб підтвердити діагноз,
has preeclampsia, which is defined as new-onset hypertension with proteinuria лікар перевірить білок в сечі за допомогою щупа. (Це виключало б можливу
or signs of end-organ damage after 20 weeks' gestation. To confirm the гестаційну гіпертензію та доброякісний набряк під час вагітності). Для точного
diagnosis, the physician would test for protein in the urine using a dipstick. вимірювання рівнів білка можна також використовувати 24-годинний збір сечі.
(This would rule out possible gestational hypertension and benign edema during Протеїнурію визначають як знаходження 1+ за допомогою щупа з сечею або,
pregnancy.) A 24-hour urine collection can also be used to accurately measure переважно, екскреції білка = 300 мг протягом 24-годинного збору сечі. У таблиці
protein levels. Proteinuria is defined as a finding of 1+ via urine dipstick or, нижче наведено характеристики гіпертензивних розладів, що спостерігаються під час
preferably, excretion of =300 mg of protein during a 24-hour urine collection. вагітності.
The table below summarizes the characteristics of hypertensive disorders seen
in pregnancy. Прееклампсія викликана аномальним розвитком плацентарних спіральних артерій.
Preeclampsia is caused by abnormal development of the placental spiral Аномальна плацентальна судинна мережа призводить до плацентарної гіпоксії і
arteries. Abnormal placental vasculature leads to placental hypoxia and ішемії, що в свою чергу призводить до вивільнення запальних факторів, які
ischemia, which in turn results in release of inflammatory factors that cause викликають ендотеліальне пошкодження материнського кровообігу. Прееклампсія
endothelial injury to the maternal circulation. Preeclampsia affects 7% of зачіпає 7% вагітних у третьому триместрі вагітності і частіше зустрічається у
pregnant women in the third trimester and is more common in patients with pre- пацієнтів з раніше існуючою гіпертонією, діабетом, хронічним захворюванням нирок і
existing hypertension, diabetes, chronic renal disease, and autoimmune аутоімунними порушеннями (наприклад, наявністю антикоагулянтних і
disorders (eg, presence of anticoagulant and anticardiolipin antibodies). антикардіоліпінових антитіл). Ускладнення нелікованої прееклампсії включають
Complications of untreated preeclampsia include placental abruption, відшарування плаценти, коагулопатію, ниркову недостатність, маткову плацентарну
coagulopathy, renal failure, uteroplacental insufficiency, and eclampsia (defined недостатність і еклампсію (визначені як ознаки і симптоми гестозу з судом) та / або
as signs and symptoms of preeclampsia with seizures) and/or HELLP синдром HELLP. Управління прееклампсією включає введення антигіпертензивних
syndrome. Management of preeclampsia involves administration of засобів і внутрішньовенного введення сульфату магнію (для запобігання судом).
antihypertensive agents and intravenous magnesium sulfate (to prevent Остаточне лікування - пологи дитини.
seizures). Definitive treatment is delivery of the baby.
Жоден з інших тестів не був би наступним кращим кроком у постановці діагнозу.
None of the other tests would be the next best step in making a diagnosis.
 A peripheral blood smear would help identify schistocytes suggesting Мазок периферичної крові допоможе ідентифікувати шистоцити, що свідчить про
hemolysis secondary to HELLP syndrome. HELLP syndrome гемоліз, вторинний до синдрому HELLP. Синдром HELLP являє собою ускладнення
represents a complication of untreated preeclampsia, a diagnosis that нелікованого гестозу, діагноз якого не встановлено у даного пацієнта.
has not been established in this patient.
 Urine ketones can be present in diabetic ketoacidosis or starvation Кетони сечі можуть бути присутніми при діабетичному кетоацидозі або станах
states. This patient has been eating a healthy diet and does not have голодування. Цей пацієнт харчується здоровою дієтою і не має анамнезу діабету, що
any history of diabetes, making either of these states unlikely. робить будь-який з цих станів малоймовірним.
 An EEG can be useful for determining seizure activity, which can occur
with eclampsia. However, this patient does not have any indication for ЕЕГ може бути корисним для визначення активності нападу, яка може відбуватися з
an EEG. еклампсією. Однак цей пацієнт не має жодних показань до ЕЕГ.
 Serum cortisol level would be useful in determining hypercortisolism, Рівень сироваткового кортизолу був би корисний при визначенні гіперкортизолізму
or Cushing syndrome. Although this patient has hypertension and або синдрому Кушинга. Хоча у цього пацієнта є гіпертонія і приріст ваги, у неї немає
weight gain, she does not have any other symptoms of excess cortisol, інших симптомів надлишку кортизолу, таких як ожиріння у шнурі, смуги черевної
such as truncal obesity, abdominal striae, or a buffalo hump. порожнини або горб бізонів.

A 37-year-old woman in her second trimester of pregnancy presents to the 37-річна жінка у своєму другому триместрі вагітності подається в кабінет акушера в
obstetrician’s office, accompanied by her husband. They are anxious to discuss супроводі чоловіка. Вони прагнуть обговорити результати скринінгу сироватки
the results of maternal serum screening performed during their previous visit. матері, проведеного під час їх попереднього візиту. Перед зустріччю з парою, акушер
Before meeting with the couple, the obstetrician reviews the results of the tests оглядає результати тестів і виявляє, що рівень сироваткового α-фетопротеїну (AFP),
and finds that the patient’s serum α-fetoprotein (AFP), unconjugated estriol, and некон'югованого естріолу та β-людського хоріонічного гонадотропіну (β-hCG) є
β-human chorionic gonadotropin (β-hCG) levels are all low. The inhibin A низьким. Концентрація інгібіну А є нормальною. Акушер занепокоєний можливістю
concentration is normal. The obstetrician becomes concerned about the вроджених аномалій у плода.
possibility of congenital anomalies in the fetus.
Which of the following cytogenetic abnormalities is most likely present in this Які з наступних цитогенетичних аномалій, швидше за все, присутні в цьому плоді?
fetus?

Мікроделекція 22q11
A.22q11 microdeletion
5p видалення
B.5p deletion Материнська детомія хромосоми 15

C.Maternal uniparental disomy of chromosome 15 Нерозрив хромосоми 13

Нероз'єднання хромосоми 18
D.Nondisjunction of chromosome 13
Нерозрив хромосоми 21
E.Nondisjunction of chromosome 18
Батьківська одноденна дезомія хромосоми 15
Показати пояснення
F.Nondisjunction of chromosome 21 Правильно!
Удосконалений вік матері є фактором ризику генетичних аномалій плода. У цьому
G.Paternal uniparental disomy of chromosome 15 37-річному пацієнті скринінг пренатальної сироватки крові виявляє сузір'я низького
сироваткового AFP, низького некон'югованого естріолу, низького β-HCG і
Show Explanations нормального інгібіну A, що свідчить про плід з трисомією 18, інакше відома як
Correct! синдром Едвардса. Клінічно, синдром Едвардса представлений з вираженим
Advanced maternal age is a risk factor for genetic fetal abnormalities. In this 37- тильцем, ніг на верхівці, інтелектуальною недостатністю, стиснутими кулаками,
year-old patient, prenatal maternal serum screening reveals a constellation of низькочастотними вухами, мікрогнатією і вродженими вадами серця. Смертність
low serum AFP, low unconjugated estriol, low β-HCG, and normal inhibin A, протягом першого року життя є загальною. Трисомії зазвичай є результатом
indicative of a fetus with trisomy 18, otherwise known as Edwards нероз'єднання хромосом під час мейозу; при синдромі Едвардса це нерозрив
syndrome. Clinically, Edwards syndrome presents with prominent occiput, хромосоми 18. Нерозривчастість описує нерівне поділ під час мейозу I, в результаті
rocker-bottom feet, intellectual disability, clenched fists, low-set ears, чого одна клітина отримує три набори однієї хромосоми і одну клітку, яка отримує
micrognathia, and congenital heart disease. Mortality within the first year of life тільки один набір цієї хромосоми. Діагноз трисомії 18 можна підтвердити за
is common. Trisomies are usually the result of nondisjunction of chromosomes допомогою амніоцентезу разом з каріотипом плоду.
during meiosis; in Edwards syndrome, it is the nondisjunction of Інші порушення несумісні з лабораторним профілем, отриманим для даного
chromosome 18. Nondisjunction describes unequal division during meiosis I, пацієнта. Три з цих аномалій не діагностуються за допомогою маркерів сироватки
resulting in one cell receiving three sets of the same chromosome and one cell матері: мікроделекція 22q11, що призводить до синдрому DiGeorge; видалення 5p,
receiving only one set of that chromosome. The diagnosis of trisomy 18 can be що призводить до синдрому кри-ду-чату (вимагає амніоцентезу або клінічного
confirmed by means of amniocentesis along with a fetal karyotype. діагнозу у дитинстві); і материнську і батьківську одноденну дезомію хромосоми 15,
The other abnormalities are inconsistent with the laboratory profile obtained for які викликають синдроми Прадера-Віллі і Ангельмана, відповідно.
this patient. Three of these abnormalities are not diagnosed by maternal serum
markers: a 22q11 microdeletion, which results in DiGeorge syndrome; a 5p Дві цитогенетичні відхилення можуть бути виявлені на пренатальному скринінгу.
deletion, which results in cri-du-chat syndrome (requires amniocentesis or Однак висновки, специфічні для цих умов, відрізняються від тих, що пов'язані з
clinical diagnosis in infancy); and maternal and paternal uniparental disomy нероз'єднанням хромосоми 18, як підсумовано в таблиці. До них відносяться
of chromosome 15, which cause Prader-Willi and Angelman syndromes, нероз'єднання хромосоми 13, що призводить до синдрому Патау і включає в себе
respectively. нормальні рівні АФП і естріолу, а також нероз'єднання хромосоми 21, що призводить
Two cytogenetic abnormalities can be identified on prenatal screening. до трисомії 21 (синдром Дауна), яка пов'язана з підвищеними рівнями β-hCG і
However, findings that are specific to these conditions differ from those інгібіном А.
associated with nondisjunction of chromosome 18, as summarized in the table.
These include nondisjunction of chromosome 13, which results in Patau
syndrome and involves normal levels of AFP and estriol, and nondisjunction of
chromosome 21, resulting in trisomy 21 (Down syndrome), which is associated
with elevated levels of β-hCG and inhibin A.
A 15-year-old girl is brought to the clinic by her mother because of primary 15-річну дівчинку привозять в клініку матір'ю через первинну аменорею. Мати
amenorrhea. The mother states that her daughter has started to refuse to стверджує, що її дочка почала відмовлятися переходити на заняття в тренажерному
change for gym class with the other girls because she feels her body does not залі з іншими дівчатами, оскільки вона відчуває, що її тіло не схоже на тіла інших
look like the other girls' bodies. She reports she has been made fun of by her дівчат. Вона повідомляє, що її висміяли її однокласники, які кажуть, що її приватні
classmates who say her private parts look like a boy's. The mother admits that частини виглядають як хлопчик. Мати визнає, що при народженні дитина мала
at birth her child had clitoromegaly; however, initial laboratory testing during кліторомегалію; однак, на початковому лабораторному тестуванні протягом раннього
infancy yielded a testosterone level of 482 ng/dL (normal: 437–707 ng/dL), an дитинства був виявлений рівень тестостерону 482 нг / дл (нормальний: 437–707 нг /
estrogen level of 12 pg/mL (normal: 10–60 pg/mL), and a luteinizing hormone дл), рівень естрогену 12 пг / мл (нормальний: 10–60 пг / мл) і лютеїнізуючий гормон
level of 8 IU/L (normal: 7–24 IU/L). No further testing was performed at that рівень 8 МО / л (норма: 7-24 МО / л). У цей час не проводилося жодного подальшого
time, and the child has been raised as a female. On physical examination at тестування, і дитина виховувалася як жінка. При медичному огляді в цей візит
this visit, the patient appears tall and very muscularly developed for a teenage пацієнт виявляється високим і дуже м'язово розвиненим для дівчинки-підлітка. Її
girl. Her voice is deeper than what would typically be expected. She declines an голос глибше, ніж можна було очікувати. Вона відхиляє обстеження своїх геніталій,
examination of her genitalia, stating she is too embarrassed. Chromosomal стверджуючи, що вона занадто збентежена. Проведено хромосомне тестування, яке
testing is performed, which reveals a genotype of XY. виявляє генотип XY.
Which of the following disorders does this patient most likely have?
Які з наступних розладів цей пацієнт найчастіше має?
A.5-a-Reductase deficiency
Дефіцит 5-а-редуктази
B.Complete androgen insensitivity
Повна нечутливість андрогенів
C.Double Y syndrome Синдром подвійного Y

D.Female pseudohermaphroditism Жіночий псевдогермафродитизм

Овотестикулярне розлад статевого розвитку


E.Ovotesticular disorder of sex development
Плацентарний дефіцит ароматази
F.Placental aromatase deficiency Показати пояснення
Show Explanations Правильно!
Correct! Ця дитина виховувалася як дівчинка, хоча неоднозначні геніталії були при
This child has been raised as a girl, although ambiguous genitalia were present народженні. Тепер, коли вона досягає статевої зрілості, її тіло змінюється таким
at birth. Now that she is reaching puberty, her body is changing in ways that are чином, що не відповідає цьому гендерному призначенню. Вона переживає зміни її
inconsistent with that gender assignment. She is experiencing changes in her маси тіла і голосу на додаток до відсутності менструації. Ці висновки найбільш
body mass and voice in addition to a lack of menstruation. These findings are узгоджуються з діагнозом дефіциту 5а-редуктази. 5a-Reductase перетворює
most consistent with a diagnosis of 5a-reductase deficiency. 5a-Reductase тестостерон у дигідротестостерон. Дигідротестостерон необхідний для розвитку
converts testosterone to dihydrotestosterone. Dihydrotestosterone is required статевого члена і мошонки під час ембріогенезу. Дитина з дефіцитом 5а-редуктази
for the development of the penis and scrotum during embryogenesis. An infant може бути фенотипічно жіночим, з нормальним рівнем тестостерону, естрогену і
with 5a-reductase deficiency may be phenotypically female, with normal levels лютеїнізуючого гормону; однак, немовля з цим дефіцитом генотипово чоловіче. Різке
of testosterone, estrogen, and luteinizing hormone; however, an infant with this підвищення рівня тестостерону під час статевого дозрівання призводить до
deficiency is genotypically male. The dramatic increase in testosterone levels маскулінізації зовнішніх статевих органів і розвитку чоловічих вторинних статевих
during puberty causes the external genitalia to be masculinized and the ознак. Недостатність менструації зазвичай повідомляється пацієнтами з дефіцитом
development of male secondary sexual characteristics. Failure to menstruate is 5а-редуктази.
commonly reported by patients with 5a-reductase deficiency.

 Placental aromatase deficiency can cause masculinization of female Дефіцит плацентарного ароматази може викликати маскулінізацію жіночих немовлят
infants (ambiguous genitalia) as mentioned in this patient's history. (неоднозначні геніталії), як згадується в історії цього пацієнта. Однак у дитинстві
However, in infancy, serum levels of testosterone and androstenedione сироваткові рівні тестостерону та андростендіона були б підвищені.
would be elevated.
 Female pseudohermaphroditism and ovotesticular disorder of sex Жіноча псевдогермафродитизм і овотестикулярний розлад статевої диференціації
differentiation would present with either internal female genitalia or будуть представлені або внутрішніми жіночими статевими органами, або змішаними
mixed internal genitalia. внутрішніми геніталіями.
 Complete androgen insensitivity would cause high levels of
testosterone. Повна нечутливість андрогенів призведе до високого рівня тестостерону.
 Double Y syndrome would present with a male phenotype from birth.
Синдром подвійного Y представляв би чоловічий фенотип від народження.
A 44-year-old woman presents to her obstetrician-gynecologist for her annual 44-річна жінка подарує своєму акушеру-гінекологу щорічне обстеження. Вона
examination. She reports having diffuse abdominal fullness for the past month повідомляє про дифузну повноту черевної порожнини протягом останнього місяця,
but is otherwise healthy. On examination, she has hypoactive bowel sounds and але в іншому випадку вона здорова. При огляді вона має гіпоактивні звуки кишечника
some abdominal guarding on deep palpation. In addition, a solid mass is noted і деяку черевну охорону при глибокій пальпації. Крім того, тверда маса відзначається
in her right adnexa on bimanual vaginal examination. The rest of the findings on в її правому придатку при бімануальному вагінальному дослідженні. Решта
physical examination are notable for decreased breath sounds and a shifting досліджень на фізичному дослідженні помітні для знижених звуків дихання і
dullness on abdominal palpation. A biopsy specimen of the mass is obtained зміщення тупості при пальпації живота. Отримано біопсійний зразок маси, який
and reveals excess collagen production and spindle cells. виявляє надлишок виробництва колагену і клітини веретена.
Which of the following is the most likely diagnosis?
Який з наступних діагнозів є найбільш вірогідним?
A.Dermoid cyst
Дермоїдна кіста
B.Endometrioma
Ендометріома
C.Meigs syndrome Синдром Мейгса

D.Struma ovarii Struma ovarii


Show Explanations Показати пояснення
Correct! Правильно!
This otherwise healthy woman presents with a 2-month history of abdominal Ця інакше здорова жінка представлена 2-місячним анамнезом абдомінальної
fullness, an ovarian mass, ascites (which manifests as a fluid wave), and повноти, масою яєчників, асцитом (що проявляється як хвиля рідини) і гідротораксом
hydrothorax (pleural effusion) demonstrated by her decreased breath sounds. (плевральним випотом), що демонструється зниженими звуками дихання. Ця тріада -
This triad is the classic presentation of Meigs syndrome, which is caused by a це класичний виклад синдрому Мейгса, який обумовлений доброякісним фібромом
benign ovarian fibroma. It is important to note that unlike other stromal cell яєчників. Важливо відзначити, що на відміну від інших пухлин стромальних клітин,
tumors, the fibromas associated with Meigs syndrome do not secrete sex steroid фіброми, пов'язані з синдромом Мейгса, не виділяють полових стероїдних гормонів.
hormones. The classic histology of a fibroma shows collagen-rich spindle cells, Класична гістологія фіброми показує колаген-багаті шпиндельні клітини, як показано
as seen in this patient's biopsy specimen. в біоптаті цього пацієнта.

The image below shows the monomorphic spindled cells with angulated nuclei,
as well as collagen (the bright pink bands in this image), which is often present На зображенні нижче показані мономорфні шпиндельні клітини з кутовими ядрами, а
in varying amounts. також колаген (яскраво-рожеві смуги на цьому зображенні), які часто присутні в
різних кількостях.
The triad of an ovarian mass, ascites, and hydrothorax is not seen in patients
with the following types of masses: Тріада маси яєчників, асцит і гідроторакс не спостерігається у пацієнтів з такими
 dermoid cyst, usually a benign ovarian tumor that is primarily found in типами мас:
women in their twenties and thirties
 endometrioma, cystic ovarian mass filled with blood and resulting from дермоїдна кіста, як правило, доброякісна пухлина яєчників, яка в основному
ovarian endometriosis зустрічається у жінок у двадцяті і тридцяті роки
 struma ovarii, a specialized, monodermal teratoma composed entirely
of mature thyroid tissue ендометріоми, кістозну масу яєчників, заповнену кров'ю і отримані в результаті
Moreover, these ovarian masses are not composed of spindle cells and ендометріозу яєчників
collagen.
struma ovarii, спеціалізована, монодермальная тератома, повністю складається з
зрілої тканини щитовидної залози

Більш того, ці маси яєчників не складаються з шпиндельних клітин і колагену.


A 58-year-old woman comes for examination because of a 6-month history of a 58-річна жінка приходить на обстеження через 6-місячний анамнез червоного,
red, painful, itchy rash with a burning sensation in her left breast, particularly хворобливого, свербіжного висипу з відчуттям печіння в лівій грудях, особливо
around the nipple and areola. She has no other systemic symptoms. Her навколо соска і ареоли. У неї немає інших системних симптомів. Історія хвороби є
medical history is significant for hypertension, asthma, and untreated solid, важливою для гіпертонії, астми та нелікованих твердих грудних грудок, виявлених у
noncancerous breast lumps discovered at age 22. Results of a biopsy specimen віці 22 років. Результати біоптату з лівого соска пацієнта показані.
from the patient's left nipple are shown.
Який з наступних діагнозів є найбільш вірогідним?

Внутрішньопротокова папілома

Інвазивна дольковая карцинома

Мастит

Хвороба Педжета

Phyllodes пухлина
Показати пояснення
Правильно!
Which of the following is the most likely diagnosis? Пацієнт має екзематозне ураження на лівому соску і відчуття печіння в грудях без
системних симптомів. Гістологія біопсії відзначена наявністю великих клітин,
оточених «ореолами». Це найбільш узгоджується з хворобою Педжета молочної
A.Intraductal papilloma залози, яка є формою аденокарциноми молочної залози, що включає шкіру і
молочнокислі пазухи сосочка.
Боротьба педжета молочної залози найчастіше пов'язана з екскоріаціями, кіркою,
B.Invasive lobular carcinoma виразками і серозно-родинним виділенням. Хвороба ідентифікована на гістології за
наявності великих клітин з чіткою цитоплазмою, що оточує гіперхроматичне ядро. Ці
C.Mastitis клітини відомі як клітини Паджета (чітка цитоплазма часто називають «гало») і може
бути знайдена по всьому епідермі при цьому захворюванні. Хоча хвороба може
проявлятися в будь-який час, вона найчастіше діагностується в п'ятому десятилітті.
D.Paget disease Хвороба грудної залози Педжет майже завжди пов'язана з основним інвазивним
протоковим раком. Існує чотири клінічні стадії хвороби Педжета молочної залози.
E.Phyllodes tumor Прогноз пов'язаний зі стадією захворювання, як і в інших видах раку молочної
Show Explanations залози.
Correct!
The patient presents with an eczematous lesion on the left nipple and a Інші варіанти відповіді не узгоджуються з презентацією цього пацієнта. Інвазивна
burning sensation in the breast with no systemic symptoms. The biopsy дольковая карцинома часто представлена двосторонньо і зазвичай не утворює чіткої
histology is marked by the presence of large cells surrounded by маси. Внутрішньопротокова папілома, як правило, представлена кров'яним
“halos”. This is most consistent with Paget disease of the breast, which is a виділенням сосків, але без шкірних змін. Мастит - це інфекція, яка зазвичай
form of breast adenocarcinoma involving the skin and lactiferous sinuses of the асоціюється з годуванням груддю і має системні симптоми, такі як лихоманка і озноб.
nipple. Філоди пухлини представлені як тверді, multinodular, безболісні маси, які можуть
Paget disease of the breast is most often associated with excoriations, crusting, рости досить великі, щоб спотворити шкіру, але не викликають зміни, що
ulceration, and serosanguineous discharge. The disease is identified on спостерігаються у цього пацієнта.
histology by the presence of large cells with clear cytoplasm surrounding a
hyperchromatic nucleus. These cells are known as Paget cells ( the clear
cytoplasm is frequently referred to as a “halo”) and can be found throughout the
epidermis in this disease. Although the disease can manifest at anytime, it is
most commonly diagnosed in the fifth decade. Paget disease of the breast is
almost always associated with an underlying invasive ductal carcinoma. There
are four clinical stages of Paget disease of the breast. The prognosis is related
to the stage of disease, as in other types of breast cancer.
The other answer choices are not consistent with this patient’s
presentation. Invasive lobular carcinoma often presents bilaterally and usually
does not form a distinct mass. Intraductal papilloma typically presents with
bloody nipple discharge but without skin changes. Mastitis is an infection which
is commonly associated with breastfeeding and has systemic symptoms like
fever and chills. Phyllodes tumors present as firm, multinodular, painless
masses that can grow large enough to distort the skin but do not cause the
changes seen in this patient.

A 36-year-old nulliparous woman presents to her primary care physician 36-річна нерождаюча жінка представляє своєму лікареві первинної медичної
reporting breast pain of several days’ duration. She denies a history of breast допомоги, яка повідомляє про біль у грудях протягом декількох днів. Вона заперечує
cancer in her family but is very worried about the possibility of malignancy. історію раку молочної залози в її сім'ї, але дуже стурбована можливістю малігнізації.
There is no warmth, swelling, or erythema; and there is no bleeding or discharge Немає тепла, набряку або еритеми; і немає ні кровотечі, ні розряду на сосках.
at the nipples. There is no cervical or axillary lymphadenopathy. Breast Відсутня цервікальна або пахвова лімфаденопатія. Обстеження молочної залози
examination reveals several small, mobile lumps on both breasts. The patient виявляє кілька невеликих, рухомих грудок на обох грудях. Пацієнт стверджує, що
states that she thinks she may have felt similar lumps before around the time of вважає, що вона, можливо, відчула подібні грудочки до того часу, коли вона
her periods. перебувала.
Which of the following would a biopsy of the breast lumps most likely reveal?
Яку з наведених нижче результатів біопсія грудної залози може виявити?

A.Central necrosis
Центральний некроз
B.Increase in number of acini and intralobular fibrosis
Збільшення кількості ацинусів і внутрішньошлункового фіброзу

C.Large cells with clear "halos" Великі клітини з чіткими "ореолами"

Залучення лімфи
D.Lymphatic involvement
Листи плеоморфних клітин інфільтрують сусідню строму
E.Sheets of pleomorphic cells infiltrating adjacent stroma Показати пояснення
Show Explanations Правильно!
Correct! Цей пацієнт представляє циклічний, пов'язаний з менструацією біль у грудях без
This patient presents with cyclic, menstruation-associated breast pain without додаткових симптомів. При обстеженні виявлено декілька двосторонніх дрібних,
any additional symptoms. On examination, she is found to have multiple bilateral рухомих грудок. Це класичне уявлення про фіброзно-кістозних змінах молочної
small, mobile lumps. This is the classic presentation of fibrocystic changes of залози.
the breast.

Fibrocystic disease is a benign condition and manifests with diffuse breast pain Фіброзно-кісткова хвороба є доброякісним станом і проявляється при дифузному
and multiple bilateral masses, commonly found in the upper outer quadrants of болі в грудях і множинних двосторонніх масах, зазвичай зустрічаються у верхніх
the breast. It is one of the most common causes of breast lumps in women from зовнішніх квадрантах грудей. Це одна з найбільш поширених причин грудних грудок
the age of 35 years to the onset of menopause. Common risk factors include у жінок у віці від 35 років до початку менопаузи. Загальні фактори ризику включають
nulliparity and being age 35 or older. Fibrocystic changes can be characterized в себе невідповідність і вік 35 років і старше. Гістологічно можна охарактеризувати
histologically as fibrosis (fibrous stroma, indicated by the red arrow in the фіброзно-кістичні зміни як фіброз (фіброзна строма, позначена червоною стрілкою
image) and cysts (indicated by the black arrow in the image). Fibrocystic на зображенні) і кісти (позначені чорною стрілкою на зображенні). Фіброзно-кістичні
changes can cause masses that fluctuate in size with menstruation and caffeine зміни можуть викликати маси, які коливаються в розмірах з менструацією і прийомом
intake. Proliferation of the acini may be seen in patients with fibrocystic кофеїну. Проліферація ацинусів може спостерігатися у пацієнтів з фіброзно-
disease; if present, it may be associated with a low risk of carcinoma. кістозною хворобою; якщо він присутній, він може бути пов'язаний з низьким ризиком
розвитку карциноми.
Central necrosis is characteristic of comedocarcinoma, a subtype of ductal
carcinoma in situ (DCIS). Comedocarcinoma would present with a palpable Центральний некроз характерний для комедокарциноми, підтипу протоковой
mass, nipple discharge, or Paget disease. This patient has no nipple discharge карциноми in situ (DCIS). Комедокарцинома повинна мати відчутну масу, виділення
and no evidence of Paget disease. Large cells with clear ?halos? refer to сосків або хвороба Педжета. У цього пацієнта відсутні соскові виділення і немає
Paget disease, which presents with unilateral eczematous nipple changes. This ознак хвороби Педжета. Великі клітини з чіткими гало? відносяться до хвороби
patient does not have any nipple or skin changes. Lymphatic involvement is Педжета, яка має односторонні зміни екзематозного соска. У цього пацієнта немає
indicative of inflammatory carcinoma, which results in a peau d’orange сосків або змін шкіри. Лімфатична активність є ознакою запальної карциноми, що
appearance of the skin. This patient does not have any erythema, swelling, призводить до появи шкіри. Цей пацієнт не має еритеми, набряків, тепла або змін
warmth, or skin changes, making lymphatic involvement unlikely. Sheets of шкіри, що робить лімфатичну активність малоймовірною. Листи плеоморфних клітин,
pleomorphic cells infiltrating adjacent stroma describes the histology of що інфільтрують сусідню строму, описують гістологію інвазивної протоковой
invasive ductal carcinoma. Invasive ductal carcinoma can present as a firm, карциноми. Інвазивна протокова карцинома може бути представлена як тверда,
irregular, and fixed mass; unilateral bloody nipple discharge; and skin changes; нерегулярна і фіксована маса; односторонній кров'яний розряд сосків; і зміни шкіри;
this patient has none of these findings. цей пацієнт не має жодного з цих знахідок.

An 8-year-old white girl presents with abdominal pain. She has mild tenderness 8-річна біла дівчинка має болі в животі. Вона має легку болючість по всьому животу і
throughout the abdomen and is unable to localize the pain to any particular не в змозі локалізувати біль до будь-якої конкретної точки. У неї також помітна
point. She also has prominent breast tissue. Menses started at 7 years of age. тканина грудей. Менструація почалася з 7 років. Візуалізація виявляє велику
Imaging reveals a large intra-abdominal mass. Histology slide of the mass is внутрішньочеревну масу. Показано слайд гістології маси.
shown.
Виходячи з описаних симптомів, який з наступних варіантів є найбільш імовірним?

Хоріокарцинома

Дисгермінома

Фіброма

Пухлина гранульозних клітин


Based on the described symptoms, which one of the following is most likely?
Аденокарцинома яєчників
A.Choriocarcinoma Показати пояснення
Правильно!
Перераховані симптоми характерні для гіперестрогенії. Пухлини гранульозних клітин
B.Dysgerminoma є пухлинами шлунково-стромальних клітин, які виділяють естроген і, таким чином,
часто проявляються ознаками і симптомами гіперестрогенії. У формі дорослої
людини, яка становить 95% випадків, ці ознаки і симптоми включають гіперплазію
C.Fibroma
або карциному ендометрію, аномальну маткову кровотечу, болючість молочних
залоз, кровотечу в постменопаузі і порушення менструального циклу. У ювенільній
D.Granulosa cell tumor формі пацієнти можуть виявляти передчасне статеве дозрівання через надмірну
продукцію естрогенів. Ці пухлини також можуть проявлятися як великі, безсимптомні
маси на абдомінальному або тазовому дослідженні. Вони мають злоякісний
E.Ovarian adenocarcinoma потенціал. Гістологічно вони свідчать про наявність тіл Call-Exner, які
Show Explanations характеризуються заповненими рідиною порожнинами з появою «розетки», що
Correct! спостерігається при мікроскопічному дослідженні (кола на зображенні).
The symptoms listed are characteristic of hyperestrogenism. Granulosa cell
tumors are sex cord-stromal cell tumors that secrete estrogen, and thus often
manifest with signs and symptoms of hyperestrogenism. In the adult form, which
comprises 95% of cases, these signs and symptoms include endometrial
hyperplasia or carcinoma, abnormal uterine bleeding, breast tenderness,
postmenopausal bleeding, and menstrual irregularities. In the juvenile form,
patients may present with precocious puberty due to excessive estrogen
production. These tumors also may present as large, asymptomatic masses on
abdominal or pelvic examination. They do have malignant potential.
Histologically, they are indicated by the presence of Call-Exner bodies, which
are characterized by fluid-filled cavities with a "rosette" appearance seen on
microscopic exam (circles in image).

An obese 35-year-old woman presents to her physician with a 6-month history Надмірна 35-річна жінка подарує своєму лікареві 6-місячний анамнез відсутності
of absent menstruation. She is extremely worried because she and her husband менструації. Вона надзвичайно стурбована тим, що вона і її чоловік намагаються
have been trying to conceive for more than 1 year with no success. Results of зачати більше 1 року без успіху. Результати фізичного обстеження є нормальними,
physical examination are normal, except the patient has noticed darkly за винятком того, що пацієнт помітив темно-пігментні волосся навколо ареоли,
pigmented hair around her areolas since she stopped menstruating. Blood оскільки перестала менструацію. Артеріальний тиск становить 128/88 мм рт.
pressure is 128/88 mm Hg. Laboratory studies show a luteinizing hormone level Лабораторні дослідження показують рівень лютеїнізуючого гормону 300 мМО / мл,
of 300 mIU/mL, a follicle-stimulating hormone level of 5 mIU/mL, and a thyroid- рівень фолікулостимулюючого гормону 5 мМО / мл і рівень тиреотропного гормону
stimulating hormone level of 0.7 μU/mL. Results of a urine pregnancy test are 0,7 мкМ / мл. Результати тесту на вагітність сечі негативні. Ультразвукове
negative. Ultrasonography shows bilaterally enlarged ovaries and the presence дослідження показує, що в кожному яєчнику двосторонньо збільшені яєчники і
of greater than 12 follicles, measuring roughly 2–9 mm in diameter, in each наявність більше 12 фолікулів діаметром приблизно 2-9 мм.
ovary.
Which of the following additional findings is most commonly associated with her Які з наступних висновків найчастіше асоціюються з її станом?
condition?

Галакторея
A.Galactorrhea
Гіперглікемія
B.Hyperglycemia
Гіпосмія

C.Hyposmia Рак яєчників

Полікістоз нирок
D.Ovarian Cancer Показати пояснення
Правильно!
E.Polycystic kidneys Виявлення пацієнтом ожиріння, аменорея, утруднення зачаття, періареолярний ріст
Show Explanations волосся, надлишок лютеїнізуючого гормону (LH) і множинні фолікули в кожному
Correct! яєчнику вказують на діагноз синдрому полікістозних яєчників (СПКЯ або синдрому
The patient’s presentation of obesity, amenorrhea, difficulty conceiving, Штейна-Левенталь).
periareolar hair growth, excess luteinizing hormone (LH), and multiple follicles
within each ovary points to a diagnosis of polycystic ovarian syndrome
(PCOS, or Stein-Leventhal syndrome). СПКЯ є найбільш поширеним ендокринним розладом у жінок дітородного віку, в
основному характеризується оліго / ановуляцією і гіперандрогенією через поєднання
PCOS is the most common endocrine disorder of women of childbearing age, генетичних та екологічних факторів. Особливості, що спостерігаються при СПКЯ,
mainly characterized by oligo/anovulation and hyperandrogenism due to a включають кісти яєчників, аменорею, безпліддя, ожиріння і гірсутизм (в даному
combination of genetic and environmental factors. Features seen in PCOS випадку, періареолярний ріст волосся), викликаний надлишковим виробництвом ЛГ
include ovarian cysts, amenorrhea, infertility, obesity, and hirsutism (in this case, та андрогенами. Діагноз заснований на двох з наступних трьох висновків:
periareolar hair growth) caused by excess LH production and androgens. ановуляція, високий рівень андрогенів і кісти яєчників. СПКЯ симптоматично
Diagnosis is based on two of the following three findings: anovulation, high управляється із змінами способу життя, оральними контрацептивами,
androgen levels, and ovarian cysts. PCOS is symptomatically managed with антиандрогенами тощо.
lifestyle changes, oral contraceptives, anti-androgens, etc.

In some women PCOS is associated with insulin resistance and У деяких жінок СПКЯ асоціюється з резистентністю до інсуліну і гіперінсулінемією,
hyperinsulinemia, which increases androgen production in the ovarian theca що підвищує вироблення андрогенів у клітинах тека яєчників і, по-друге, у
cells and, secondarily, LH production. Insulin resistance leads to виробництві ЛГ. Резистентність до інсуліну призводить до гіперглікемії, а також
hyperglycemia and also suppresses hepatic steroid hormone-binding globulin пригнічує синтез печінкового стероїдного гормон-зв'язуючого глобуліну (SHBG).
(SHBG) synthesis. The decrease in SHBG along with the increase in androgen Зниження SHBG поряд зі збільшенням виробництва андрогенів призводить до
production leads to a vicious cycle of amenorrhea and infertility. порочного циклу аменореї та безпліддя.

Hyposmia, or decreased sense of smell, is a presentation of Kallmann


syndrome. Amenorrhea and galactorrhea are signs of hyperprolactinemia Гіпосмія, або зменшений нюх, являє собою синдром Каллмана. Аменорея і
which is associated with decreased levels of LH. Polycystic kidneys are not галакторея є ознаками гіперпролактинемії, яка пов'язана зі зниженням рівня ЛГ.
associated with PCOS. This patient’s condition can be associated with ovarian Полікістоз нирок не пов'язаний з СПКЯ. Стан цього пацієнта може бути пов'язаний з
cancer, but a link is not confirmed. PCOS is more often associated with an раком яєчників, але зв'язок не підтверджено. СПКЯ частіше асоціюється з
increased risk of endometrial cancer. підвищеним ризиком раку ендометрію.
A 36-year-old woman, G3P2, presents to her obstetrician for her first scheduled 36-річна жінка, G3P2, подарує своєму акушер-гінекологу для першого запланованого
prenatal examination. Her uterus is larger than it should be for the gestational дородового обстеження. Її матка більше, ніж повинна бути для гестаційного віку 10
age of 10 weeks, as calculated on the basis of the date of her last menstrual тижнів, розрахована на основі дати її останньої менструації. Рівень β-людського
period. Her β-human chorionic gonadotropin (β-hCG) level is elevated to хоріонічного гонадотропіну (β-hCG) підвищений до 268000 мУ / мл. Здійснюється
268,000 mU/mL. A transvaginal ultrasound of the uterus is performed, and трансвагінальне ультразвукове дослідження матки, а на зображенні показані
findings are shown in the image. результати.

Які найбільш поширені каріотипи і найбільш звичайні хромосомні походження,


пов'язані з цими знахідками?

45, XO; як материнські, так і батьківські

46, XX; всі материнські похідні

46, XX; всі батьківські похідні

69, XXX; додатковий материнський набір

69, XXY; додатковий батьківський набір


Показати пояснення
Правильно!
Цей пацієнт має матку, більшу, ніж очікувалося, для гестаційного віку, рівень β-hCG
підвищений до> 100.000 мУ / мл і ультразвук, який виявляє появу «мішка винограду»
або «метели», але не має частин плоду. Для нормальної одиночної вагітності на 10-
му тижні рівень β-ХГЧ може становити від 25,700 до 288000 ммоль / мл; однак у
матку повинна спостерігатися одна вагітність, що не стосується цього пацієнта.

Презентація даного пацієнта свідчить про наявність повної або класичної молі, яка
складається з виключно батьківської вагітності без будь-яких частин плоду, і
пов'язана з розміром матки, що є значно більшим за тривалістю вагітності і високим
рівнем β-hCG. Ультразвук цього пацієнта показує, що матка розширена гетерогенно
ехогенною масою з множинними гіпоехогенними вогнищами (зовнішній вигляд
«мішок винограду»). Навпаки, ультразвукове дослідження часткової молі може
показувати частини плоду.

Повні молі є диплоїдними, і обидва набори хромосом одержані від батьків. Повні
моли найчастіше виникають, коли енуклеірованную яйцеклітину запліднені однією
спермою, яка потім дублюється. Часткові молі виникають в результаті поліплоїдного
запліднення з додатковим набором хромосом, отриманих від батьків. Обидва типи
кротів можуть мати при вагінальних кровотечах, тазовому болі, збільшенні β-hCG і
збільшенні матки. Однак, коли присутня повна моль, ці ознаки спостерігаються
раніше, і симптоми більш серйозні.

Опис 69, XXY; додатковий батьківський набір застосовується до часткової молі, що є


результатом поліплоїдного запліднення з додатковим набором хромосом, отриманих
від батьків. Частково молі зазвичай мають ознаки втрати плоду, а не збільшену
матку і підвищену β-hCG. Опис 69, XXX; додатковий материнський набір не
застосовується ні до часткової, ні до повної молі, тому що всі молярні вагітності
Image courtesy of Mikael Häggström мають додатковий набір хромосом батьківства. Каріотип 45, XO характерний для
What are the most common karyotype and the most usual chromosomal origin синдрому Тернера, який не має видимих аномалій матки. Повна моль може мати
associated with these findings? каріотип 46, XX, але не матиме набір материнських хромосом.

A.45,XO; both maternally and paternally derived

B.46,XX; all maternally derived

C.46,XX; all paternally derived

D.69,XXX; extra maternal set

E.69,XXY; extra paternal set


Show Explanations
Correct!
This patient presents with a uterus larger than expected for gestational age, a ß-
hCG level elevated to >100,000 mU/mL, and an ultrasound revealing a “bag of
grapes” or “snowstorm” appearance but no fetal parts. For a normal singleton
pregnancy at 10 weeks, the ß-hCG level can range from 25,700–288,000
mIU/mL; however, a single gestation should be seen in the uterus, which is not
the case for this patient.
This patient's presentation suggests the presence of a complete or classic
mole, which consists of an exclusively paternally derived pregnancy without any
fetal parts, and associated with a uterine size that is significantly large for length
of gestation and a highly elevated ß-hCG level. This patient's ultrasound shows
the uterus to be distended by a heterogeneously echogenic mass with multiple
hypoechoic foci (“bag of grapes” appearance). In contrast, an ultrasound of a
partial mole might show fetal parts.
Complete moles are diploid, and both sets of chromosomes are paternally
derived. Complete moles most commonly occur when an enucleated ovum is
fertilized by a single sperm, which then duplicates. Partial moles result from
polyploid fertilization with an extra set of paternally derived chromosomes. Both
types of moles can present with vaginal bleeding, pelvic pain, increased ß-hCG,
and uterine enlargement. However, when a complete mole is present, these
signs are observed earlier, and symptoms are more severe.

The description 69,XXY; extra paternal set applies to a partial mole, which
results from polyploid fertilization with an extra set of paternally derived
chromosomes. Partial moles usually present with signs of fetal loss rather than
an enlarged uterus and increased ß-hCG. The description 69,XXX; extra
maternal set would not apply to either a partial or complete mole because all
molar pregnancies have an extra set of paternal chromosomes. The karyotype
45;XO is characteristic of Turner syndrome, which does not present with visible
uterine abnormalities. A complete mole can have a karyotype of 46,XX but
would not have a set of maternal chromosomes.

An 18-year-old woman comes to the clinic with a “stomach ache.” Upon further 18-річна жінка приїжджає до клініки з «болем у животі». Після подальшого
investigation, she describes the pain as crampy and says it only occurs during дослідження вона описує біль як спастичну хворобу і каже, що вона виникає лише
menstruation. Ibuprofen provides only partial relief. She has no significant past під час менструації. Ібупрофен забезпечує лише часткове полегшення. Вона не має
medical history and previous investigations into her pain have been істотної історії хвороби і попередні дослідження її болю були безрезультатними. Її
inconclusive. Her last menstrual period 4 weeks ago was regular. She is останній менструальний період 4 тижні тому був регулярним. Вона сексуально
sexually active and endorses consistent condom use. She denies constipation, активна і підтримує послідовне використання презервативів. Вона заперечує запори,
painful sexual intercourse, or heavy menstrual bleeding. Her temperature is хворобливі статеві зносини або важкі менструальні кровотечі. Її температура
98.7° F (37° C), blood pressure is 110/72 mm Hg, pulse is 71/min, respiratory становить 98,7 ° F (37 ° C), артеріальний тиск 110/72 мм рт. Її живіт м'який і
rate is 14/min, and oxygen saturation is 99% on room air. Her abdomen is soft нешкідливий, хоча вона стверджує, що біль виникає навколо надлобкової ділянки. Її
and nontender, though she endorses that the pain occurs around the suprapubic матка передня, не збільшена, непухла. Двосторонні присадки є непухлинними до
area. Her uterus is anterior, not enlarged, and nontender. Bilateral adnexa are пальпації. Бета-hCG з сечею негативний. Оцінка ультразвуком не виявляє значних
nontender to palpation. A urine beta-hCG is negative. Ultrasound evaluation анатомічних змін.
reveals no significant anatomic changes.
Which of the following is the most likely cause of the patient's condition? Яке з наступних причин є найбільш вірогідною причиною стану пацієнта?

A.Ectopic endometrial tissue Ектопічна тканина ендометрію

Зростання фолікула викликає розтягнення яєчників


B.Follicle growth causing ovarian distension
Збільшення продукції простагландинів
C.Increased prostaglandin production
Кісти яєчників розміром більше 8 мм в діаметрі
D.Ovarian cysts measuring greater than 8mm in diameter Плазми гладких м'язів

E.Smooth muscle neoplasm Потовщений міометрій


Показати пояснення
Правильно!
F.Thickened myometrium Пацієнт - це інакше здорова, сексуально активна 18-річна жінка, яка має болі в
Show Explanations животі, що виникають під час менструального циклу. Її фізичний огляд непримітний,
Correct! а тест на вагітність сечі негативний. Таким чином, біль більшою мірою нагадує
The patient is an otherwise healthy, sexually active 18-year-old female первинну дисменорею.
presenting with abdominal pain occurring during her menstrual cycle. Her
physical exam is unremarkable and urine pregnancy test is negative. Thus, her Патогенез первинної дисменореї пов'язаний з менструальним циклом. Незадовго до
pain is most suggestive of primary dysmenorrhea. початку менструацій, рівні простагландину збільшуються, що звужує кровоносні
The pathogenesis of primary dysmenorrhea is linked to the menstrual cycle. судини і стискає матку, викликаючи тазову біль і дискомфорт. Оскільки нестероїдні
Shortly before the onset of menses, prostaglandin levels increase, which протизапальні препарати (НПЗП) пригнічують вироблення простагландинів, вони є
constricts blood vessels and contracts the uterus, causing pelvic pain and ефективним методом першої лінії лікування первинної аменореї. Пероральні
discomfort. Because nonsteroidal anti-inflammatory drugs (NSAIDs) inhibit протизаплідні таблетки також ефективні за рахунок зменшення проліферації
prostaglandin production, they are effective first-line treatment for primary ендометрію та виробництва простагландинів.
amenorrhea. Oral contraceptive pills are also effective by decreasing
endometrial proliferation and prostaglandin production. Якщо біль у пацієнта не реагує на НПЗП або оральні контрацептиви, слід
If a patient’s pain is unresponsive to NSAIDs or oral contraceptives, one should враховувати етіологію вторинної дисменореї. Такі етіології включають в себе стеноз
consider etiologies of secondary dysmenorrhea. Such etiologies include cervical шийки матки, лейоміоми, запальні захворювання органів малого тазу, або тазові
stenosis, leiomyomas, pelvic inflammatory disease, or pelvic adhesions. спайки. Однак лікування вторинної дисменореї є специфічним для основної причини.
However, treatment for secondary dysmenorrhea is specific to the underlying
cause. Інші варіанти відповіді неправильні з наступних причин:
The other answer choices are incorrect for the following reasons:
A thickened myometrium suggests adenomyosis. Adenomyosis occurs when Потовщений міометрій припускає аденоміоз. Аденоміоз виникає, коли ектопічні
ectopic endometrial glands and stroma are present within the myometrium. ендометріальні залози і строма присутні в міометрії. Часті ознаки і симптоми
Common signs and symptoms include a symmetrically enlarged and boggy включають симетрично збільшену і заболоченную матку. Ектопічна тканина
uterus. Ectopic endometrial tissuedescribes endometriosis and typically ендометрію описує ендометріоз і зазвичай має меноррагію, дисхезію і / або
presents with menorrhagia, dyschezia, and/or dyspaurenia. None of these are дисфоуренню. Жоден з них не спостерігається у даного пацієнта. Лейоміоми, також
seen in this patient. Leiomyomas, also known as fibroids, are benign smooth відомі як фіброміоми, є доброякісними пухлинами гладких м'язів, які можуть бути
muscle tumors that can present as menorrhagia with a bulky uterus on exam представлені у вигляді меноррагії з громіздкою маткою після обстеження, але
but is unlikely given this patient’s normal physical exam findings. Ovarian малоймовірно з урахуванням нормальних результатів обстеження пацієнта. Кісти
cysts can be suggestive of polycystic ovarian syndrome but would typically be яєчників можуть свідчити про синдром полікістозних яєчників, але, як правило,
accompanied with signs of hyperandrogenism, which is not seen in this супроводжуються ознаками гіперандрогенії, що не спостерігається у даного пацієнта.
patient. Mittelschmerz is typically mid-menstrual cycle pain caused by rapid Mittelschmerz, як правило, біль в середньому менструальному циклі, викликаний
growth of the follicle before release. швидким ростом фолікула до вивільнення.
A 29-year-old woman who has never given birth presents to her gynecologist, 29-річна жінка, яка ніколи не народила подарунки своєму гінекологу, повідомляє про
reporting heavy vaginal bleeding during and occasionally between her periods. важкі кровотечі з піхви під час і іноді між періодами. Вона має індекс маси тіла 23.
She has a body mass index of 23. Pelvic examination reveals a normal vagina Тазове дослідження виявляє нормальне піхву і шийку матки без уражень або
and cervix with no lesions or abnormal discharge. Bimanual examination is ненормальних виділень. Бімануальне дослідження характерне для нормальних
notable for normal adnexa and an enlarged uterus with asymmetric contour. придатків і збільшеної матки з асиметричним контуром.

The gynecologist orders an an ultrasound of the pelvis, which reveals a mass Гінеколог замовляє ультразвукове дослідження малого тазу, який виявляє масу в
measuring 6 × 5 cm in the uterine submucosa. The submucosal mass is розмірі 6 × 5 см в підслизовій оболонці матки. Ендоскопічно видаляють підслизову
resected endoscopically, and a histopathologic specimen is obtained (see масу, отримують гістопатологічний зразок (див. Малюнок).
image).
Яке з наступного найчастіше асоціюється зі станом цього пацієнта?

Збільшення маси при менопаузі

Збільшення маси при вагітності

Формування шоколадних кіст

Злоякісна трансформація маси

Запальні захворювання тазових органів


Показати пояснення
Правильно!
У даного пацієнта спостерігаються важкі менструальні кровотечі (меноррагія),
кровотечі між періодами і асиметрично збільшена матка. Патологічний слайд
демонструє монотонну популяцію шпильчастих клітин з тупими, "сигарообразними"
ядрами і рясною еозинофільної цитоплазмою. Клітини розташовані в пересічних
пучках, що викликають потік ядер, як школа риби - класичний виклад і мікроскопічний
опис фіброми або лейоміоми. Фіброміди є звичайними пухлинами гладких м'язів, які
зазвичай виявляються при численних масах. Ці пухлини є доброякісними і можуть
бути пов'язані з дисменореєю (менструальна біль), меноррагією, менометрорагією
(часті менструальні кровотечі, що є надмірними і нерегулярними за обсягом і
тривалістю), безпліддям і ненормальними результатами при тазовому обстеженні,
включаючи пальпуються маси, що проходять від матки.
Оскільки міома чутлива до естрогенів, вони мають тенденцію до збільшення розмірів
під час менструації або вагітності та зменшення розміру після менопаузи. Лікування
міоми залежить від тяжкості симптомів і проблем фертильності. Гістеректомія
виконується для жінок з важкими симптомами, які не бажають зберегти
фертильність. Деяким жінкам може бути корисною емболізація міоми матки,
лікування, що зберігає фертильність, при якому емболічні агенти вводяться в маткові
артерії. Терапевтична мета полягає в блокуванні фіброїдних кровоносних судин,
викликаючи зменшення розміру пухлин і загибелі. Важливо відзначити, що
міомектомія не використовується при лікуванні підслизової лейоміоми, оскільки
призводить до рубцювання і подальшого безпліддя. Медикаментозну терапію можна
спробувати за допомогою монотерапії або комбінації аналогів гонадотропін-
вивільняючого гормону (леупролида), комбінованих оральних контрацептивів,
прогестинів та / або антагоністів гонадотропного вивільнення гормону (цетрорелікса).

Маса, що збільшується після менопаузи, може свідчити про рак матки або
ендометрію. Формування шоколадних кіст пов'язане з ендометріозом. Ендометріоз
зазвичай має диспареунию, дизезію і пальпируемую, ніжну вузликость, якої у цього
пацієнта немає. Злоякісна трансформація маси дуже рідко зустрічається з ділянками
некрозу і кровотечами мікроскопічно. Запальні захворювання на тазову оболонку
часто притаманні віддушним виділенням з піхви, болю при статевому акті і
сечовиділенням, а також історією декількох незахищених сексуальних контактів.

Image courtesy of Dr. Matt Quick.


Which of the following is most commonly associated with this patient's
condition?

A.Enlargement of the mass with menopause

B.Enlargement of the mass with pregnancy

C.Formation of chocolate cysts

D.Malignant transformation of the mass

E.Pelvic inflammatory disease


Show Explanations
Correct!
This patient presents with heavy menstrual bleeding (menorrhagia), bleeding
between periods, and an asymmetrically enlarged uterus. The pathology slide
shows a monotonous population of spindled cells with blunt-ended, "cigar-
shaped" nuclei and abundant eosinophilic cytoplasm. The cells are arranged in
intersecting fascicles causing the nuclei to stream like a school of fish—the
classic presentation and microscopic description of a fibroid or leiomyoma.
Fibroids are common smooth muscle tumors that usually manifest with multiple
masses. These tumors are benign and can be associated with dysmenorrhea
(menstrual pain), menorrhagia, menometorrhagia (frequent menstrual bleeding
that is excessive and irregular in amount and duration), infertility, and abnormal
findings on pelvic examination including palpable masses extending from the
uterus.
Because fibroids are estrogen sensitive, they tend to increase in size during
menses or pregnancy and decrease in size after menopause. Treatment of
fibroids is dependent on the severity of symptoms and fertility concerns.
Hysterectomy is performed for women with severe symptoms who do not wish
to preserve fertility. Some women may benefit from uterine fibroid embolization,
a fertility-sparing treatment in which embolic agents are introduced to the uterine
arteries. The therapeutic goal is to block the fibroid blood vessels, causing the
tumors to shrink and die. It is important to note that myomectomy is not used in
the treatment of submucosal leiomyomas, since it leads to scarring and
subsequent infertility. Medical therapy can be tried with monotherapy or
combinations of gonadotropin-releasing hormone analogs (leuprolide),
combined oral contraceptives, progestins, and/or gonadotropin-releasing
hormone antagonists (cetrorelix).
A mass that enlarges after menopause would be suggestive of uterine or
endometrial cancer. Formation of chocolate cysts is associated with
endometriosis. Endometriosis usually presents with dyspareunia, dyschezia,
and palpable, tender nodularity, which this patient does not have. Malignant
transformation of the mass is very rare and would present with areas of
necrosis and hemorrhage microscopically. Pelvic inflammatory
disease commonly presents with odiferous vaginal discharge, pain with sexual
intercourse, and urinary symptoms, as well as a history of multiple unprotected
sexual encounters.
A 30-year-old man presents to his primary care physician after noticing a 30-річний чоловік подарує своєму лікареві первинної медичної допомоги після того,
painless lesion on his penis. He denies any burning when urinating and does not як помітив безболісне ураження на його статевому члені. Він заперечує будь-яке
report itchiness. He denies having traveled anywhere within the last 5 years. He печіння при сечовипусканні і не повідомляє про свербіж. Він заперечує, що проїхав
has had four new male and female sexual partners over the past 6 months, and десь протягом останніх 5 років. За останні 6 місяців він мав чотирьох нових чоловічих
he admits to not using contraception regularly. He drinks a couple of beers on і жіночих статевих партнерів, і він визнає, що регулярно не використовує
weekends but does not use tobacco or illicit drugs. His vital signs are as follows: контрацепцію. Він випиває пару сортів пива у вихідні дні, але не вживає тютюну або
Temperature: 98.9°F (37.2°C) незаконних наркотиків. Його життєві ознаки такі:
Heart rate: 71
Respiratory rate: 16 Температура: 98 ° F (37,2 ° C)
Blood pressure: 132/82 mm Hg
Частота серцевих скорочень: 71
On examination of his genitalia, a single, nontender ulcerative lesion is seen on
the dorsal aspect of the penis. There are no palpable masses or Частота дихання: 16
lymphadenopathy. The rest of the findings of the physical examination are
unremarkable. Артеріальний тиск: 132/82 мм рт
If left untreated, which of the following is most likely to occur in this patient?

При огляді його статевих органів на дорсальній стороні статевого члена видно одне,
A.Esophagitis, encephalitis, and keratitis невразливе виразкове ураження. Немає відчутних мас або лімфаденопатії. Решта
висновків фізичного обстеження не є примітними.
B.Fever with joint pain and synovial inflammation
Якщо не лікувати, яке з наступних випадків найчастіше має місце у даного пацієнта?
C.Inguinal adenopathy and fevers
Езофагіт, енцефаліт і кератит
D.Purulent urethral discharge, with swelling and fever
Лихоманка з болями в суглобах і синовіальним запаленням

E.Subcutaneous granulomas, dementia, and personality changes Пахові аденопатії і лихоманки


Show Explanations
Correct! Гнійне відділення уретри, з набряком і лихоманкою
This sexually active patient with a painless ulcerative lesion on his penis most
likely has primary syphilis caused by Treponema pallidum. Venereal Disease Підшкірні гранульоми, деменція і особистість змінюються
Research Laboratory (VDRL) titers and fluorescent treponemal antibody Показати пояснення
absorption (FTA-ABS) testing may be used to confirm the diagnosis. Правильно!
Syphilis is categorized by the following stages: Цей сексуально активний пацієнт з безболісним виразковим ураженням на своєму
статевому члені, швидше за все, має первинний сифіліс, викликаний Treponema
 Primary syphilis: In men, this stage is characterized by a painless pallidum. Для підтвердження діагнозу можна використовувати титри лабораторних
chancre, usually on the penis, which develops about 3 weeks after досліджень венеричних захворювань (VDRL) і тестування флуоресцентних
exposure. Women with primary syphilis may appear to be free of трепонемних антитіл (FTA-ABS).
symptoms because the chancre may develop in the vagina or rectum.
 Secondary syphilis: This disseminated stage of the disease develops Сифіліс класифікується за такими стадіями:
within 4 to 10 weeks after primary infection and may manifest with a
maculopapular rash, condyloma lata, lymphadenopathy, hair loss,
muscle aches, fever, sore throat, and swollen lymph nodes. A latent, Первинний сифіліс: У чоловіків цей етап характеризується безболісним шанкром, як
asymptomatic stage may follow and last for years. правило, на статевому члені, який розвивається приблизно через 3 тижні після
 Tertiary syphilis develops late in about 40% of patients who have gone опромінення. Жінки з первинним сифілісом можуть виявитися вільними від
untreated. It is characterized by subcutaneous granulomas (gummas). симптомів, оскільки може розвиватися шанкр у піхві або прямої кишки.
Although neurosyphilis can occur at any time after initial infection, it is
more likely to present with tertiary syphilis and manifest with dementia, Вторинний сифіліс: Ця дисемінована стадія захворювання розвивається протягом від
personality changes, and posterior spinal cord degeneration (tabes 4 до 10 тижнів після первинної інфекції і може проявлятися з макулопапулярними
dorsalis). висипами, кондиломою, лімфаденопатією, випаданням волосся, м'язовими болями,
Note that symptoms are generally associated with a single stage but may also лихоманкою, біль у горлі і набряклими лімфатичними вузлами. Латентна,
overlap. безсимптомна стадія може слідувати і триватиме протягом багатьох років.
Syphilis is treated with penicillin G.
Herpes simplex, which can disseminate to cause encephalitis, keratitis, and Третинний сифіліс розвивається пізно у близько 40% пацієнтів, які не пройшли
esophagitis, presents with painful, shallow ulcers. Gonorrheal infections, which лікування. Характеризується підшкірними гранульомами. Хоча нейросифіліс може
can cause septic arthritis—characterized by fever with joint pain and synovial виникнути в будь-який час після первинної інфекції, він, швидше за все, представляє
inflammation—do not present with penile ulcers or lesions and are usually третинний сифіліс і проявляється деменцією, змінами особистості і задньою
asymptomatic in men. Likewise, chlamydia is usually an asymptomatic infection дегенерацією спинного мозку (tabes dorsalis).
but can present with purulent urethral discharge and fever or inguinal
lymphadenopathy and swelling, depending on the serovar. (D-K serovars Зауважте, що симптоми, як правило, пов'язані з однією стадією, але можуть також
cause urethritis; L1-L3 serovars cause lymphogranulosa venereum.) перекриватися.
Сифіліс обробляють пеніциліном Г.

Симплекс герпесу, який може поширюватися, щоб викликати енцефаліт, кератит і


езофагіт, має болючі, дрібні виразки. Гонорейні інфекції, які можуть викликати
септичний артрит - характеризуються лихоманкою з болями в суглобах і
синовіальним запаленням - не мають виразки або ураження статевого члена і
зазвичай асимптоматичні у чоловіків. Крім того, хламідія зазвичай є безсимптомною
інфекцією, але може проявлятися з гнійними виділеннями уретри і лихоманкою або
пахової лімфаденопатією і набряком, залежно від серовару. (Серовари D-K
викликають уретрит; серовари L1-L3 викликають lymphogranulosa venereum).
A 32-year-old man visits his primary care physician with concerns about his 32-річний чоловік відвідує свого лікаря первинної медико-санітарної допомоги з
fertility. He and his wife have attempted to conceive for the past 2 years and побоюваннями щодо його родючості. Він і його дружина намагалися зачати протягом
have grown frustrated with lack of success. He was recently diagnosed with останніх 2 років і розчарувалися відсутністю успіху. Нещодавно йому був
moderate hypertension but is otherwise healthy. He recently lost his job and поставлений діагноз помірної гіпертензії, але він інакше здоровий. Нещодавно він
admits to having increased life stressors and difficulty sleeping. The patient’s втратив роботу і визнає, що має підвищені стресові ситуації та труднощі зі сном.
blood tests show his total serum testosterone level is somewhat low and his Аналізи крові пацієнта показують, що його загальний рівень тестостерону в сироватці
luteinizing hormone level is high. трохи низький, а рівень лютеїнізуючого гормону високий.
Which of the following procedures will diagnose the most likely cause of this
man’s infertility? Які з наступних процедур діагностують найбільш імовірну причину безпліддя цієї
людини?

A.Abdominal CT scan
КТ черевної порожнини
B.Electron microscopy showing ciliary movement
Електронна мікроскопія, що демонструє цилиарное рух

C.Karyotyping Каріотипування

Тест на запах
D.Smell test
Дослідження сперми / моторики
E.Sperm count/motility studies Показати пояснення
Show Explanations Правильно!
Correct! Цей пацієнт чоловічої статі має безпліддя, помірну гіпертензію, низький рівень
This male patient presents with infertility, moderate hypertension, low levels of тестостерону та високий рівень лютеїнізуючого гормону. Цей чоловік, ймовірно, має
testosterone, and high levels of luteinizing hormone. This man likely has синдром Klinefelter, генетичне захворювання, яке виникає у 1/850 чоловічих суб'єктів.
Klinefelter syndrome, a genetic disorder that occurs in 1/850 male subjects. У чоловіків з Klinefelter є атрофія яєчка, форми жіночого тіла (часто, але не завжди, з
Men with Klinefelter have testicular atrophy, feminine body shapes (often, but гінекомастією), довгі кінцівки і розріджені волосся на тілі. Цей діагноз є поширеною
not always, presenting with gynecomastia), long extremities, and sparse body причиною гіпогонадизму, виявленого в процесі безпліддя.
hair. This diagnosis is a common cause of hypogonadism found in an infertility
work-up.
Сироватковий рівень ЛГ у цього пацієнта є високим вторинним по відношенню до
This patient’s serum level of LH is high secondary to his underlying testicular його атрофії основних яєчок. Результативна аномальна функція клітин Лейдіга
atrophy. Resultant abnormal Leydig cell function leads to the decrease in serum призводить до зниження рівня тестостерону в сироватці, таким чином змінюючи
testosterone, thus altering the normal feedback loop with the anterior pituitary нормальну петлю зворотного зв'язку з передньою гіпофізою, яка зазвичай вивільняє
that normally releases LH to stimulate testosterone synthesis in Leydig cells. ЛГ для стимулювання синтезу тестостерону в клітинах Лейдіга.

The genotype in Klinefelter syndrome is XXY (shown in the image). As there are
two X chromosomes in these men, a Barr body (inactivated X chromosome) can Генотип у синдромі Клайнфельтера - це ХХY (показано на зображенні). Оскільки у
be seen on karyotyping and occasionally on microscopic examination at the цих чоловіків є дві Х-хромосоми, то тіло Барра (інактивована Х-хромосома) можна
cellular level. побачити на каріотипуванні, а іноді і на мікроскопічному дослідженні на клітинному
Note that in rare situations, 46,XX men also can have Klinefelter syndrome; in рівні.
this setting, the development of testes presumably is due to translocation of a
small portion of chromosomal material containing the testis-determining factor to Зауважимо, що в рідкісних ситуаціях 46, XX чоловіки також можуть мати синдром
an X chromosome. Клайнфельтера; у цій ситуації розвиток яєчок, ймовірно, обумовлено транслокацією
невеликої частини хромосомного матеріалу, що містить фактор, що визначає яєчко,
An abdominal CT scan may reveal abnormalities in Turner syndrome (streak до Х-хромосоми.
ovaries) or androgen insensitivity syndrome, and it could reveal atrophied testes
in Klinefelter syndrome. However, karyotyping is more accurate for determining
this syndrome. КТ черевної порожнини може виявити аномалії синдрому Тернера (смуга яєчників)
або синдрому нечутливості до андрогенів, і це може виявити атрофовані яєчка при
Ciliary movement may be useful in diagnosing Kartegener syndrome, but the синдромі Клайнфельтера. Однак каріотипування є більш точним для визначення
syndrome does not cause abnormal testosterone and LH levels, which are seen цього синдрому.
in this patient. Defective migration of GnRH cells and anosmia are classic
symptoms of Kallman syndrome, which this man does not appear to have.
Циліарне рух може бути корисним при діагностиці синдрому Картегенера, але
Sperm counts are commonly done in suspected male infertility, but the blood синдром не викликає аномальних рівнів тестостерону і ЛГ, які спостерігаються у
test findings point towards a diagnosis of Klinefelter, which is best diagnosed даного пацієнта. Дефектна міграція клітин GnRH і аносмії - це класичні симптоми
with karyotyping. синдрому Каллмана, який, здається, цей чоловік не має.

The smell test is used to diagnosis anosmia, which is a characteristic of


Kallmann syndrome. Testosterone levels are low in these patients, but levels of Підрахунок сперми зазвичай робиться при підозрі на чоловіче безпліддя, але
gonadotropin-releasing hormones are also low (vs this patient’s high LH level). результати дослідження крові вказують на діагноз Клайнфельтера, який найкраще
діагностується каріотипуванням.
Тест на запах використовується для діагностики аносмії, що є характерним для
синдрому Каллмана. Рівень тестостерону у цих пацієнтів низький, але рівні
гонадотропін-рилізинг гормонів також низькі (у порівнянні з високим рівнем ЛГ у
даного пацієнта).
A 16-year-old male presents to the clinic with a testicular mass. On examination, 16-річний чоловік подається в клініку з масами яєчка. При обстеженні виявлено, що
it is found that the mass does not transilluminate. Blood work shows a serum α- маса не просвічує. Зразки крові показують α-фетопротеїн сироватки 132 мкг / мл
fetoprotein of 132 µg/mL (normal: <10 µg/mL). A radical orchiectomy is (нормальний: <10 мкг / мл). Проводиться радикальна орхіектомія, яка показує велику
performed, which grossly shows a large mass that is well demarcated, масу, яка добре розмежована, слизова і жовта.
mucinous, and yellow.
Which of the following is the most likely diagnosis? Який з наступних діагнозів є найбільш вірогідним?

A.Choriocarcinoma Хоріокарцинома

Чиста ембріональна карцинома


B.Pure embryonal carcinoma
Семінома
C.Seminoma
Пухлина жовткового мішка
Показати пояснення
D.Yolk sac tumor Правильно!
Show Explanations Цей пацієнт має масу тестикули, яка не просвічує пухлина, що свідчить про пухлину.
Correct! Разом зі своїм підвищеним рівнем сироваткового a-фетопротеїну (AFP), ці дані
This patient presents with a testicular mass that does not transilluminate, свідчать про пухлину жовткового мішка (також називається пухлиною
suggesting a tumor. Together with his elevated serum a-fetoprotein (AFP) level, ендодермальної синуси). Ці пухлини мають найвищий рівень захворюваності у
these findings suggest a yolk sac tumor (also referred to as an endodermal дитинстві та ранньому дитинстві. Чисті пухлини жовткового мішка рідкісні у дорослих.
sinus tumor). These tumors have a peak incidence in infancy and early
childhood. Pure yolk sac tumors are rare in adults.
Вважається, що пухлини жовткового мішка є похідними від злоякісних клітин
Yolk sac tumors are thought to be derived from malignant endodermal cells that ендодерми, які секретують AFP, які можуть бути виявлені в сироватці. Симптоми
secrete AFP, which can be detected in serum. Symptoms are similar to other подібні до інших типів раку яєчка і включають нетранслюмінуючу групу або набряк
types of testicular cancer and include a non-transilluminating lump or swelling in яєчка, що супроводжується болем. Гістологічно пухлини жовткового мішка показують
the testicle accompanied by pain. Histologically, yolk sac tumors would show тіла Шіллера-Дюваля, які нагадують примітивні клубочки з еозинофільними
Schiller-Duval bodies, which resemble primitive glomeruli with eosinophilic глобулами.
globules.

Both choriocarcinoma and pure embryonal carcinoma would demonstrate І хоріокарцинома, і чиста ембріональна карцинома демонструють підвищений рівень
elevated levels of human chorionic gonadotropin (hCG) rather than a- людського хоріонічного гонадотропіну (hCG), а не а-фетопротеїну. Семінома не
fetoprotein. A seminoma would not demonstrate elevated levels of a- продемонструвала б підвищених рівнів а-фетопротеїну, а на грубому обстеженні
fetoprotein, and on gross exam would appear gray or white, not yellow and виявилася б сіра або біла, а не жовта і слизова.
mucinous.
A 48-year-old woman (gravida 2, para 1) presents to her gynecologist with a 3- 48-річна жінка (gravida 2, пункт 1) представляє своєму гінекологу 3-місячний анамнез
month history of ″vague″ abdominal pain with no significant change in her bowel «туманного» болю в животі без суттєвих змін у звичках кишечника. За цей час вона
habits. She has unintentionally lost 4.5 kg (10 lb) of body weight during this time. ненавмисно втратила 4,5 кг маси тіла. Вона любить обідати зі своєю сім'єю, але має
She enjoys dining with her family but has had some difficulty with lower певні труднощі з нижчим тиском у животі. При медичному огляді відзначається
abdominal pressure. On physical examination, bilateral adnexal fullness is двостороння придаткова повнота. Рентген грудної клітки нормальний. КТ черевної
noted. Chest x-ray is normal. CT scan of the abdomen shows bilateral ovarian порожнини показує двосторонні маси яєчників і потовщення стінки шлунка.
masses and stomach wall thickening. Pathologic examination of surgically Патологічне обстеження хірургічно висічених мас яєчників показує круглі клітини, що
excised ovarian masses shows round, mucin-secreting cells as seen in the секретують муцин, як видно на зображенні.
image.
На додаток до патологічного дослідження маси яєчників цього пацієнта, докази того,
які з наступних симптомів, швидше за все, допоможуть діагностувати цього пацієнта?

Галакторея

Гематохезія

Пальпується жовчний міхур

Перлинні папули на обличчі

Надклавікулярна лімфаденопатія
Показати пояснення
Правильно!
Пацієнтка має ознаки болю в животі, втрати ваги і придаткової повноти. У її черевній
КТ показані двосторонні маси яєчників і потовщення стінки шлунка. Результати
біопсії яєчникових мас показують круглі, секретують муцину клітини, відомі як клітини
«перстн-кільця» (показано на зображенні питання). Пухлини Крукенберга є раковими
метастазами в яєчники, які складаються з цих клітин і пояснюють двосторонні маси
пацієнта. У більшості випадків (до 70%) первинною ділянкою пухлини Крукенберга є
шлунок. Ці результати на додаток до абдомінальних симптомів пацієнта і втрати ваги
припускають рак шлунка як діагноз.

In addition to pathologic examination of this patient’s ovarian masses, evidence


Рак шлунка часто є аденокарциномою, яка агресивно поширюється на лімфатичні
of which of the following symptoms is most likely to help diagnose this patient?
вузли і печінку. Класичним ознакою метастатичного раку шлунка є залучення лівого
надключичного лімфатичного вузла, який називається вузлом Вірхова, в якому
A.Galactorrhea виникають надключичні лімфаденопатії. Залучення відбувається з лівого боку, тому
що грудний канал зливає всі структури зліва в грудної порожнини і всі структури під
діафрагмою з обох сторін.
B.Hematochezia
Галакторея пов'язана з підвищеними рівнями пролактину, що є вторинними по
відношенню до пролактиномів передньої долі гіпофіза, які рідко метастазують.
C.Palpable gallbladder
Гематохезія часто є раннім ознакою колоректального раку, який зазвичай метастазує
в печінку. Пальпируемий, непухлинний жовчний міхур пов'язаний з дистальною
D.Pearly papules on the face обструкцією загальної жовчної протоки, що є вторинною по відношенню до
аденокарциноми підшлункової залози. Перлинні папули на ділянках, що знаходяться
E.Supraclavicular lymphadenopathy на сонці, такі як обличчя і руки, свідчать про базально-клітинний рак, який майже
Show Explanations ніколи не метастазує. Жоден з них не пов'язаний з круглими, секретуючими муцином
Correct! клітинами.
The patient presents with signs of abdominal pain, weight loss, and adnexal
fullness. Her abdominal CT scan shows bilateral ovarian masses and stomach
wall thickening. Biopsy results of the ovarian masses show round, mucin-
secreting cells known as “signet-ring” cells (shown in the question
image). Krukenberg tumors are cancer metastases to the ovaries that are
composed of these cells and account for the patient’s bilateral masses. In most
cases (up to 70%), the primary site of Krukenberg tumors is in the stomach.
These results in addition to the patient’s abdominal symptoms and weight loss
suggest gastric cancer as the diagnosis.

Gastric cancer is often an adenocarcinoma that can spread aggressively to


lymph nodes and the liver. A classic sign of metastatic gastric cancer is
involvement of the left supraclavicular lymph node called Virchow node in
which supraclavicular lymphadenopathy results. Involvement is on the left
side because the thoracic duct drains all structures on the left in the thoracic
cavity and all structures below the diaphragm on both sides.

Galactorrhea is associated with elevated prolactin levels secondary to


prolactinomas in the anterior pituitary that rarely metastasize. Hematochezia is
often an early sign of colorectal carcinoma which usually metastasizes to the
liver. A palpable, nontender gallbladder is associated with distal common bile
duct obstruction secondary to pancreatic adenocarcinoma.Pearly papules on
sun-exposed areas, such as the face and arms, are indicative of basal cell
carcinoma, which almost never metastasizes. None of these are associated with
round, mucin-secreting cells.
A G2P2 woman has given birth prematurely to her second child. She maintained Жінка G2P2 народила передчасно народженої дитини. Вона підтримувала здорову
a healthy diet and lifestyle before and throughout her pregnancy. She does not дієту і спосіб життя до і протягом усієї вагітності. Вона не має істотної історії хвороби
have any significant past medical history, and denies use of tobacco, alcohol, і заперечує використання тютюну, алкоголю та незаконних наркотиків. Перша
and illicit drugs. Her first pregnancy was without complications. Her vital signs вагітність була без ускладнень. Її життєво важливі ознаки включають: температуру
include: temperature 98.6°F (37°C) and blood pressure 120/80 mm Hg. On (37 ° C) і артеріальний тиск 120/80 мм рт. При фізичному огляді у новонародженого є
physical examination, the neonate has gross hepatosplenomegaly and ascites, груба гепатоспленомегалія і асцит, а також периферичні набряки і пурпура.
as well as peripheral edema and purpura. The placenta is pale, thickened, and Плацента бліда, потовщена і збільшена. Скринінг антитіл для препаратів
enlarged. Antibody screening for cytomegalovirus and toxoplasmosis agents are цитомегаловірусу і токсоплазмозу є негативними. Дитина помирає незабаром після
negative. The child dies soon after birth. народження.
What is most likely responsible for the outcome of this pregnancy?
Що, ймовірно, відповідає за результат цієї вагітності?
A.Failure of neural tube to close Невдача закриття нервової трубки

Збільшення навколоплідних вод


B.Increased amniotic fluid
Конвульсії матері з гіпертензією
C.Maternal convulsions with hypertension
Передчасні скорочення
D.Premature contractions Резус-несумісність

E.Rh incompatibility Удушення пуповини


Показати пояснення
Правильно!
F.Umbilical cord strangulation У новонародженого у віньєтці були грубий гепатоспленомегалія, асцит і периферичні
Show Explanations набряки разом з пурпурою. Ці ознаки, крім блідої, товстої і збільшеної плаценти,
Correct! підтримують діагностику водянки фетальних. Це є потенційним наслідком важкої
The neonate in the vignette had gross hepatosplenomegaly, ascites, and гемолітичної хвороби новонародженого. Hydrops fetalis відбувається, коли мати
peripheral edema, along with purpura. These characteristics, in addition to a дитини є резус-негативною, але сенсибілізується раніше проведеної Rh-позитивною
pale, thick, and enlarged placenta, support the diagnosis of hydrops дитиною. Якщо її наступна дитина є резус-позитивною, і вона не лікується
fetalis. This is a potential consequence of severe hemolytic disease of the імуноглобуліном проти резус-антигену, її друга вагітність піддається ризику для
newborn. Hydrops fetalis occurs when the mother of the child is Rh negative, фетальної водянки через резус-несумісність. Перша вагітність зазвичай не
but is sensitized by a previously carried Rh-positive child. If her next child is Rh ускладнена.
positive, and she is not treated with immunoglobulin against the Rh antigen, her У водянках плоду, материнський IgG проти Rh перетинає плацентарний бар'єр і
second pregnancy is at risk for hydrops fetalis due to Rh incompatibility. The нападає на ембріональні еритроцити з Rh-маркером, що призводить до симптомів,
first pregnancy is usually uncomplicated. що спостерігаються у цієї дитини. Непряма анемія плода і набряк від гемолітичної
In hydrops fetalis, maternal IgG against Rh crosses the placental barrier and хвороби новонародженого призводить до появи блідої, потовщеної і збільшеної
attacks fetal RBCs with the Rh marker, leading to the symptoms seen in this плаценти. Ці немовлята піддаються екстрамедулярному кровотворення,
child. The consequential fetal anemia and edema from hemolytic disease of the намагаючись не відставати від втрати клітин крові. Hydrops fetalis має поганий
newborn results in a pale, thickened, and enlarged placenta. These infants прогноз, причому смерть зазвичай відбувається незабаром після або до пологів. У
undergo extramedullary hematopoiesis in an attempt to keep up with the loss of новонароджених, які хворіють при народженні або поставлені передчасно, існує ще
blood cells. Hydrops fetalis has a poor prognosis, with death usually occurring більший ризик смерті.
shortly after or before delivery. Neonates who are ill at birth or delivered
prematurely have an even higher risk of death. Невдача закриття нервової трубки може викликати аненцефалію, але симптоми,
Failure of neural tube to close can cause anencephaly, but the symptoms представлені новонародженим у цьому випадку, призводять більше до водям плоду.
presented by the neonate in this case lead more toward hydrops Материнські судоми при гіпертонії пов'язані з еклампсією. Еклампсія може викликати
fetalis. Maternal convulsions with hypertensionare associated with смерть плода. Однак мама здається здоровою з нормальним артеріальним тиском
eclampsia. Eclampsia can cause fetal death. However, the mother appears to 120/80 мм рт.ст. і не має значних медичних проблем. Підвищена амніотична рідина
be healthy with a normal blood pressure of 120/80 mm Hg and does not have характерна для багатоводдя, що не є наслідком водянки плоду. Передчасні
any significant medical concerns. Increased amniotic fluid is characteristic of скорочення можуть викликати передчасні пологи, але величезна кількість
polyhydramnios, which is not an implication in hydrops fetalis. Premature представлених симптомів не призводить до передчасних скорочень. Ущільнення
contractions can cause premature delivery, but the vast number of symptoms пуповини може призвести до загибелі плода, але знову ж таки, симптоми
presented does not lead to premature contractions. Umbilical cord новонародженого вказують на діагноз фетальних водянки.
strangulation can result in fetal death, but again, the symptoms of the neonate
point more to a hydrops fetalis diagnosis.

A 31-year-old man presents for his annual physical and reports noticing a 31-річний чоловік подає на своєму щорічному фізичному звіту і повідомляє про те,
nontender “lump" in his right testicle a few weeks earlier. He denies що він не знає "ущільнення" у своєму правому яєчку кілька тижнів тому. Він
gynecomastia, changes in his voice, or recent trauma to the testicle. An заперечує гінекомастію, зміни голосу або нещодавню травму яєчка. лікар виявляє
ultrasound completed in the office reveals a nontender, homogenous right неплідність, однорідне розширення правого яєчка, лікар сканує КТ черевної
testicular enlargement. The physician orders an abdominal CT scan, which порожнини, яка виявляє збільшені, правобічні парааортальні лімфатичні вузли.
reveals enlarged, right-sided para-aortic lymph nodes.
What testicular pathology does this patient most likely have? Яку патологію яєчка найчастіше має цей пацієнт?

A.Embryonal carcinoma Ембріональна карцинома

Епідидиміт
B.Epididymitis
Клітинна пухлина Лейдіга
C.Leydig cell tumor
Семінома
D.Seminoma Перекрут яєчка апендикса

E.Torsion of the appendix testis Пухлина жовткового мішка


Показати пояснення
Правильно!
F.Yolk sac tumor Ця молода людина представлена безболісним грудкою яєчок, що не має чіткої
Show Explanations етіології. Взяті в поєднанні з висновками неблагополучного, гомогенного розширення
Correct! правого яєчка на УЗД і збільшених парааортальних лімфатичних вузлів на КТ
This young man presents with a painless testicular lump that has no clear черевної порожнини, диференційний діагноз включає рак яєчка з можливими
etiology. Taken in combination with findings of a nontender, homogenous right метастазами. Найбільш поширеним типом є семіноми, на які припадає приблизно
testicular enlargement on ultrasound and enlarged para-aortic lymph nodes on 40% випадків раку яєчка. Найчастіше семіноми діагностуються у чоловіків у віці від
abdominal CT, the differential diagnosis includes testicular cancer with 25 до 40 років і не зустрічаються в дитинстві. Оскільки яєчка протікають до
possible metastasis. Seminomas are the most common type, accounting for парааортальних лімфатичних вузлів, у парааортальній ланцюга часто
approximately 40% of testicular cancers. Seminomas are most often diagnosed спостерігається лімфатичне поширення раку яєчка. Пацієнти з семіномами зазвичай
in men between the ages of 25 and 40 years and do not occur in infancy. мають підвищений рівень плацентарної лужної фосфатази (пухлинний маркер).
Because the testicles drain to the para-aortic lymph nodes, lymphatic spread of
testicular cancers is often seen in the para-aortic chain. Patients with
seminomas will typically have an elevated level of placental alkaline Епідидиміт і кручення яєчка апендикса обидва будуть присутні з болем і не будуть
phosphatase (tumor marker). викликати парааортальній лімфаденопатії. Пухлини клітин Лейдіга з ознаками
гіперандрогенії, яких немає у даного пацієнта, і пухлини жовткового мішка зазвичай
Epididymitis and torsion of the appendix testis would both present with pain зустрічаються у дітей віком до 4 років. Ембріональна карцинома є ще одним типом
and would not cause para-aortic lymphadenopathy. Leydig cell tumors present пухлини яєчка, яка представляє собою геморагічну масу з некрозом, але «чиста»
with signs of hyperandrogenism, which are not present in this patient, and yolk ембріональна карцинома зустрічається рідко.
sac tumors are usually found in children under the age of 4 years. Embryonal
carcinoma is another type of testicular tumor, which presents as a hemorrhagic
mass with necrosis, but a “pure” embryonal carcinoma is rare.
A 25-year-old woman presents to her physician with an upper respiratory tract 25-річна жінка подарує своєму лікареві інфекцію верхніх дихальних шляхів. Це вже
infection. It is her fifth such infection in the past year. Along with her history of п'ята така інфекція в минулому році. Поряд з історією респіраторних інфекцій, вона
respiratory infections, she reports that she has a very poor sense of smell. She повідомляє, що вона має дуже поганий нюх. Вона пояснює свою хворобу
attributes her illness to increased stress at home, reporting that she and her підвищеним стресом вдома, повідомляючи, що вона та її чоловік намагаються зачати
husband have been trying to conceive unsuccessfully for 13 months. Physical безуспішно протягом 13 місяців. Фізичне обстеження виявляє нормальні S1 і S2
examination reveals normal S1 and S2 heart sounds over the right intercostal серцеві звуки над правим міжребер'єм в середньо-ключичній лінії. Вершина биття
space in the mid-clavicular line. The apex beat is palpable over the right 5th пальпується по правому 5-му міжребер'ї.
intercostal space.
Which organizational unit represents the structure most likely to be defective in Яка організаційна одиниця являє собою структуру, яка, ймовірно, є дефектною у
this patient? даного пацієнта?

A.Collagen α chains in triplets Колагенові α-ланцюги в триплетах

Важкі ланцюги з котушками і стеблами, прикріпленими за годинниковою стрілкою від


B.Heavy chains with coils and stems attaching clockwise from microtubule
пар мікротрубочок до сусідніх пар
pairs toward adjacent pairs
Багатоблочне ядро з заповненими ферментами гранулами
C.Multilobular nucleus with enzyme-filled granules
Численні сусідні молекули E-кадгерину з актиновими нитками
D.Numerous adjoining E-cadherin molecules with actin filaments Дві довші важкі ланцюги і дві коротші легкі ланцюги, пов'язані дисульфідними
зв'язками
E.Two longer heavy chains and two shorter light chains linked by disulfide Показати пояснення
bonds Правильно!
Show Explanations Пацієнт представляє аносмію, субфертильність, анамнез інфекцій верхніх дихальних
Correct! шляхів та ситусарний інверсус (про що свідчить розташування її серцевих звуків). Ці
The patient presents with anosmia, subfertility, a history of upper respiratory симптоми вказують на діагноз синдрому Картагенера.
tract infections, and situs inversus (as indicated by the location of her heart У синдромі Картагенера спостерігається дефект динеїну, що призводить до поганого
sounds). These symptoms point to a diagnosis of Kartagener syndrome. кліренсу слизу. Руки Dynein є важкими ланцюгами з котушками і стеблами, які
In Kartagener syndrome, there is a defect in dynein, leading to poor mucus прикріплюються від пари мікротрубочок в межах аксонеми війок або джгутиків до
clearance. Dynein arms are heavy chains with coils and stems that attach сусідньої пари. Під час інсульту моторний домен рук динеїну зазнає конформаційних
from a microtubule pair within the axoneme of cilia or flagella toward an змін, що призводить до того, що стебло, що зв'язує мікротрубочки, повертається, в
adjacent pair. During a power stroke, the motor domain of the dynein arm результаті чого одна мікротрубочка ковзає щодо іншої. Це призводить до згинання
undergoes a conformational change that causes the microtubule-binding stalk to руху, необхідного для того, щоб вії або джгутики билися і просували клітину. У
pivot, resulting in one microtubule sliding relative to another. This produces the чоловіків цей синдром може призвести до безпліддя внаслідок азооспермії, рухливих
bending movement needed for cilia or flagella to beat and propel the cell. In сперматозоїдів, але і безплідних війок, або незручних сперматозоїдів. У жінок це
men, the syndrome can lead infertility due to azoospermia, motile spermatozoa може призвести до субфертильності внаслідок зменшення функції маткової труби,
but immotile cilia, or immotile spermatozoa. In women, it can cause subfertility що призводить до затримки транзиту яйцеклітини з яєчника в ампулу, що є найбільш
due to decreased fallopian tube function, leading to delay in the transit of the поширеним місцем запліднення. Крім того, в окремих випадках знижена функція
ovum from the ovary to the ampulla, the most common site of fertilization. Also, маткової труби може призвести до позаматкової вагітності. Situs inversus також
in rare cases, the decreased function of the fallopian tube may lead to ectopic пов'язаний з цим розладом.
pregnancy. Situs inversus is also associated with this disorder.
Collagen α chains in triplets describes the structure of collagen fibrils. A Колагенові α-ланцюги в триплетах описують структуру колагенових фібрил. Дефект у
defect in collagen structure results in connective tissue diseases such as scurvy, структурі колагену призводить до захворювань сполучної тканини, таких як цинга,
Ehlers-Danlos syndrome, or osteogenesis imperfecta. Numerous adjoining E- синдром Елерса-Данлоса або недосконалого остеогенезу. Численні сусідні молекули
cadherin molecules with actin filamentsdescribes the zona adherens E-кадгерину з актиновими філаментами описують перехід zona adherens.
junction. Modifications in the expression and function of adherens junction Модифікації експресії та функціонування компонентів adherens junction можуть
components can result in pathologic conditions such as cerebral cavernous призвести до патологічних станів, таких як церебральна кавернозна мальформація
malformation and alterations of vascular morphology seen in та зміни судинної морфології у пухлинах. Багатоблочне ядро з заповненими
tumors. Multilobular nucleus with enzyme-filled granules describes ферментами гранулами описує нейтрофіли. Дефект нейтрофілів може призвести до
neutrophils. A defect in neutrophils can result in conditions such as chronic таких станів, як хронічна гранулематозна хвороба і синдром Чідіака-Хігасі, які
granulomatous disease and Chédiak-Higashi syndrome, both of which affect впливають на імунітет, але не викликають безпліддя. Дві більш довгі важкі ланцюги і
immunity but would not cause infertility. Two longer heavy chains and two дві більш короткі легкі ланцюги, пов'язані дисульфідними зв'язками, описують
shorter light chains linked by disulfide bonds describes antibodies. A defect антитіла. Дефект структури антитіл може впливати на імунітет, але не на
in antibody structure may affect immunity, but not fertility. фертильність.
A 52-year-old white woman who has never given birth comes to the physician 52-річна біла жінка, яка ніколи не народила, приходить до лікаря на щорічний огляд.
for an annual check-up. She has a 10–pack-year smoking history and a family У неї 10-тирічна історія куріння та сімейний анамнез, важливий для ранньої
history significant for early mastectomies in her mother and grandmother. She мастектомії у її матері та бабусі. Кожну ніч вона має дві склянки вина з вечерею і
has two glasses of wine every night with dinner and occasionally drinks three or іноді п'є три або більше порцій алкоголю у вихідні з друзями. Життєві ознаки
more servings of alcohol on the weekends with friends. Vital signs are within знаходяться в межах норми. За останні 5 років вона збільшилася на 14 кг (30 фунтів).
normal limits. She has had a 14 kg (30 lb) weight gain over the past 5 years. Висновки з фізичного обстеження помітні червоною лусковою плямою на правому
Findings on physical examination are notable for a red scaly patch on her right соску і пальпирующим пахвовим вузлам. Пальпація виявляє тверду масу в правому
nipple and palpable axillary nodes. Palpation reveals a firm mass in her right відділі грудей і виділення з правого соска, що складається з крові і сироватки.
breast and discharge from the right nipple, which consists of blood and serum.
Which of the following would most likely be seen on histologic examination of Яке з перерахованих нижче випадків, швидше за все, буде видно при гістологічному
the scaly patch? дослідженні лускатого пластиру?

A.Cells presenting in a linear pattern within breast stroma Клітини, що представляють лінійну структуру в межах грудної строми

Позаклітинні слизу, що оточують скупчення пухлинних клітин


B.Extracellular mucus surrounding clusters of tumor cells
Фіброваскулярні структури вистелені протоковим епітелієм
C.Fibrovascular structures lined by ductal epithelium
Великі клітини з чіткими "ореолами"
D.Large cells with clear "halos" Лімфоцитарна інфільтрація

E.Lymphocytic infiltration Проліферація нормальних епітеліальних клітин

Тверда картина з ділянкою центрального некрозу


F.Proliferation of normal epithelial cells Показати пояснення
Правильно!
52-річний нерождаючий білий пацієнт представляє червону лускату пляму на
G.Solid pattern with an area of central necrosis правому соску з серозносудинним виділенням, пальпируемими пахвовими вузлами і
Show Explanations твердою масою в правій грудях. Це вказує на діагноз хвороби Педжета молочної
Correct! залози. Хвороба грудної залози Педжет складається з екзематозного пластиру на
The 52-year-old nulliparous white patient presents with a red scaly patch on her соску або ареолі, часто з нижньою протоковою карциномою (іпсилатеральна
right nipple with serosanguinous discharge, palpable axillary nodes, and a firm протокова карцинома in situ або інвазивна протокова карцинома), що відчутно при
mass in her right breast. This points to a diagnosis of Paget disease of the обстеженні молочної залози.
breast. Paget disease of the breast consists of an eczematous patch on the
nipple or areola, often with underlying ductal carcinoma (ipsilateral ductal Гістологічне дослідження пластиру виявляє великі клітини з яскраво вираженими
carcinoma in situ or invasive ductal carcinoma) that is palpable on breast ядрами і блідими до ясних ореолів? цитоплазми в епідермісі, як показано на
examination. зображенні. Фактори ризику для раку молочної залози включають нульовий рівень,
Histologic examination of the patch reveals large cells with prominent ранню менархе, пізню менопаузу, ожиріння, дієту з високим вмістом жирів і
nucleoli and pale to clear ?halos? of cytoplasm in the epidermis, as shown in позитивну сімейну історію.
the image. Risk factors for breast cancer include nulliparity, early menarche, late
menopause, obesity, high-fat diet, and a positive family history.

A 50-year-old woman, G5P5, presents for a yearly well-woman examination. 50-річна жінка, G5P5, подарує на щорічне обстеження для жінок-жінок. Її останній
Her last menstrual period was 2 years ago. She has no symptoms but is менструальний період був 2 роки тому. У неї немає симптомів, але вона стурбована
concerned because her grandmother died of endometrial cancer. The patient тим, що її бабуся померла від раку ендометрію. Пацієнт стає сексуально активним у
became sexually active at the age of 13, has a 30–pack-year smoking history, віці 13 років, має історію куріння на 30 пачок і допускає до пиття два пива за ніч.
and admits to drinking two beers per night. She has been taking combination Вона приймала комбіновані таблетки для пероральних контрацептивів для
oral contraceptive pills for symptom relief from hot flashes. Her body mass index полегшення симптомів від припливів. Її індекс маси тіла становить 34 кг / м2.
is 34 kg/m2. Results of a bimanual examination, pelvic examination, and breast Результати бімануального обстеження, тазового обстеження та обстеження
examination are normal. молочної залози є нормальними.

Which of the following is most likely to be the patient’s greatest risk factor for
endometrial cancer? Яке з перерахованих нижче факторів, швидше за все, є найбільшим фактором
ризику для раку ендометрію?

A.Combination hormone replacement therapy


Комбінована гормонозамінна терапія

B.Early menopause Рання менопауза

C.Early sexual activity Рання сексуальна активність

Множинність
D.Multiparity
Ожиріння
Показати пояснення
E.Obesity
Show Explanations Правильно!
Correct! 50-річна жінка подарувала своєму щорічному жінці-екзаменаторові турботу про ризик
A 50-year-old woman presents for her yearly well-woman examination розвитку раку ендометрію, оскільки її бабуся померла від неї. Історія хвороби
concerned about her risk for endometrial cancer because her grandmother died включає статеву активність до 13 років і вживання тютюну та алкоголю. Її індекс маси
from it. The patient’s history includes sexual activity by age 13 and tobacco and тіла (ІМТ) становить 34 кг / м2. З усіх цих висновків, фактор, який представляє
alcohol use. Her body mass index (BMI) is 34 kg/m2. Of all these findings, the найбільший ризик раку ендометрію, є ожирінням. ІМТ 30 кг / м2 або більше є
factor that presents the greatest risk for endometrial cancer is obesity. A BMI of діагностикою ожиріння. Використання тютюну та алкоголю частіше збільшує ризик
30 kg/m2 or greater is diagnostic of obesity. The use of tobacco and alcohol is інших захворювань. Ожиріння (особливо, якщо жінка становить 22,7 кг [50 фунтів]
more likely to increase her risk for other diseases. Obesity (especially when a або більше надмірної ваги) збільшує ризик розвитку раку ендометрію в п'ять разів до
woman is 22.7 kg [50 lb] or more overweight) increases the risk for development 10 разів.
of endometrial cancer fivefold to 10-fold.

The increased risk in obese patients is due to higher levels of estrogen because Підвищений ризик у пацієнтів з ожирінням обумовлений більш високим рівнем
testosterone is converted into estrogen in fat cells, as well as in the ovaries. естрогену, оскільки тестостерон перетворюється в естроген в жирових клітинах, а
Endometrial tissue is estrogen sensitive, and higher levels of circulating також в яєчниках. Ендометріальна тканина чутлива до естрогену, і більш високі рівні
estrogen lead to increased glandular proliferation and increased risk for циркулюючого естрогену призводять до посилення проліферації залоз і підвищеного
malignant transformation. Anything that increases exposure of the ризику злоякісної трансформації. Все, що збільшує вплив ендометрію на
endometrium to unopposed estrogen (without progesterone) will cause незамещенний естроген (без прогестерону), призведе до посилення проліферації
increased endometrial proliferation, thus increasing the risk of endometrial ендометрію, збільшуючи таким чином ризик розвитку раку ендометрію. Естроген у
cancer. The estrogen in combination oral contraceptive pills is mitigated by комбінованих оральних контрацептивах пом'якшується прогестином і тому не є
progestin and is therefore not a risk factor for this patient. Because weight is one фактором ризику для цього пацієнта. Оскільки вага є одним з факторів ризику для
of the modifiable risk factors for endometrial cancer, lowering the BMI in patients раку ендометрію, зниження ІМТ у пацієнтів з ризиком розвитку раку ендометрію є
at risk for developing endometrial cancer is a good method of primary хорошим методом первинної профілактики.
prevention.

Early menopause and multiparity (both of which characterize the patient) Рання менопауза та мультипартія (обидва з яких характеризують пацієнта) фактично
actually decrease, not increase, the risk of endometrial cancer because both зменшують, а не збільшують ризик раку ендометрію, оскільки як рання менопауза,
early menopause and multiparity decrease the amount of time the endometrium так і мультипартія зменшують кількість часу, коли ендометрій піддається впливу
is exposed to unopposed estrogen. (By the same logic, menopause occurring at естрогену. (Згідно тієї ж логіки, менопауза, що виникає у віці або після 53-річного віку,
or after 53 years of age would increase a patient's risk for endometrial cancer збільшить ризик захворювання на ендометрій у пацієнта через збільшення
because of an increased lifetime amount of endometrial proliferation.) проліферації ендометрію протягом життя). у поєднанні з прогестероном.
Combination hormone replacement therapy does not increase the risk of Прогестерон стабілізує ендометрій і запобігає неконтрольованій проліферації. Рання
endometrial cancer because the estrogen is paired with progesterone. сексуальна активність є фактором ризику раку шийки матки, а не раком ендометрію.
Progesterone stabilizes the endometrium and prevents uncontrolled
proliferation. Early sexual activity is a risk factor for cervical cancer, not
endometrial cancer.

A 45-year-old woman comes to her gynecologist for an annual examination. The 45-річна жінка приходить до гінеколога на щорічне обстеження. Пацієнт піклується
patient is concerned about developing cancer after a friend was recently про розвиток раку після того, як недавно був діагностований друг. Вона має два
diagnosed. She has two glasses of wine with dinner and has smoked one pack келихи вина з вечерею і викурює одну пачку сигарет щотижня протягом останніх 27
of cigarettes weekly for the past 27 years. She has had several sexual partners років. У неї було кілька статевих партнерів з 15 років, але ніколи не вагітна. Вона
since age 15 years but has never been pregnant. She began menarche at age почала менархе у віці 10 років і лікувалася від генітальних бородавок 5 років тому .. Її
10 and was treated for genital warts 5 years ago.. Her mother had a breast мама мав груди, видалену 10 років тому, і тітка матері мала рак шийки матки у 20
mass removed 10 years ago and a maternal aunt had cervical cancer in her років. Ліки включають оральні контрацептиви. Результати мазка Pap показують
20s. Medications include oral contraceptives. Results of a Pap smear show нерегулярні плоскоклітинні епітеліальні клітини з розширенням ядра і
irregular squamous epithelial cells with nuclear enlargement and perinuclear перинуклеарними ореолами, що простягаються від базального шару
halos extending from the basal layer of the squamocolumnar junction. squamocolumnar переходу.

Which of the following factors puts this patients at increased risk for developing
cervical cancer? Який з перелічених факторів спричиняє підвищенню ризику розвитку раку шийки
матки?

A.Early menarche
Рання менархе

B.Early onset of sexual activity Раннє початок сексуальної активності

C.Family history Історія сім'ї

Історія генітальних бородавок


D.History of genital warts
Нульпітальність
Показати пояснення
E.Nulliparity
Правильно!
Show Explanations Нерегулярні плоскоклітинні епітеліальні клітини з розширенням ядра і
Correct! перинуклеарними ореолами, що проходять від базального шару плоскоклітинного
Irregular squamous epithelial cells with nuclear enlargement and perinuclear переходу, описують койлоцити, які виникають в результаті інфікування вірусом
halos extending from the basal layer of the squamocolumnar junction is папіломи людини (ВПЛ). Більшість інфекцій ВПЛ є безсимптомними і спонтанно
descriptive of koilocytes that occur as a result of infection by human виникають. Деякі інфекції можуть зберігатися і призводять до бородавок або
papillomavirus (HPV). Most HPV infections are asymptomatic and resolve передракових поразок. Рак шийки матки майже завжди діагностується на додаток до
spontaneously. Some infections can persist and result in warts or precancerous інфекції ВПЛ. ВПЛ типу 16 і 18 в даний час викликають майже 70% випадків раку
lesions. Cervical cancer is almost always diagnosed in addition to HPV infection. шийки матки, і обидві профілактичні вакцини, які в даний час використовуються,
HPV types 16 and 18 currently cause nearly 70% of cervical cancer cases, and захищають від цих типів. Раки шийки матки найчастіше є плоскоклітинними
both preventative vaccines that are currently in use protect against these types. карциномами і виникають внаслідок невпорядкованого епітеліального росту,
Cervical cancers are most often squamous cell carcinomas and arise from класифікованого як цервікальна інтраепітеліальна неоплазія 1, 2 або 3, залежно від
disordered epithelial growth, classified as cervical intraepithelial neoplasia ступеня залучення епітелію. Мазка папілопатії зменшила смертність цих видів раку,
1, 2, or 3, depending on the extent of epithelial involvement. Pap smears have тому що вони ідентифікують диспластичні ураження попередників, перш ніж тканини
reduced the mortality of these cancers because they identify the precursor стають раковими.
dysplastic lesions before the tissues become identifiably cancerous.

Risk factors for developing cervical cancer include early sexual activity, Фактори ризику розвитку раку шийки матки включають ранню сексуальну активність,
multiple sex partners, smoking, and low socioeconomic status. множинні статеві партнери, куріння та низький соціально-економічний статус.
Nulliparity is a risk factor for breast cancer and endometrial cancer. Cervical
cancer is not sensitive to estrogen and thus nulliparity has no impact on cervical Нульпаратність є фактором ризику раку молочної залози та раку ендометрію. Рак
cancer. Early menarche is a risk factor for endometrial cancer and has no шийки матки не чутливий до естрогену і, отже, нульовий рівень не впливає на рак
impact on cervical cancer. Genital warts are caused by types of HPV that are шийки матки. Рання менархе є фактором ризику раку ендометрію і не впливає на рак
not carcinogenic. Family history is a risk factor for many cancers with a genetic шийки матки. Генітальні бородавки викликані типами ВПЛ, які не є канцерогенними.
factor. The primary cause of cervical cancer is HPV, thus having a family history Сімейний анамнез є фактором ризику для багатьох видів раку з генетичним
imparts little or no additional risk. фактором. Основною причиною раку шийки матки є ВПЛ, тому сімейний анамнез
надає невеликий або зовсім відсутній додатковий ризик.
An asymptomatic 30-year-old woman presents to her physician’s office for a Асимптоматична 30-річна жінка подається в офіс свого лікаря для планового
routine health maintenance examination. She notes that she became sexually медичного обстеження. Вона зазначає, що в юному віці вона стала сексуально
active at a young age, has had a total of 15 sexual partners, and does not often активною, у неї було 15 сексуальних партнерів і не часто використовує захист. Також
use protection. She also smokes a pack of cigarettes daily. A Papanicolaou вона щодня палить пачку сигарет. Мазок Papanicolaou (Pap) і лабораторні
(Pap) smear and laboratory findings indicate high-grade dysplasia. результати свідчать про високу ступінь дисплазії.
What is the mechanism by which the responsible agent causes disease?
Який механізм, за допомогою якого відповідальний агент викликає захворювання?
A.Activation of DNA replication
Активація реплікації ДНК
B.Activation of RNA-dependent DNA polymerization
Активація РНК-залежної полімеризації ДНК
C.Continuation of cell cycle after DNA damage Продовження клітинного циклу після пошкодження ДНК

D.Increased blood vessel growth factors Збільшення факторів росту судин крові

Інгібування мітогенної передачі сигналу


E.Inhibition of mitogenic signal transduction Показати пояснення
Show Explanations Правильно!
Correct! Цей безсимптомний пацієнт з історією статевої активності в ранньому віці, нечастому
This asymptomatic patient with a history of sexual activity at an early age, застосуванні захисту та кількома партнерами виявляє високу ступінь дисплазії на
infrequent use of protection, and multiple partners is found to have high-grade мазці Пап та лабораторних дослідженнях. Збудником дисплазії і раку шийки матки є
dysplasia on a Pap smear and laboratory findings. The causative agent of папіломавірус людини (ВПЛ). Чорний квадрат в гістологічному зображенні показує
cervical dysplasia and cancer is the human papillomavirus (HPV). The високоякісне інтраепітеліальне ураження.
black square in the histology image shows a high-grade intraepithelial lesion. Генні продукти HPV Е6 і Е7 знижують регулювання р53 і pRb відповідно, дозволяючи
The E6 and E7 HPV gene products downregulate p53 and pRb, клітці виходити з-під контролю, незважаючи на будь-яке пошкодження клітинної ДНК.
respectively, allowing the cell to cycle out of control, despite any damage Супресори пухлини, такі як p53 і гипофосфорилированний RB, зазвичай інгібують
to cellular DNA. Tumor suppressors, such as p53 and прогресування G1-S; мутації в цих генах призводять до нестримного поділу клітин.
hypophosphorylated RB, normally inhibit G1-to-S progression; mutations in Інтраепітеліальна неоплазія шийки матки (CIN) може розвиватися в три стадії:
these genes result in unrestrained cell division. Cervical intraepithelial neoplasia
(CIN) can progress in three stages: CIN 1 виникає, коли дисплазія включає нижню третину епітелію або менше,
 CIN 1 occurs when dysplasia involves lower third of epithelium or less, ЦІН 2 виникає, коли дисплазія включає нижню третину до нижньої дві третини
 CIN 2 occurs when dysplasia involves lower third to lower two-thirds of епітелію
the epithelium CIN 3 виникає, коли дисплазія проходить вище нижньої дві третини епітелію, якщо
 CIN 3 occurs when dysplasia extends above lower two-thirds of повне залягання тоді можна назвати карциномою in situ.
epithelium, if full thickess involvement then can be called carcinoma in Якщо дисплазія присутній у повному епітелії, то у пацієнта є карцинома in situ.
situ.
 If dysplasia is present in the full epithelium, the patient has carcinoma in
situ. Фактори росту кровоносних судин не націлені на віруси ВПЛ, і онкогени вірусу ВПЛ
безпосередньо не активують реплікацію ДНК. Онкопротеїни, такі як генні продукти
Blood vessel growth factors are not targeted by the HPV virus and the HPV, малоймовірно інгібують мітогенну трансдукцію сигналу, і вірус HPV не впливає
oncogenes from the HPV virus do not directly activate DNA на активацію РНК-залежної полімеризації ДНК.
replication. Oncoproteins such as the HPV gene products are unlikely
to inhibit the mitogenic signal transduction, and the HPV virus does not
affect the activation of the RNA-dependent DNA polymerization.
A healthy 30-year-old woman presents to the physician for a checkup. Her Здорова 30-річна жінка подарує лікареві для обстеження. Її фізичний огляд є
physical examination is unremarkable, and results of laboratory studies all are непримітним, і результати лабораторних досліджень все знаходяться в межах
within normal limits. She expresses concern about inheriting breast cancer норми. Вона висловлює занепокоєння з приводу успадкування раку молочної залози,
because she had a maternal aunt who recently died of the disease at age 42 оскільки у неї була тітка, яка нещодавно померла від хвороби у віці 42 років. Крім
years. Additionally, the patient's mother died of ovarian cancer at 56 years of того, мати пацієнта померла від раку яєчників у 56-річному віці, а її бабуся також
age, and her grandmother also had breast cancer, but she is unsure at what мала рак молочної залози, але вона не впевнена в тому віці. Лікар рекомендує
age. The physician recommends genetic testing. генетичне тестування.

A different mutation on the same chromosome most likely affected in this


patient’s family results in which of the following diseases? Різні мутації на тій самій хромосомі, найімовірніше, впливають на результати цієї
сім'ї, у яких із наступних захворювань?

A.Familial adenomatous polyposis


Сімейний аденоматозний поліпоз

B.Neurofibromatosis type 1 Нейрофіброматоз типу 1

C.Neurofibromatosis type 2 Нейрофіброматоз 2 типу

Рак підшлункової залози


D.Pancreatic cancer
Синдром фон Гіппеля-Ліндау
E.Von Hippel-Lindau syndrome
Пухлина Вільмса
Показати пояснення
F.Wilms tumor Правильно!
Show Explanations Здорова жінка, яка подарує лікареві, турбується про успадкування раку молочної
Correct! залози через сімейну історію раку молочної залози та яєчників. Гени BRCA1 і BRCA2
A healthy woman who presents to the physician is concerned about inheriting сильно пов'язані з раком молочної залози. BRCA є прикладом гена-супресора
breast cancer because of a family history of breast and ovarian cancer. The пухлини. Мутація призводить до втрати функції, і тому обидва алелі повинні бути
BRCA1 and BRCA2 genes are strongly associated with breast cancer. BRCA is втрачені для розвитку раку. Гени BRCA1 і BRCA2 знаходяться на хромосомах 17 і 13
an example of a tumor suppressor gene. A mutation results in loss of function, відповідно.
and thus both alleles must be lost for development of cancer. The BRCA1 and
BRCA2 genes are found on chromosomes 17 and 13, respectively.
Сімейний анамнез цього пацієнта свідчить про мутацію BRCA1, оскільки вона більш
This patient's family history is suggestive of a BRCA1 mutation because it is сильно пов'язана з раком яєчників, ніж BRCA2, хоча пацієнти з мутаціями BRCA2
more strongly associated with ovarian cancer than BRCA2, although patients також мають підвищений ризик розвитку раку яєчників у порівнянні з загальною
with BRCA2 mutations are also at an increased risk for ovarian cancer популяцією. Хромосома 17 також є джерелом мутації гена NF1 в нейрофіброматозі
compared with the general population. Chromosome 17 is also the source of the типу 1, який є аутосомно-домінантним захворюванням, що характеризується
NF1 gene mutation in neurofibromatosis type 1, which is an autosomal плямами кафе-ау-лайт, вузликами Lisch і нейрофібромами. Корисним натяком на
dominant disease characterized by cafe-au-lait spots, Lisch nodules, and згадку про це є слово «Нейрофіброматоз», яке містить 17 літер, тому NF1 виникає
neurofibromas. A useful hint to remember this by is the word внаслідок мутації на хромосомі 17. Оскільки BRCA1 і NF1 знаходяться на хромосомі
“Neurofibromatosis,” which has 17 letters, therefore NF1 occurs due to a 17, нейрофіброматоз типу 1 є правильною відповіддю.
mutation on chromosome 17. Because BRCA1 and NF1 are both found on
chromosome 17, neurofibromatosis type 1 is the correct answer.
Ген APC, який відповідає за утворення сімейного аденоматозного поліпозу,
The APC gene, which is responsible for causing familial adenomatous виявляється на хромосомі 5. Ген NF2, який викликає нейрофіброматоз типу 2,
polyposis, is found on chromosome 5. The NF2 gene, which causes виявляється на хромосомі 22. Гени DCC і DPC причетні до раку товстої кишки і раку
neurofibromatosis type 2, is found on chromosome 22. The genes DCC and підшлункової залози, відповідно. і обидва вони знаходяться на хромосомі 18. Ген
DPC are implicated in colon and pancreatic cancers, respectively, and are VHL призводить до синдрому фон Хіппеля-Ліндау і знаходиться на хромосомі 3. Ген
both found on chromosome 18. The VHL gene results in von Hippel-Lindau WT1, який викликає пухлину Вілмса, знаходиться на хромосомі 11.
syndrome and is found on chromosome 3. The WT1 gene, which causes
Wilms tumor, is found on chromosome 11.

A pediatrician is evaluating a 3-month-old boy born to a 23-year-old mother. At Педіатр оцінює 3-місячного хлопчика, народженого 23-річною матір'ю. При
birth, the boy was significantly below average in size and had a small head and народженні хлопчик був значно нижче середнього за розміром і мав маленьку голову
a cleft lip. On presentation today, the pediatrician notices that the infant's і щілину губи. На сьогоднішній презентації педіатр зауважує, що нігті та нігті
fingernails and toenails have not begun to develop. The infant also has a low немовляти не почали розвиватися. Немовля також має низький носовий міст. При
nasal bridge. A heart murmur is detected on auscultation. The mother reports аускультації виявляється шум серця. Мати повідомляє, що її попереджали про ліки,
she was cautioned about a medication she was taking during pregnancy, which які вона приймала під час вагітності, яка може бути пов'язана з вродженими вадами.
may be associated with birth defects.

Which of the following drugs is most likely responsible for the findings in this Який з перерахованих вище препаратів, швидше за все, відповідає результатам
patient? цього пацієнта?

A.Alcohol Алкоголь

Літій
B.Lithium
Метімазол
C.Methimazole
Фенітоїн

D.Phenytoin Ретиноєва кислота


Показати пояснення
Правильно!
E.Retinoic acid У цього немовляти є численні порушення, включаючи аномалії черепа і риси
Show Explanations обличчя, дефіцит росту, недорозвинені нігті пальців рук і ніг, щілину губ і
Correct! мікроцефалії. Особливості немовляти узгоджуються з фетальним синдромом
This infant has multiple irregularities including abnormalities of the skull and гідантоїну внаслідок використання антіепілептичних препаратів, таких як фенитоин,
facial features, growth deficiency, underdeveloped nails of the fingers and toes, під час вагітності.
cleft lip, and microcephaly. The infant’s features are consistent with fetal
hydantoin syndrome due to maternal use of antiepileptic drugs, such as
phenytoin, during pregnancy. Інші висновки, які іноді спостерігаються при синдромі гідантоїну плода, включають
затримки розвитку та вади розвитку мозку. Жінки з епілепсією, які приймають
Other findings sometimes seen with fetal hydantoin syndrome include протиепілептичні препарати під час вагітності, мають приблизно 5% шансів (удвічі
developmental delays and brain malformations. Women with epilepsy who take більший ризик загальної популяції) дітей з вадами розвитку. Також відомо, що
antiepileptic drugs during pregnancy have about a 5% chance (twice the risk of фенитоин пов'язаний з горизонтальним ністагмом на бічному погляді на
the general population) of having children with malformations. Phenytoin is also терапевтичних рівнях.
known to be associated with horizontal nystagmus on lateral gaze at therapeutic
levels.
Інші варіанти відповіді вплинули на плід різними способами. Фетальний алкогольний
The other answer choices would have affected the fetus in different ways. Fetal синдром характеризується специфічними черепно-лицьовими, скелетними і нейро-
alcohol syndrome is characterized by specific craniofacial, skeletal, and поведінковими симптомами. Літій пов'язаний з аномалією серця Ебштейна, в якій
neurobehavioral symptoms. Lithium is linked to Ebstein anomaly of the heart in листки трикуспідального клапана зміщуються до верхівки правого шлуночка, що
which the leaflets of the tricuspid valve are displaced toward the apex of the призводить до «атріалізації» правого шлуночка. Ретиноєва кислота призводить до
right ventricle, leading to “atrialization” of the right ventricle. Retinoic acid leads високих показників спонтанного аборту, а також до великих дефектів, включаючи
to high rates of spontaneous abortion, as well as major defects including мікрогнатію, розщілини неба, дефекти тимусу та вади центральної нервової системи.
micrognathia, cleft palate, thymic defects, and central nervous system Застосування метимазолу під час вагітності пов'язане з aplasia cutis congenita,
malformations. Methimazole use during pregnancy is associated with aplasia порушенням якої є відсутність шкіри у дітей раннього віку.
cutis congenita, a disorder in which there is an absence of skin in infants.
A 32-year-old pregnant woman comes to the emergency department because of 32-річна вагітна жінка приходить до відділення невідкладної допомоги через
vaginal bleeding. She reports that her last menstrual period was 24 weeks ago вагінальні кровотечі. Вона повідомляє, що її останній менструальний період був 24
and that she has been receiving routine prenatal care. She denies any pain or тижні тому, і що вона отримувала звичайний допологовий догляд. Вона заперечує
recent trauma. The patient’s temperature is 99.2°F (37.3°C), blood pressure is будь-який біль або недавню травму. Температура пацієнта становить 37,3 ° C,
100/59 mm Hg, and pulse is 102/min. A transabdominal ultrasound is performed артеріальний тиск становить 100/59 мм рт.ст., а пульс - 102 / хв. Проведено
and confirms the presence of a gestational sac and an intrauterine fetal трансабдомінальне ультразвукове дослідження, що підтверджує наявність
heartbeat. Transvaginal ultrasound is ordered and reveals a placenta that is гестаційного мішка і внутрішньоутробного серцебиття плода. Трансвагінальне
encroaching on the cervical os. ультразвукове дослідження упорядковано і виявляє плаценту, яка зазіхає на шийний
Discovery of which of the following factors in the patient's medical history would осі.
most likely increase the risk for her current complication?
Відкриття якого з наступних факторів в історії хвороби пацієнта, швидше за все,
збільшить ризик для її поточного ускладнення?
A.History of endometriosis

Історія ендометріозу
B.History of pelvic inflammatory disease
Історія запалення тазових органів
C.Prior cesarean section delivery Попередня операція кесаревого розтину

D.Prior ectopic pregnancy Перед позаматковою вагітністю

Використання допоміжних репродуктивних технологій


E.Use of assisted reproductive technologies Показати пояснення
Show Explanations Правильно!
Correct! Цей пацієнт з вагінальним кровотечею і плацентою, що зазіхає на шийний осі,
This patient presenting with vaginal bleeding and a placenta encroaching over швидше за все, страждає від передлежання плаценти або прикріплення плаценти до
the cervical os is most likely suffering from placenta previa, or the attachment нижнього сегмента матки. Передлежання плаценти зазвичай проявляється
of the placenta to the lower uterine segment. Placenta previa typically manifests безболісним вагінальним кровотечею після 20 тижнів вагітності. Фактори ризику
with painless vaginal bleeding after 20 weeks of gestation. Risk factors include включають історію попереднього кесаревого розтину, збільшення кількості
history of a prior cesarean section delivery, increased number of pregnancies, вагітностей, подвійний термін вагітності та історію вискоблювання. З іншого боку,
twin gestation, and history of curettage. Ectopic pregnancy, on the other hand, позаматкова вагітність найчастіше зустрічається в першому триместрі і
most commonly occurs in the first trimester and is characterized by vaginal характеризується вагінальними кровотечами та / або болем у животі. Ця вагітність
bleeding and/or abdominal pain. This pregnancy also has evidence of a також має ознаки гестаційного мішка і внутрішньоматкового серцебиття.
gestational sac and intrauterine heartbeat. Історія ендометріозу призводить до підвищення ризику безпліддя і епітеліального
History of endometriosis leads to increased risk of infertility and epithelial раку яєчників. Історія запальних захворювань тазових органів (ПІД) призводить до
ovarian cancer. History of pelvic inflammatory disease (PID) leads to an підвищеного ризику позаматкової вагітності і часто асоціюється з безпліддям.
increased risk of ectopic pregnancy and is often associated with infertility. Prior Попередня позаматкова вагітність призводить до підвищеного ризику майбутніх
ectopic pregnancy leads to an increased risk for future ectopic позаматкових вагітностей. Використання допоміжних репродуктивних технологій
pregnancies. Use of assisted reproductive technologies leads to an призводить до підвищеного ризику багаторазових вагітностей.
increased risk for multiple gestations.

A 28-year-old woman, G2P1, at 34 weeks’ gestation presents to the emergency 28-річна жінка, G2P1, у 34-тижневій вагітності подається до відділення невідкладної
department saying that she is experiencing severe vaginal bleeding. She допомоги, що вона відчуває сильну вагінальну кровотечу. Вона описує проходження
describes the passage of enough bright red blood to thoroughly soak through a достатньо яскраво-червоної крові, щоб ретельно просочитися через кілька
few sanitary pads over the past few hours. She denies vaginal or abdominal санітарних прокладок за останні кілька годин. Вона заперечує вагінальну або
trauma, passage of other fluids, uterine contractions, or any pain. She denies абдомінальну травму, проходження інших рідин, скорочення матки або будь-який
bleeding or complications during her previous pregnancy. She has no significant біль. Вона заперечує кровотечу або ускладнення під час попередньої вагітності.
history of medical problems or surgeries. She denies use of illicit drugs, tobacco, Вона не має істотної історії медичних проблем або операцій. Вона заперечує
or alcohol. Vital signs include: temperature, 98.6°F (37°C); blood pressure, використання незаконних наркотиків, тютюну або алкоголю. Життєві ознаки
135/80 mm Hg; pulse, 90; and respiratory rate, 14. Her abdomen is gravid, soft, включають: температуру, 98,6 ° F (37 ° C); артеріальний тиск, 135/80 мм рт. пульс,
and nontender. Bimanual and speculum examinations are both deferred. She 90; і частота дихання, 14. Її живіт - тяжкий, м'який і непухлинний. Обидві екзамени з
becomes more comforted when her physician tells her that the fetal heart rate is бімануалу та зразка відкладені. Вона стає більш втішною, коли її лікар говорить їй,
normal at about 130–140 beats/min. An ultrasound is scheduled to be що частота серцевих скорочень у плода становить близько 130–140 уд / хв.
performed. Планується виконати ультразвукове дослідження.
Without knowing the ultrasound results, what is the most likely diagnosis?
Не знаючи результатів ультразвуку, який найбільш вірогідний діагноз?
A.Abruptio placentae
Плацента Abruptio
B.Placenta accreta Акрета плаценти

C.Placenta previa Передлежання плаценти

Розрив матки
D.Uterine rupture
Ваза передлежання
E.Vasa previa Показати пояснення
Show Explanations Правильно!
Correct! Кров'яне кровотеча в половому відділі відноситься до значної вагінальної кровотечі
Antepartum hemorrhage refers to significant vaginal bleeding after 20 weeks’ після 20-тижневої вагітності, яка не пов'язана з пологами. Двома основними
gestation that is unrelated to labor and delivery. The two major causes of third- причинами кровотечі третього триместру є передлежання плаценти і плацента
trimester bleeding are placenta previa and abruptio placentae; uterine rupture abruptio; розрив матки і вазове передлежання рідше викликаються. Передлежання
and vasa previa are rarer causes. Placenta previa occurs when the placenta плаценти виникає, коли плацента перекриває внутрішній шийний ос і може бути
overlies the internal cervical os and can be further described as complete описана як повне передлежання плаценти, часткове передлежання плаценти,
placenta previa, partial placenta previa, marginal placenta previa, or low-lying крайове передлежання плаценти або низько лежачу плаценту, залежно від точного
placenta, depending on the exact relationship to the os. Placenta previa відношення до ос. Передлежання плаценти, як правило, представляє безболісне
generally presents with painless bleeding, making this the most likely diagnosis. кровотеча, що робить це найбільш вірогідним діагнозом. Оскільки це перший епізод
Because this is the first episode of bleeding, this patient can be treated кровотечі, цей пацієнт можна лікувати консервативно, з метою уповільнення пологів і
conservatively, with the goal of delaying delivery and maximizing fetal maturity. максимізації зрілості плода. Вона, ймовірно, спостерігатиметься під час
She will likely be observed during prescribed bed rest unless another bleeding призначеного постільного режиму, якщо не виникне інший епізод кровотечі, при
episode occurs, at which time delivery may be necessary. якому може знадобитися час доставки.
Abruptio placentae refers to the premature separation of the placenta from the
uterus, despite its implantation in a normal location. It is also associated with Abruptio плацента відноситься до передчасного відділення плаценти від матки,
painful antepartum bleeding. The distinction between abruptio placentae and незважаючи на її імплантацію в нормальному місці. Це також пов'язано з болючою
placenta previa is classically made based on the presence (abruptio placentae) передпорожньою кровотечею. Відмінність між abruptio placentae і передлежанням
or absence (placenta previa) of pain. The fetus may also show signs of distress плаценти класично зроблено на основі присутності (abruptio placentae) або
when abruptio placentae has occurred; no signs of fetal distress have been відсутності (передлежання плаценти) болю. У плода також можуть з'являтися ознаки
detected in this patient. дистрессу, коли відбувається абсурдіоплацента; у цього пацієнта не було виявлено
Uterine rupture and vasa previa are rarer causes of vaginal bleeding in жодних ознак дистрес у плоді.
pregnancy. A patient with uterine rupture will normally present during delivery
and have a history of cesarean delivery. On physical examination, fetal parts will Розрив матки і вазове передлежання рідше виникають при вагінальних кровотечах
commonly be palpated. In a patient with vasa previa, vessels transverse the під час вагітності. Пацієнт з розривом матки, як правило, присутній під час пологів і
membrane over the the internal opening of the uterus. Vasa previa can lead to має історію кесаревого розтину. При медичному огляді часто промацуються плодові
exsanguination and fetal death. In this patient, the fetus has a normal heart rate. частини. У пацієнта з вазі-передлежанням судини поперечно мембрани над
внутрішнім отвором матки. Ваза передлежання може призвести до знекровлення і
In a patient with placenta accreta, the placental villi attach directly to the смерті плода. У даного пацієнта плід має нормальну частоту серцевих скорочень.
myometrium due to a defect in the decidua basalis layer, which leads to
incomplete separation of the placenta after delivery. It can cause severe У пацієнта з аккретом плаценти ворсинки плаценти прикріплюються безпосередньо
postpartum hemorrhage, rather than antepartum hemorrhage as seen in this до міометрію внаслідок дефекту шару децидуальної основи, що призводить до
patient. неповного відділення плаценти після пологів. Це може викликати важкі післяпологові
кровотечі, а не кровотеча передпорожніни, як це спостерігається у даного пацієнта.
A 21-year-old African-American primigravid woman at 30 weeks’ gestation is 21-річна афро-американська первобережна жінка на терміні вагітності 30 тижнів
evaluated for hypertension that started after week 25 of pregnancy. She denies оцінюється на предмет гіпертонії, яка почалася після 25-го тижня вагітності. Вона
nausea, vomiting, or abdominal pain. She follows her recommended diet closely заперечує нудоту, блювоту або біль у животі. Вона уважно стежить за
and takes prenatal vitamins. Review of medical records show a blood pressure рекомендованою дієтою і приймає вітаміни для вагітних. Огляд медичних записів
reading of 130/80 mm Hg at 20 weeks gestation. Her current serial blood показує показник артеріального тиску 130/80 мм рт.ст. у терміні вагітності 20 тижнів.
pressure checks include four sitting manual pressures 15 minutes apart, all Її поточні перевірки серійного кров'яного тиску включають чотири тиску ручного тиску
yielding pressures >150/110 mm Hg. A dipstick urine test shows 4+ proteinuria. 15 хвилин один від одного, всі тиск> 150/110 мм рт. Аналізатор зразків сечі показує
Her serum glutamic oxaloacetic transaminase level is also elevated to 2300 4+ протеїнурію. Її рівень глутамінової оксалоацетатної трансамінази в сироватці
mIU/mL (normal <35 mIU/mL). She denies a history of hypertension before the також підвищений до 2300 мМЕ / мл (нормальний рівень <35 мМЕ / мл). Вона
pregnancy and states that her face has become “puffier” than normal since her заперечує анамнез гіпертонії перед вагітністю і стверджує, що її обличчя стало
blood pressure has increased. «посилювати», ніж зазвичай, оскільки її кров'яний тиск збільшився.
Which of the following laboratory changes is most closely associated with a
complication of her condition? Які з наступних лабораторних змін найбільш тісно пов'язані з ускладненням її стану?

A.Decreased creatinine Знизився креатинін

Підвищені D-димери
B.Increased D-dimers
Підвищений рівень глюкози в крові
C.Increased blood glucose levels
Підвищений гаптоглобін
D.Increased haptoglobin Тромбоцитоз
Показати пояснення
E.Thrombocytosis Правильно!
Show Explanations Пацієнтка має гіпертонію, що почалася після 25-го тижня вагітності. У неї не було
Correct! гіпертонії до вагітності, що свідчить про діагностику гестаційної гіпертензії. Однак
The patient presents with hypertension that started after week 25 of pregnancy. наявність протеїнурії крім гіпертензії відповідає прееклампсії. Хоча підвищений
She did not have hypertension prior to the pregnancy, which suggests a рівень глютамінової оксалоацетатної трансамінази в сироватці в цьому випадку
diagnosis of gestational hypertension. However, the presence of proteinuria in може свідчити про синдром HELLP, трансамініт також можна спостерігати при
addition to hypertension is consistent with preeclampsia. Although the elevated прееклампсії внаслідок зниженого печінкового кровотоку при гіпертонії. Крім того,
serum glutamic oxaloacetic transaminase level in this case may suggest HELLP фактори ризику для прееклампсії включають нульовий рівень, афроамериканську
syndrome, transaminitis can also be seen in preeclampsia due to decreased етнічну приналежність, екстремальний вік (<20 або> 35 років), множинність
hepatic blood flow secondary to hypertension. In addition, risk factors for вагітності, молярну вагітність, захворювання нирок (через СКВ або цукровий діабет 1
preeclampsia include nulliparity, African American ethnicity, extremes of age типу), сімейний анамнез прееклампсії і хронічна гіпертензія.
(<20 or >35 years), multiple gestation, molar pregnancy, renal disease (due to
SLE or type 1 diabetes mellitus), a family history of preeclampsia, and chronic Прееклампсія зазвичай відбувається через 20 тижнів гестації і проявляється при
hypertension. гіпертонії, набряках і протеїнурії. Інші ознаки і симптоми можуть включати головний
Preeclampsia usually occurs after 20 weeks of gestation and manifests with біль, погіршення зору, біль у животі, зміну ментації та гіперрефлексию. Фактори
hypertension, edema, and proteinuria. Other signs and symptoms can include ризику для прееклампсії та еклампсії включають попередню гіпертензію, діабет,
headache, blurred vision, abdominal pain, altered mentation, and hyperreflexia. хронічне захворювання нирок і аутоімунні порушення. Остаточне лікування є
Risk factors for preeclampsia and eclampsia include pre-existing hypertension, доставкою. Серйозним ускладненням прееклампсії є дисемінована
diabetes, chronic renal disease, and autoimmune disorders. Definitive treatment внутрішньосудинна коагуляція (DIC), підтверджена і контрольована продуктами
is delivery. A major complication of preeclampsia is disseminated розщеплення фібрином (наприклад, D-димерами), протромбіновим часом, частковим
intravascular coagulation (DIC), confirmed and monitored by fibrin split тромбопластиновим часом і кількістю тромбоцитів. Іншими ускладненнями
products (eg, D-dimers), prothrombin time, partial thromboplastin time, and прееклампсії / еклампсії є гостра жирова печінка, гострий канальцевий некроз і
platelet count. Other complications of preeclampsia/eclampsia are acute fatty HELLP (гемоліз, ферменти підвищеної печінки, низький рівень тромбоцитів).
liver, acute tubular necrosis, and HELLP (Hemolysis, Elevated Liver enzymes,
Low Platelet count) syndrome. Зменшення креатиніну зазвичай не пов'язане з прееклампсією, однак для
Decreased creatinine is not commonly associated with preeclampsia, however діагностики може бути корисним співвідношення білка до креатиніну 3,0 або вище.
a protein to creatinine ratio of 3.0 or higher can be useful for diagnosis. Прееклампсія має підвищену частоту у пацієнтів з вже існуючими підвищеними
Preeclampsia has an increased incidence in patients with pre-existing elevated рівнями глюкози, що спостерігаються при цукровому діабеті. Прееклампсія може в
glucose levels as seen in diabetes. Preeclampsia can eventually lead to HELLP кінцевому підсумку привести до синдрому HELLP (гемолізу, ферментів підвищеної
(Hemolysis, Elevated Liver enzymes, Low Platelet count) syndrome. Hemolysis печінки, низького числа тромбоцитів). Гемоліз при синдромі HELLP призводить до
in HELLP syndrome would lead to decreased levels of haptoglobin rather than зниження рівнів гаптоглобіну, а не до підвищення рівня. Цей синдром призводить до
increased levels. This syndrome also results in thrombocytopenia rather than тромбоцитопенії, а не до тромбоцитозу.
thrombocytosis.

A 28-year-old woman presents to the clinic concerned because coarse hair has 28-річна жінка подається в клініку, оскільки на її верхній губі, підборідді і навколо
begun growing on her upper lip, chin, and around her nipples. She says she has сосків почали рости грубі волосся. Вона каже, що вона завжди мала надлишкову
always been overweight but has gained 9.1 kg (20 lb) in the past 3 months. She вагу, але за останні 3 місяці отримала 9,1 кг (20 фунтів). Вона сексуально активна з
is sexually active with her husband of 3 years and is not using contraception but чоловіком 3 роки і не використовує контрацепцію, але ніколи не вагітна. Менархе
has never been pregnant. Menarche occurred at age 12, and her menstrual сталося у віці 12 років, і її менструальні цикли відбуваються кожні 3–4 місяці і
cycles occur every 3–4 months and last 7–10 days. Her last menstrual period тривають 7–10 днів. Її останній менструальний період був 40 днів тому.
was 40 days ago.
Which of the following laboratory findings is most commonly associated with this Які з наступних лабораторних досліджень найчастіше пов'язані зі станом цього
patient’s condition? пацієнта?

A.Hypercortisolism Гіперкортизолізм

Гіперінсулінемія
B.Hyperinsulinemia
Гіпермагніємія
C.Hypermagnesemia
Гіперурикемія
D.Hyperuricemia Гіпотиреоз
Показати пояснення
E.Hypothyroidism Правильно!
Show Explanations Цей пацієнт представлений гірсутизмом, збільшенням ваги, нерегулярними
Correct! менструальними циклами та безпліддям. Ці симптоми свідчать про діагноз синдрому
This patient presents with hirsutism, weight gain, irregular menstrual cycles, and полікістозних яєчників (СПКЯ). СПКЯ є наслідком порушень гормонів (гіперсекреція
infertility. These symptoms suggest a diagnosis of polycystic ovarian лютеїнізуючого гормону є ознакою СПКЯ), які проявляються як ожиріння, гірсутизм,
syndrome (PCOS). PCOS results from hormone derangements (luteinizing олігоменорея і акантоз нігрікани (бархатиста гіперпігментація, часто навколо шиї і
hormone hypersecretion is the hallmark of PCOS), which manifest as obesity, аксіл).
hirsutism, oligomenorrhea, and acanthosis nigricans (velvety hyperpigmentation, СПКЯ часто асоціюється з резистентністю до інсуліну, що призводить до
often around the neck and axillae). гіперінсулінемії. СПКЯ також може викликати гіперглікемію і гіперліпідемію.
PCOS is often associated with insulin resistance, which leads Пероральні протизаплідні таблетки часто призначаються для зниження рівнів
to hyperinsulinemia. PCOS can also cause hyperglycemia and hyperlipidemia. циркулюючих андрогенів, які викликають гірсутизм і допомагають регулювати
Oral contraceptive pills are often prescribed to reduce the levels of circulating менструальний цикл. Кломіфен призначають жінкам, які бажають вагітності.
androgens that cause the hirsutism and to help regulate the menstrual cycle.
Clomiphene is prescribed for women who desire pregnancy. Гіпермагніємія спостерігається у пацієнтів з нирковою недостатністю. Ниркова
Hypermagnesemia can be seen in patients with renal failure. Renal failure недостатність характеризується зменшенням сечовиділення, набряками, втомою,
presents with decreased urine output, edema, fatigue, shortness of breath, and задишкою і сплутаністю, які не є симптомами, що спостерігаються у даного пацієнта.
confusion, which are not symptoms seen in this patient. Hyperuricemia is Гіперурикемія пов'язана з подаграми. Пацієнти з подаграми зазвичай мають історію
associated with gout. Patients with gout typically have a history of extremely надзвичайно болючого моноартикулярного артриту, гіперурикемії та підкіркових кіст
painful monoarticular arthritis, hyperuricemia, and subcortical bone cysts (tophi), (кістяк), яких у цього пацієнта немає. Гіпотиреоз має приріст ваги і порушення
which this patient does not have. Hypothyroidism presents with weight gain менструального циклу, але у пацієнта немає інших симптомів цього стану, таких як
and menstrual irregularities, but the patient does not have any other symptoms крихке волосся і непереносимість холоду. Подібним чином, гіперкортизолізм, який
of this condition, such as brittle hair and cold intolerance. спостерігається у пацієнтів з синдромом Кушинга, може викликати збільшення ваги і
Similarly, hypercortisolism, seen in patients with Cushing syndrome, can порушення менструального циклу, але цьому пацієнтові не вистачає багатьох інших
cause weight gain and menstrual irregularities, but this patient lacks many of the симптомів, таких як місячні фації, жирні подушечки і смуги живота.
other symptoms, such as moon facies, fat pads, and abdominal striae.

A 35-year-old woman (gravida 4, para 2, aborta 1) woman at 37 weeks' 35-річна жінка (gravida 4, пункт 2, aborta 1) жінка в терміні вагітності 37 тижнів
gestation comes to the emergency department because of the onset of надходить до відділення невідкладної допомоги через настання скорочень. Вона
contractions. She reports that her “water broke” about an hour ago while at повідомляє, що її «водяний пролом» близько години тому, коли вдома. Її єдиний
home. Her only prenatal care includes a first trimester visit and the only medical допологовий догляд включає візит першого триместру і єдину історію хвороби, яка
history on file includes two prior low transverse cesarean sections and a history містить два попередніх кесаревих розрізу, а також історію виборчого розширення і
of an elective dilation and curettage. On physical examination, she appears to вискоблювання. При медичному огляді вона виявляється в тяжкому стані, коли
be in distress with contractions occurring every 4 minutes. Her temperature is скорочення відбуваються кожні 4 хвилини. Її температура становить 37,4 ° C,
37.4°C (99.3°F), blood pressure is 133/70 mmHg, pulse is 92/min, and артеріальний тиск - 133/70 мм рт.ст., пульс - 92 / хв, частота дихання - 20 / хв. Вона
respiratory rate is 20/min. She is fully dilated and effaced. The baby is vertex повністю розширена і стирається. Дитина - вершина і станція -2. Пацієнт незабаром
and at -2 station. The patient soon delivers a healthy 6 lb 10 oz girl with Apgar доставляє здорову дівчинку з 6 фунтами 10 унцій з оцінками за шкалою Апгар 7 і 9 на
scores of 7 and 9 at one and five minutes, respectively. During delivery, the одну і п'ять хвилин відповідно. Під час пологів пуповина відходить від плаценти,
umbilical cord avulses from the placenta, after which the placenta is manually після чого плацента вручну видобувається на шматки. Кровотеча важко
extracted in pieces. Bleeding is difficult to control despite uterine massage and контролювати, незважаючи на матковий масаж і введення утеротоніків. Лікар записує
the administration of uterotonics. The physician records approximately 1000 приблизно 1000 мілілітрів крововтрати. Існують розриви статевих органів першого
milliliters of blood loss. There are first-degree genital lacerations. The uterus is ступеня. Матка м'яка вище рівня пупка.
soft above the level of the umbilicus.
Which of the following is the most likely explanation for the patient’s prolonged Яке з наступних причин є найбільш вірогідним поясненням тривалого кровотечі
bleeding? пацієнта?

A.Laceration of genital tract during delivery Розрив статевих шляхів під час пологів
Відсутність ефективного скорочення матки
B.Lack of effective uterine contraction
Передчасне відділення плаценти
C.Premature separation of placenta
Утримується плацентарна тканина
D.Retained placental tissue Розрив міометрія матки
Показати пояснення
E.Rupture of uterine myometrium Правильно!
Show Explanations Цей пацієнт є 35-річною жінкою з анамнезом двох попередніх кесаревих розтинів і
Correct! дилатацією і кюретажем, які мають надлишкове (> 500 мл) післяпологове кровотеча.
This patient is a 35-year-old female with a history of two prior cesarean sections Враховуючи її історію avulsed пуповини під час пологів, плаценти доставляється в
and a dilatation and curettage who is presenting with excessive (>500 mL) шматки, і м'який, не-контракт матки на іспит, найбільш ймовірна етіологія її
postpartum bleeding. Given her history of avulsed umbilical cord during delivery, післяпологового кровотечі зберігається плацентарної тканини.
a placenta delivered in pieces, and a soft, non-contracted uterus on exam, the
most likely etiology of her postpartum bleeding is retained placental tissue. Виникнення шнура може виникати спонтанно під час пологів або як наслідок аккрета
Cord avulsion can occur spontaneously during delivery or as a consequence of плаценти. В анамнезі цієї пацієнта травма матки (тобто історія кесаревого розтину,
placenta accreta. This patient’s history of uterine trauma (ie, history of C- дилатація та кюретаж, множинні терміни вагітності) ставить її під загрозу розвитку
section, dilatation and curettage, multiple gestations) puts her at risk of аккрета плаценти.
developing placenta accreta.
Placenta accreta exists on a spectrum of diseases characterized by placentas Акрета плаценти існує на спектрі захворювань, що характеризуються плацентами,
that are abnormally adherent to the myometrium. Placenta accreta is the most які аномально прилипають до міометрію. Акрета плаценти є найбільш м'яким
mild variant, whereby the placenta attaches to the myometrium without варіантом, за допомогою якого плацента приєднується до міометрію, не проникаючи
penetrating into it. Placenta increta and percreta refer to more adherent varieties в нього. Плацента інкрета і перкрета відносяться до більш прихильних сортів, де
where placenta villi penetrate into myometrium or through myometrium, ворсинки плаценти проникають в міометрій або через міометрій відповідно. В цілому,
respectively. Overall, this spectrum of disease is thought to arise from a цей спектр захворювання, як вважають, виникає з дефектного децидуального шару
defective uterine decidual layer, which is often a result of uterine trauma. матки, який часто є результатом травми матки. Як правило, плацента аккрета або її
Typically, placenta accreta or its variants can be diagnosed during prenatal варіанти можна діагностувати під час пренатального ультразвукового дослідження і
ultrasound and managed with a planned Caesarean-hysterectomy upon управляти планованим кесарево-гистерэктомией після пологів. Непослідовне
delivery. This patient’s inconsistent prenatal care likely allowed her case of допоміжне лікування пацієнта дозволило їй залишитися непоміченим.
accreta to go unnoticed.
Because the most important consequence of placenta accreta is postpartum Оскільки найважливішим наслідком аккрета плаценти є післяпологове кровотеча,
hemorrhage, complications of the disorder include Sheehan syndrome ускладнення порушення включають синдром Шихана (ішемія гіпофіза) або смерть.
(ischemia of pituitary) or death. Treatment involves removal of placental tissue, Лікування передбачає видалення плацентарної тканини, емболізацію матки або
uterine embolization, or emergency hysterectomy if bleeding cannot be екстрену гістеректомію, якщо кровотеча неможливо контролювати.
controlled.
The other choices are not the best answers for the following reasons: Інші варіанти не є найкращими відповідями з наступних причин:
 Lack of effective uterine contraction (uterine atony) is the most
common cause of postpartum hemorrhage. Uterine atony typically Відсутність ефективного скорочення матки (атонія матки) є найбільш поширеною
presents with a soft or boggy uterus and is managed with uterine причиною післяпологового кровотечі. Атонія матки зазвичай має м'яку або болотну
massage and uterotonics. Although this patient presents with a soft матку і керується матковим масажем і утеротонікою. Хоча цей пацієнт має м'яку
uterus, her bleeding is unresponsive to uterine massage or uterotonics. матку, її кровотеча не реагує на матковий масаж або утеротоніку. Крім того, той
Furthermore, the fact that her placenta was delivered in pieces makes факт, що її плацента доставлялася на шматки, призводить до виникнення кровотечі,
hemorrhage secondary to retained placenta tissue more likely. яка є вторинною до тканини плаценти.
 First-degree laceration of genital tract during delivery (involving only Перша ступінь розриву статевих шляхів під час пологів (за участю тільки шкіри
perineal skin) is unlikely to explain excessive (>500 cc) postpartum промежини) навряд чи може пояснити надмірне (> 500 см) післяпологового
bleeding. Risk factors for genital tract trauma include precipitous labor кровотечі. Фактори ризику для травми статевих шляхів включають стрімкі пологи і
and operative vaginal delivery. оперативне вагінальне розродження.
 Rupture of uterine myometrium is a cause of painful vaginal bleeding Розрив міометрія матки є причиною хворобливої вагінальної кровотечі до або під час
prior to or during delivery. It can occur in individuals with a history of a пологів. Це може статися у осіб з історією класичного кесаревого розтину або
classical C-section or invasive uterine surgery (submucosal fibroid інвазивної хірургії матки (видалення підслизової фіброми). Фізичний огляд може
removal). Physical exam can reveal palpable fetal parts and loss of fetal виявити пальпуючі частини плоду і втрату плодової станції до пологів. Плоди
station prior to delivery. Fetuses typically show non-reassuring heart зазвичай показують несприятливі ритми серця.
rates. Передчасне відділення плаценти описує механізм abruptio плаценти, який полягає у
 Premature separation of placenta describes the mechanism of відділенні плаценти від матки до пологів. Це не є причиною післяпологового
abruptio placentae, which is the separation of the placenta from the кровотечі. Відповідні фактори ризику включають травму, куріння, кокаїн і гіпертензію.
uterus before delivery. This is not a cause of postpartum hemorrhage.
Relevant risk factors include trauma, smoking, cocaine, and
hypertension.

A 20-year-old woman presents to the emergency department with severe 20-річна жінка подається до відділення невідкладної допомоги з важкою нудотою,
nausea, vomiting, and abdominal pain. She tells the emergency physician that блювотою і болем у животі. Вона повідомляє лікареві швидкої допомоги, що її
her last menstrual period was 3 months ago. The patient claims not to use останній менструальний період був 3 місяці тому. Пацієнт стверджує, що не вживає
alcohol but has been smoking a pack of cigarettes per day for 5 years. On алкоголь, але курив пачку сигарет на день протягом 5 років. При фізичному огляді
physical examination, abdominal swelling is observed. Laboratory tests indicate спостерігається набряк живота. Лабораторні дослідження показують, що рівень β-
that her β-human chorionic gonadotropin (β-hCG) level is 200,000 mIU/mL. людського хоріонічного гонадотропіну (β-hCG) становить 200000 мМЕ / мл.
Ultrasonography reveals numerous vesicles without any fetal parts in the uterus. Ультрасонографія виявляє численні везикули без будь-яких частин плоду в матці.
Which of the following is the most likely karyotype of the uterine contents?
Який з наступних варіантів є найбільш вірогідним каріотипом вмісту матки?

A.45,X0
45, X0
B.46,XX
46, XX

C.47,XXY 47, XXY

48, XXYY
D.48,XXYY
69, XXX
E.69,XXX Показати пояснення
Show Explanations Правильно!
Correct! Цей пацієнт відчуває нудоту і блювоту, а також аменорею. Її підвищені рівні β-hCG і
This patient is experiencing nausea and vomiting, as well as amenorrhea. Her абдомінальний набряк свідчать про те, що вона вагітна. Ультразвукові висновки
increased ß-hCG levels and abdominal swelling suggest that she is pregnant. везикул без частин плода допомагають підтвердити, що у цього пацієнта є
The ultrasound findings of vesicleswithout fetal parts help to confirm that this гестаційна трофобластна хвороба. Надмірне куріння збільшує ризик цього
patient has gestational trophoblastic disease.Excessive smoking increases захворювання. На додаток до hyperemesis gravidarum і високих рівнів β-hCG,
the risk for this condition. In addition to hyperemesis gravidarum and high levels пацієнти з цим захворюванням можуть мати матку, велику для гестаційного віку,
of ß-hCG, patients with this disease can also have a uterus that is large for гіпертиреоз і прееклампсію до 20 тижнів гестації. Оскільки ультразвук не показує
gestational age, hyperthyroidism, and preeclampsia before 20 weeks’ gestation. частин плоду, це повинно бути повний моль з каріотипом 46, XX або 46, XY. Це
Because the ultrasound does not show fetal parts, this must be a complete mole відбувається, коли гаплоїдна сперма повторюється в межах порожнього яйця.
with a karyotype of 46,XX or 46,XY. This occurs when a haploid sperm 69, ХХХ - каріотип часткової моль, який би містив частинки плоду на ультразвуку. 48,
replicates within an empty egg. XXYY відноситься до каріотипу синдрому XXYY, а ультразвук виявить плід з
69,XXX is the karyotype of a partial mole, which would have fetal parts visible багатоводдям і двосторонніми клубочками. 45, X0 - каріотип синдрому Тернера, а
on ultrasound. 48,XXYY refers to the karyotype of XXYY syndrome, and an ультразвук виявив би плід з аномаліями, такими як дефект серця, асцит і кістозний
ultrasound would reveal a fetus with polyhydramnios and bilateral гігрома. 47, XXY - каріотип синдрому Клайнфельтера, який не має специфічних
clubfeet. 45,X0 is the karyotype for Turner syndrome, and an ultrasound would ультразвукових особливостей, хоча може спостерігатися підвищена нуляльна
reveal a fetus with abnormalities such as a heart defect, ascites, and cystic прозорість і зменшений обсяг навколоплідних вод.
hygroma. 47,XXY is the Klinefelter syndrome karyotype, which does not have
specific ultrasound features, although increased nuchal translucency and
decreased amniotic fluid volume may be seen.

A 21-year-old woman comes for evaluation of a small, firm mass on her right 21-річна жінка приходить на оцінку невеликої твердої маси на своїй правій грудях,
breast, which she noticed 3 months ago. She says the mass increases in size яку вона помітила 3 місяці тому. Вона говорить, що маса збільшується в розмірах під
during menstruation. She has no personal or family history of breast cancer. On час менструації. У неї немає особистої або сімейної історії раку молочної залози.
palpation, the mass is firm, nontender, and mobile in the lower inner quadrant of При пальпації маса є твердою, непухлинною і рухливою в нижньому внутрішньому
the right breast. She has no overlying skin changes or nipple discharge. There is квадранті правої молочної залози. Вона не має перекритих шкірних змін або розряду
no palpable axillary or supraclavicular lymphadenopathy. The results of a urine сосків. Немає відчутної пахвової або надключичної лімфаденопатії. Результати тесту
pregnancy test is negative. на вагітність сечі негативні.

Histologic examination of the mass will most likely show which of the following
results? Гістологічне дослідження маси, швидше за все, покаже, який з наступних
результатів?

A.Branching fibrovascular core extending from a dilated duct


Розгалужується фиброзно-судинне ядро, що проходить від розширеного протоки

B.Fibrotic stroma around normal glandular tissue Фіброзна строма навколо нормальної залозистої тканини

C.Large cells with clear halos Великі клітини з чіткими ореолами

Паралельні масиви малих, мономорфних клітин з мізерною цитоплазмою


D.Parallel arrays of small, monomorphic cells with scant cytoplasm
Листи плеоморфних клітин інфільтрують сусідню строму
Показати пояснення
E.Sheets of pleomorphic cells infiltrating adjacent stroma
Правильно!
Show Explanations У цього пацієнта є невелика, тверда, непухлинна, рухлива маса в нижньому
Correct! внутрішньому квадранті правої грудей, яка змінюється в розмірах з менструацією.
This patient has a small, firm, nontender, mobile mass in the lower inner Відсутні зміни шкіри, розряд соску або лімфаденопатія. Ця презентація, швидше за
quadrant of her right breast that changes in size with menstruation. There are no все, вказує на діагноз фіброаденоми.
skin changes, nipple discharge, or lymphadenopathy. This presentation most
likely points to a diagnosis of fibroadenoma.
Фіброаденоми є найбільш поширеними доброякісними пухлинами молочної залози і
Fibroadenomas are the most common benign breast tumors and usually occur зазвичай зустрічаються у жінок віком від 20 до 35 років. Пухлини присутні як малі,
in women who are 20 to 35 years of age. The tumors present as small, firm, тверді, рухливі маси. При гістологічному дослідженні фіброзна междольковая строма
mobile masses. On histologic examination, fibrotic interlobular stroma is спостерігається навколо нормальних структур протоки і залози. Фіброаденоми часто
seen around normal duct and gland structures. Fibroadenomas are є одиночними, добре обмеженими, гумовими і безболісними масами. Вони реагують
frequently single, well-circumscribed, rubbery, and painless masses. They are на гормональні зміни під час менструального циклу і часто стають гіалінізованими і
responsive to hormonal changes during the menstrual cycle and often become можуть кальцифікувати, імітуючи карциноми молочної залози на мамографії.
hyalinized and can calcify, mimicking carcinomas of the breast on Діагностика стабільної фіброаденоми у молодої жінки зазвичай супроводжується
mammography. Diagnosis of a stable fibroadenoma in a young woman is УЗД; однак, біопсія вказана, якщо є будь-які сумніви щодо злоякісності або якщо
usually followed by ultrasonography; however, a biopsy is indicated if there is маса зростає.
any doubt about malignancy or if the mass is growing.

The other histologic findings are indicative of conditions that produce skin
changes, nipple discharge, or both—symptoms this patient does not have. Інші гістологічні знахідки свідчать про умови, що викликають зміни шкіри, виділення
сосків, або і те, і інше - симптоми, яких цього пацієнта не має.
Branching fibrovascular core extending into a dilated duct describes the
composition of intraductal papilloma.
Large cells with clear halos are histologic findings associated with Paget Розгалуження фиброзно-судинного ядра, що поширюється в розширений проток,
disease of the breast. описує склад внутрішньопротокової папіломи.
Parallel arrays of small, monomorphic cells with scant cytoplasm Великі клітини з чіткими ореолами є гістологічними висновками, пов'язаними з
infiltrating in lines are often associated with lobular carcinoma in situ or хворобою Педжета грудей.
invasive lobular carcinoma. Паралельні масиви малих, мономорфних клітин з мізерною інфільтрацією
Sheets of pleomorphic cells infiltrating adjacent stroma is a typical цитоплазми в лініях часто асоціюються з лобулярною карциномою in situ або
histologic finding that indicates infiltrating ductal carcinoma with a radiating інвазивною дольковою карциномою.
pattern. Листи плеоморфних клітин, що інфільтрують сусідню строму, є типовим гістологічним
висновком, який вказує на інфільтрацію протокової карциноми з радіаційною
картиною.

Вам также может понравиться